You are on page 1of 134
@d fe) ea} fe GIAI BAbTAP TRAC NGHIEM mh + LE BICH NGOC - NGUYEN VIET HOA LE HONG DUC - LE HUU TRI PHUONG PHAP GIAI BAI TAP TRAC NGHIEM HINH HOC 11 NHA XUAT BAN DAI HOC QUOC GIA HA NOI LOI NOI DAU Sif tau viét ctia phuong phdp thi trac ughiém d@ va dang duge ching mink ut nhiing nude cé nén gido duc tién tién trén thé’ gidi boi nhitng we diém nlue tinh Khdch quan, tinh bao qudt va tinh kinh 16: Theo chi truong cia BED&DT cdc trudng Dai hoc, Cao déng va Trung hoc chuyén nghiép sé chuyén sang hink thitc tuyén sinh bang phuong phdp tréc nghigm. Va dé 06 duge thoi gian chudn bi tot nhat, cdc bai kiém tra kién thie trong chuong tinh THCS va THPT ciing sé cé phén tréc nghiém dé céc em hoc sinh lam quen. Tuy nhién, viéc bién soan cdc cau hoi trac nghiém can tudn thi mét sé yéu cau co ban vé mat li ludn su pham va ¥ nghia dich thuc ctia cdc sé liéu thong ké. Ngoai ra, mt dé thi mén todn duge chém hodn toan dua wén két quad trac nghiém chac chéin sé chua phi hop voi hién trang gido duc cia nude ta boi nhiéu li do, tit dé dén 161 viée khéng dim bdo duce tinh khéch quan trong viéc dénh gid két qud hoc tép cia hoc sinh. Dé khéc phuc nhuge diém nay Nhém Cue Mon ching t6i dé xudt hong thyc hién hue sau: 1. Voi méi dé thi hoge dé kiém wa van tudn thi ding cau tric chung va diém trdc nghiém khong qué 3.5 diém 2. Oday, thong thudng ccic em hoc sinh sé phdi lua chon mot trong bén dap s6 va céin biet rétng 36 diém a ctia cau hoi nay duoc chia tam doi * Nein lua chon diing loi gidi re nghiém sé uhn duge > dic * Nei the hién ding Ii gidi ne ludn cho cau hoi sé nhgn duge $ dim con la. Daiy chinh la yéi 16 dé ddm bao tinh khiéch quan bai: 1. V6i nhiing hoc sinh chi mé mam dap én hode nhén duge né thong qua nhing yéit 1 ung quanh sé chi nbn duage 161 da © dim vik xc sud 25%. 2. V6i nhiing hoc sinh hiéu duoc noi dung cau hoi tit dé dinh hudng duge ede phép thit bang tay hodc bing mdy tinh fx ~ S70MS chée chén sé nhdn duge a ons ee a diém. Thi du vei cau hoi. (1 diém): Gidi phuong trinh vx =2~-x. A. x=0. B.x=5. Gx D. x=1. Cach J: Thue hién phép thi bang tay, cdc em sé can thit cho céc nghiém x=0,x=5,x=4,x= 1, cu thé: = V6i x =0, ta duge: VO =2~-0, mau thudn => x =0 khong 1a nghiém. * V6i x =5, ta duge: V5 =2-5= * VGi x = 4, ta duge: V4 =2-4 = Véix=1,taduge: vi 3, mau thudn => x = 5 khong 1a nghiém. 2, mau thudn => x = 4 khong la nghiém. ~ 1 = 1, ding = x = 1 1a nghiem. Vay, cdc em sé lua chon cau tra loi tric nghiem Ia x = 1. Céch 2: Sit dung méy tinh fx ~ 570MS bing céch lén lugt thyc hién: Nhap phuong trinh vx — 2 + x = 0 vao may tinh bang cach 4n: MALPHA)| M2 BRALPHA = Dé thir véi x = 0, ta a 5.236067973 = Dé thir voi x = 1, ta dn: cad 1 . [9 Vay, cdc em sé la chon cau tra Idi trac nghiém 1a x = 1. 3. Voi nhimg hoc sinh kha hom bid hién being viéc hiéu duge ngi dung cdu hdi va 6 thé thuc hién duoc mét phan cau héi nay dudi dang we lugn sé nhdn duoc 7 3a Khodng © + 2 = | 4. Cudi cing, véi nhing hoc sinh biét cdch thuc hién céu héi duéi dang tue ludn sé nhdn duoc a diém. Dua trén tw tudng nay, Nhém Cu Mon duoi su phu trach cia Lé Héng Ditc xin trén trong gidi thiéu tdi ban doc bé séch: GIAL BAL TAP TRAC NGHIEM TOAN THPT doThac siTodn hoc Lé Héng Dite chii bién. B6 sdch gém 6 cuén: Cuén 1: Giai bai ap tric nghiém Dai sé 10 1 Giai bai tap trac nghiém Hinh hoc 10 ii bai tap trac nghiém Dai s6 va Gidi tich 11 Giai bai tp tric nghiém Hinh hoc [1 Giai bai tap tric nghiém Dai s6 va Gidi tich 12 Giai bai tap trac nghiém Hinh hoc 12 Cudi citng, cho dit dd rat cé géng, nhung that khé tanh khdi nhitng thiéu st béi nhiing hiéu biét_ va kinh nghiém cén han ché, rdt mong nhan dugc nhitng ¥ kién déng gdp quy bau cia ban doc gan xa. Moi ¥ kién déng gép xin lién hé toi: Dia chi: Nhém tac gia Cy Mon do Th.S Todn hoc Lé Héng Dic phy trach S6 20 - Ng6 86 - Dutng To Ngoc Van - Quan Tay Hé - Ha Noi Dién thoai: (04) 7196671 hodc 0893046689 E-mail: cumon@hn.vnn.vn hoac lehongduc39@ yahoo.com. Ha Noi, ngay 10 théng 8 nam 2007 NHOM CU MON CHUONG L PHEP DGi HINH VA PHEP DONG DANG TRONG MAT PHANG §1. MODAU VE PHEP BIEN HiINH 1. KIEN THUC CAN NHO 1. PHEP BIEN HINH Dinh nghia 1: Phép bién hinh la m6t quy tac dé voi méi diém M cia mat phang xdc dink duoc mét diém duy nhdt M' cia mat phang, diém M’ oi la dnh cia diém M qua phép bién hinh dé. INéu ta kf hi¢u mot phép bién hinh nao dé 18 f thi: = -M=f(M). = Néu H 1a mot hinh nao dé thi tap hop cdc diém M' = f(M), v6i M € H, tao thanh hinh Ht, ta viét H' = f(A). 2. CAc vipu Vidu 1. Cho.duimg thing d. Voi méi-diém M, ta xéc M dinh M' a hinh,chiéu (vuong géc) cita M trén d thi ta duge mot phép bién hinh. () Phép bién hinh nay goi Ia phép chiéu vudng géc én “M duong thang d. Vidu 2. Cho vecto u , véi méi M=T__, (M’)" Ia ding hay sai ? A. Ding. B. Sai. Cho hai duéng thang song song d va d’ . Cé bao nhiéu phép tinh tién bien duong thang d thanh dutmg thang d’ ? A. Khong cs phép tinh tin nao. C.Chicé hai phép tinh tién, B. C6 duy nhat | phép tinh tién. D. C6 rat nhiéu phép tinh tién. Cho bén dung thing a, b, a’, b' trong dé a // a’, b //b’, a cat b. C6 bao nhiéu phép tinh tién bién dudng thing a va b Jan lugt thanh cdc dugng thang a’ va b’. A. Khong cé phéptinh tién nao. C. Chi cé hai phép tinh tién. B. C6 duy nhat | phéptinh tién. — D._ Co rat nhiéu phép tinh tién. Qua phép tinh tin T theo vecto u ¥ 0, duéng thing d bign thanh dudng thing d’. Trong trudng hgp no thi: a. dtringd'? A. d song song véi gid clia vecto u. B. d khong song song véi gid cia vecto u C. d vung géc véi gid ciia vecto u. D. Khong cé. b._d song song véi d'? ‘A. d song song véi gid cia vecto u. B. d khong song song véi gid cia vecto u. C. d vuong géc véi gid cla vecto u. D. Khong cé6. ec. deatd'? A. d song song véi gid cia vecto u. B. d khong song song véi gid cia vecto u. C. d vuong géc véi gid cila vecto u. D. Khong cé. :. Cho phép tinh tign T; theo u va phép tinh tién T; theo v. Véi diém M bat ki, T; bién M thanh diém M’, T- bién M' thinh M". Khéng dinh phép bién hinh bin diém M thanh M" 1a mot phép tinh tién a duing hay sai ? A. Ding. B. Sai. Bai 6: Cho dudng trdn (O) va hai diém A va B. Mot diém M thay déi trén dutng tron (O). Quy tich diém M' sao cho MM' + MA = MB Ia: A. 0) = Tyg). C. 0) = TO). B. (0) = T,; ((O)). D. ©) = Tz (0). Trong mat phing toa d6 Oxy, vdi a, a, b Ia nhimg s6 cho trudc. xét phép inh F bién méi diém M(x ; y) thanh diém M(x’, y’), trong d6: cosa -ysina +a y'=xsina+ycosa+b’ Cho hai diém M(x,; y,), N(x; y2) va. goi M’, N’ lan Iuot Ia anh cla M, N qua phép f. a. Hay tim toa dQ cita diém M’. A. M'(x,.cosa ~ y,.sinay; x,.sina + y,.cosc.). B. M((x,.cosa. + y,.sina; x,.sinat + y,.cosa). C. M'(x,.cosa ~ y,.sino; x,.sina— y,.cosa). D. M((x,.coso. + y,.sinax; x,.sinet ~ y,.cosa). b. Hay tim toa do cia diém N’. A. N'(%.cosot ~ y,.Sina; x,.sino + y,.cosct). B. N'(x.coso. + y2-sinor; Xz.sino. + y2.coset). C. N'(x,.cosa — y,.sina; x;.sina — y.cosa). D. N'(x,.cosa. + y;.sino; x,.sina ~ y,.cosa). c. Tinh khoang cdch d gitta M va N. A. d= V(x, - (:-y,)). CO. d= Vx, +x, +02 -y,)? B. d= J, +x, +0, +9, De d=, -x,? +0, ty). d. Tinh khoang cach d' giita M' va N’. A. d= J0,-x,)'+G2-y))- ©. d= On -x) +0, +y)- B. d= (x, +x, +, -y,)- De d= Jo, +x, +0, +9". e. Phép F cé phai Ia phép doi hinh hay khong ? A. C6. B. Khong. f. Khia =0, khang dinh F 1a phép tinh tign 1a ding hav sai ? A. Diing.- B. Sai. Bai 8: Trong mat phing toa do) Oxy, xét cic phép bién hinh sau day: = Phép bién hinh F, bien moi diém M(x, y) thanh diém M'(y 3-x). * Phép bién hinh F bien moi diém M(x, y) thanh diém M'(Q2x; y). Trong hai phep bin inh tren, phép nao [a phép ddi hinh. Cho luc giéc déu ABCDEF tam O. Tim anh cia AAOF qua phép tinh tién theo vecto AB. A. AABO. —-B. ABCO. ~—C. ACDO._—iOD«y. ADEEO.. Bai 10: Cho tit giéc ABCD ¢6 M,N, P; Q lan lugt 1a trung diém cdc CP, DA. Ching minh ring tit giéc ABCD [a hinh binh hanh khi va chi MP +NQ= 5 (AB + BC+ CD + DA). Bai 11: Trong mat phing Oxy, cho v(2;-1) va diém M(-3; 2). Anh cia diém M qua phép tinh tién theo vecto v 1a diém c6 toa do nado trong cdc toa do sau day ? A. (5:3). B. (1; 1). Cc. Cl D. D. (15-1). Bai 12: Cho dudng thing d c6 phuong trinh 2x — y + 1 = 0. Dé phép tinh tién theo vécto v bién d thanh chinh né thi Vv ‘phai 1a vecto nao trong cdc trudng hgp sau : A v=). BoV=(-l. G v=(h2). De v=Ch2). Bai 13: Trong mat phing Oxy, cho diém A(-1; 2) va dudng thang d c6 phuong tinh 3x + y + 1 =0. Goi A’ vad’ la anh cla A va d qua phép tinh tién theo vecto v(2; 1). a. Tim toa do ciia diém A’. A. A(3) °° BAG D.C. AG3 Dd. D. ACH 3). b. Tim phuong trinh ciia dutng thang d’, A. (d): 3x +y-2=0. ©. (d): x + 3y -2=0. B. (d’): 3x +y- 6 =0. D. (d’): x + 3y -6=0. Bai 14: Trong mat phing toa do Oxy cho veto v = (-1; 2), AG; 5), BI; 1) va dutng thang d cé phuong trinh x — 2y +3 =0. Goi A’, B’ theo thif ty 1a anh ota A,B qua phép tinh tién theo v. a. Tim toa d6 ciia diém A’. A. A(Q2;7). Be A(7;2). Ce A'(7;-2). D. A'(2;-7). b. Tim toa d6 cia diém B’. A. BY-3; B. B(-2;3). CC. B(2;3). —-D. B(3;-2). . Tim toa dQ cita diém C sao cho A'la nh ciia C qua phép tinh tién theo v. A.CU;3). BB. C(3;4). GC. (453). DD. CGD. d. Tim phuong trinh ca dudng thing‘d’ 1a anh cia d qua phép tinh tién theo v. A. (d): 2x-y+4=0. C. @):x-2y +4 =0. B. (d’): 2x -y +8 =0. D. (d’):x = 2y +8=0. Trong mat phing Oxy, cho dudng trdn tam 1(3; -2), ban kinh 3. Viet phuong trinh dudng tron dé A. (x= 3) +(y +2) =9. C. (x+3P +(yt+2P=9. B. (x +3 +(y-27% =9. Dz (x-3P +(y-2)7' =9. b. Viét phuong trinh anh cia dudng trdn (1, 3) qua phép tinh tién theo vecto v(—2; 1). A. (x-1P + (y+ DP =9. C. (x- 1 +(y- IP = 9. B. (x+y +(y+1P=9. Dz (x+ LP +(y- DP =9. Bai16: Tim phuong tinh cua dudng won (C,) la anh cia dudng tron (C): (& + 2 + (y — 1? = 4 qua phép tinh tign vecto v (2; 1). A. xP +(y-1P=4. CG. P+ (y- 2) =4. B. xP +(y+ 1 =4. D. xv +(y+2)'=4. Bai 17: Hay tim vecto ¥ (a; b) sao cho khi tinh tign dé th y = f(x) = x" + 3x + 1 theo ¥ ta nhan duge dé thi ham so y = g(x) = x" — 3x? + 6x - 1. A. v(1;-2). Be ¥(-1:2). C. v(132). DY ¥(-15-2). Bai 18: Cho hai diém A, B va dudng tron tam O khong c6 diém chung véi dutmg thing AB. Qua- méi diém M chay tren dudng tron (O) dumg hinh binh hanh MABN. Chimg minh ring diém N thu6c mot dudng trn xdc dinh. §3. PHEP DOI XUNG TRUC 1. KIEN THUC CAN NHG 1. PHEP DOI XUNG TRUC Nhde lai: Diém M' duac goi la déi ximg voi diém M qua duéng théng a néu a la duéng trung truc ctia doan thing MM’. Truong hop dac biét, néu M ndm tren a thi ta. xem M d6i xing voi chinh né qua a. Dinh nghia 1: Phép doi xing qua duéng thang a la phép bién hinh bién méi diém M thanh M' déi ximg voi M qua duéng thang a. Phép d6i xing qua dudmg thing a thuémg duoc ki higu la B,. Vay: M'= B,(M) © a [a trung truc doan MM’. Chi ¥: Taluon cd: « M'=D,(M) > M= DM). *"Mea=>D(M)=M. Dinh lé: Phép doi xtmg truc a phép ddi hinh. 2. TRUC BOI XUNG CUA MOT HiNH Dinh nghia 2: Dudng thang d duce goi la truc doi xing ctia hinh H néu phép doi xing truc Dy bién H thank chinh né, tite la DACA) = H. 1. BAL TAP TRAC NGHIEM VA TULUAN Bai 19: Qua phép d6i xtmg truc D, (a l& truc déi xtmg), dudmg thang d bién thanh dudng thang d’. Hay tra lai cdc cau hoi sau: a. Khi nao thi d song song véi d' ? A. d/fa. B. d=a CG dia. D. g(d,a)=45". b. Khi nao d vung géc véi d’? A. d//a. B. d=a CG. dia D. g(d.a)=45". Bai 20: Cho hai dutmg thing cat nhau d va d’. C6 bao nhieu phép déi xting truc bién dudng thang d thanh dudng thang d'? A. Khong 06 phép doi xtimg truc nao. C. Chicé hai phép d6i xting truc. B. Céduy nhat | phép déi ximgtruc. D. Cé rat nhiéu phép d6i xtmg truc. ‘Trong céc hinh sau day. hinh nao cé 4 truc déi xing. A. Hinh binh hanh. C. Hinh thoi. B. Hinh chit nhat. D. Hinh vuéng. Trong céc ménh dé sau day, ménh dé nao SAI ? ‘A. Hinh gém hai dudng tron khong bang nhau cé truc d6i xing. B. Hinh gém mot dudmg tron va mot doan thang tuy ¥ c6 truc déi ximg. C. Hinh gém mot dudmg tron va mot dudng thang tuy ¥ cé truc déi xing. D. Hinh gém mot tam gidc can va dudng trdn ngoai tiép tam gidc dé cd truc d6i ximg. Bai 23: Hinh vung co may truc déi ximg ? A.l B.2 C4 D. v6 sé 4: Trong mat phing Oxy, dudmg thang d cé phuong trinh 3x — 2y +1 =0. Anh cia dutmg thing d qua phép d6i xtig truc Ox c6 phuong trinh 1a: A. 3x+2y+1=0. C. 3x +2y-1=0. B, -3x + 2y+1=0. D. 3x-2y+1=0. Trong mat phang Oxy cho dudng thang (d): 3x - y + 2 = 0. Viét phuong trinh duéng thang (d’) IA dnh cla d qua phép d6i xting truc Oy. A. 3x-y+2=0. C. 3x-y-2=0. B. 3x+y+2=0. D. 3x+y-2=0. Bai 26: Viet phuong trinh anh ciia cdc dudng tron sau qua phép d6i xing c6 truc Oy: a. (C): x? +y’—4x + Sy+1=0. A. xty?t+4x+Syt] B. x +y°-4x +5y+1=0. C. xv +y?-4x-Sy+1=0. D. x +y?—4x-Sy+1=0. b. (C): x? +y? + 10y-5=0. A. xt+y?+10x-5=0. C. x+y? 10x-5=0. B. x? +y?+l0y-5=0. D. x? +y?- l0y-5=0. wai 27: Trong mat phing Oxy cho A(1; -2) va B(3; 1). Tim anh cia A qua phép d6i ximg truc Ox. A. A(1;-2). B. A(-1:-2). C. A32). De ACI 2). b. Tim anh cia B qua phép déi xting truc Ox. A. B(3; 1). B. BY-3; 1). C, BG;-1). -D. BY-3; -1). c. Tim anh ciia duong thing AB qua phép d6i xtmg truc Ox. A. 3x-2y-7 C. 3x+2y-7=0. B. 3%-2y-1=0. D. 3x-2y+7=0 Bai 28: Cho géc nhon xOy va mot diém A nim trong géc 46. Hay xéc dinh diém Buren Ox va diém C tren Oy sdo cho tam giée ABC c6 chu vi nhé nhat. 29: Cho dutmg thing d di qua hai diém phan biét P , Q va hai diém A, B im vé mot phia doi vi d. Hay xdc dinh trén d hai diém M, N_ sao cho MN = PQ va AM + BN bé nhat. Bai 30: Cho hai diém B va C cé dink nam trén dudng tron (O; R) va diém A thay di tren dudng trdn 6. Hay ding phép déi xting truc dé chtmg minh ring truc tam H cia tam gidc ABC nam trén mot duéng trdn cé dinh. 31: Cho hai dutng tron (O; R), (O'; R') va mot dutmg thang d. a. Tim hai diém M, N én luot nam trén hai dudng tron 6 sao cho-d la trung truc ciia doan thing MN. b. Xéc dinh diém I trén d sao cho tiép tuyén IT cla (O ; R) va tiép tuyén cia IT cia (O' ; R') hop thanh céc géc ma d Ia mdt trong céc duéng phan gidc cia géc d . Bai 32: Cho AABC cé BC = a, CA = b, AB =c, p 18 nifa chu vi, h, 18 d6 dai dudng cao tit A. Chimg minh ring h,< Jp(p—a). Bai 33: Cho AABC*n6i ti¢p trong dudng tron (O, R). Goi H 1a tc tam cia tam gidc. a. Chimg minh ring cdc diém déi xtmg cla H qua cdc canh cla AABC nam trén dudng tron (O, R)..Tir dé suy ra céc duémg tron (HBC), (HCA), (HAB) va (O) bang nhau. b. Goi 0}, O,, O, lan lugt la tam céé dutmg tron (HBC), (HCA), (HAB). Ching minh AABC va AO,0,0, bang nhaw. Bai 34: Cho hai diém A(1; 1) va BG; 3) a. Tim trén truc hoanh diém P sao.cho téng cdc khoang céch tit P t6i. cdc diém A E Bla nhé nhat. 5 az A. PAS +0) B. PAS 50) Cc. PaCS 50). D. Pa(5 30). b. Tim gid tri nhé nhat do, A. V5. B. 2V5. Cc. 35. D. 4V5. Bai 35: Tim truc d6i xting cita dé thj ham s6'y = at : +1 A. y=x41. B. y=x+2. C. y=x+3. D. y=x+4 § 4. PHEP QUAY VA PHEP DOI XUNG TAM 1. KIEN THUC CAN NHO 1. PHEP QUAY Dinh nghia: Trong mat phdng cho diém O cé'dinh va géc luong gidc a khong déi. Phép bién hinh bién méi diém M thanh diém M' sao cho OM = OM' va (OM, OM’) = a duge goi la phép quay tam O véi gdc quay a. Ki higu QS, hay Q(O; a). Dinh tf: Phép quay 1 mot phép ddi hinh 2. PHEP BOIXUNG TAM Dinh nghia: Phép 46i xing qua diém O 1a mot phép ddi hinh bién méi diém M thanh M’ d6i xtmg véi M qua O, tic ld OM + OM' = 0. Ki higu Dg hay So. Cha ¥: 1. Phép quay tam O, géc quay o = 180° IA phép doi ximg tam O. 2. Trong mat phing -véi he truc toa do Oxy, cho diém I(a; b). Phép d6i xing tam D, bién diém M(x; y) thanh diém M(x’; y’) véi: x'=2a-x : =2b-y" Tam di xing cia mot hink: Dié™G —.oc goi la tam doi xing ciia hinh Ht néu phép d6i ximg tam Do biéh hinh H thanh chink né, titc la Do(H) = H 3. UNG DUNG CUA PHEP QUAY Bai tovan 1: Cho hai tam giéc déu OAB va OA‘B' nhu hinh vé. Goi C va D lan lugt I& trung diém cia céc doan thing AA' va BB’. Chimg minh ring OCD 1a tam gidc déu Gidi Xét phép quay Q tam O véi géc quay bing mot géc lugng gide (OA, OB). Ro rang Q bién doan AA’ thanh doan BB’. Do dé: OC = OD va COD = 60°. Vay, ta duge AOCD déu. Bai toxin 2: Cho dutmg tron (O ; R) va hai diém A, B c6 dinh. Vi méi diém M, ta x4c: dinh diém M' sao cho MM' = MA+MB. Tim qu tich diém M' khi diém M chay trén (O ; R). , 13 Gidi Goi 1 1a trung diém cla AB thi I cé dinh va Do dé, MM’ = MA+MB khi va chi khi MM! = 2MI, tic a MM’ nhan I lam trung diém hay phép d6i xting tam D, bién diém M thanh M’. Vay khi M chay trén dutmg tron (O ; R) thi quy tich M’ 1a anh cla dung tron dé qua B,. Néu ta goi O' la diém d6i ximg cua O qua diém I thi quy tich cla M’ la dudng tron (O ; R). Bai toan 3: Cho hai dudng tron (O ; R) va (O, ; R,) cat nhau tai hai diém A. B. Hay dung mot dudng thang d di qua A cat (O ; R) va (O, : R,) lan luot tai M va M, sao cho A Ia trung diém cua MM,. Gidi Gia sit ta da dung duge dudng thang d thoa man yéu cdu bai ton. Goi D, 1a phép déi xtmg qua A thi D, bién diém M thanh diém M, va bign dudng tron (O; R) thanh dudng tron (O' ; R). Vi M nam trén (O ; R) nén M, nam trén (O': R). Mat khéc, M, lai nim tren (O, ; R,) nén M, [a giao diém khac A cia hai dudng tron (O' ; R) va (O, ; R,). Tir do, suy ra cach dung: = Dumg dutng trdn (O' ; R) doi xing véi (O ; R) qua A (O' 1a 4iém doi xting véi O qua A). "Lay giao diém M, ciia hai duémg tron (O' ; R) va (O, ; R,), M, khée A. = Dudng thang d 1a dudng thang di qua A va M,. 11. BAL TAP TRAC NGHIEM Bai 36: Trong cdc hinh sau day, hinh nao khong cé tam déi xing. A. Hinh gém mot dudng trdn va mot hinh chit nhat ngi tigp. B. Hinh gém mot dudng tron va mot tam gidc déu néi tiép. C. Hinh luc gidc déu. D. Hinh gém mot hinh vung va dudng tron noi tigp. Bai 37: Cho hinh vuong ABCD tam O. Xét phép quay Q cé tam quay O va phép quay . V6i gid tri nao sau day cia @, phép quay Q bién hith vudng ABCD thanh chinh n6 ? x x x x 9-2. Bo=®, Co=®. Doak. Ars era v5 a2 Bai 38: Khang dinh "Néu mot hinh nao dé cé hai true ddi xting vudng géc véi nhau thi hinh dé cé tam doi xtmg " 1a ding hay sai ? A. Ding. B. Sai. Bai 39: Cho hai tam gidc vudng can QAB va OA'B' c6 chung dinh O sio cho O nam trén doan thing AB' va nim ngoai doan thang A'B. Goi G va G' Kin luot 1a trong tam cdc tam gide OAA' va OBB. Hay xéc dinh dang cia AGOG’. 14 A. Can, B. Vuong. —C. Vuongcan. D. Déu. Bai 40: Cho phép d6i xing tam Dy va duémg thing d khong di qua O. C6 thé dug d’ ma chi sirdung compa mot lin va thudc thing ba lin hay khong ? A. C6. B. Khong. ‘Trong mat phang Oxy, dudng thang (d): 3x - 2y - 1 = 0. Anh cia duéng thing d qua phép doi xig tam O cé phuong trinh Ia: A. 3x+2y+1=0. C. 3x+2y-1=0. B. D. 3x-2y-1=0. @ Oxy, cho diém A(-1; 2) va dudng thang d cé phuong . Tim anh cla A va d: xting qua truc Oy. A. A‘(1; 2) va 3x-y-1 C. A(2; 1) va 3x-y-1=0. B. A(1;2)va3x+y-1=0. D. A(2: 1 va3xt+y-1=0. b. Qua phép doi xtmg qua géc toa do. A. A"(1; -2)va3x-y-1=0. 9 C. A"GI; 2)va3x-y-1=0. B. A"(1; -2)va3x+y-1=0. D. A'CH; 2)va3x+y-1=0. Bai 43: Trong mat phang toa do Oxy cho diém A(-1; 3) va dudng thang d cé phuong trinh x —- 2y +3=0. a. Tim anh cia A qua phép doi xing tam O. A. A(-1;-3). B. A(-1;3). C. A\(1;-3). De A‘C:; 3). b. Tim anh ciia d qua phép d6i xting tam O. A. x+2y+3=0. C. x-2y+3=0. B. x+2y-3=0. D. x-2y- . Bai 44: Trong mat phing Oxy, cho dudng tron tam 1(3; -2), bin kinh 3. Viét phuong trinh anh cua dutng tron (I, 3) qua phép doi xting qua géc toa do. A. (x4 39 + (y= 2759. C. (x= 3) +(y +2) =9. B. (x +3) +(y +2" =9. Dz (x- 37 +(y-27 =9. Bai 45: Cho AABC néi tiép trong dudng tron (O) va mot diém M thay déi tren M, la diém déi xting voi M qua A, M, la diém déi xing voi M, qua B, M; lla diém doi xing voi M, qua C. . a, Chimg t6 rang phép bién hinh F bién diém M thanh M, 1 mot phép di xtmg tam. 'b. Tim quy tich diém M,. Bai 46: Trong mat phang toa do Oxy, cho dutng thang (A): Ax + By +C = 0 va diém (a, b). Phép d6i xtmg tam B, bién dudng thing (A) thanh dudng thing (A)... Viét phuong trinh ctia (A’). A. (4’): Ax + By + C- 2aA - 2bB =0. B. (A’): Ax - By + C - 2aA - 2bB =0. C. (A): Ax - By - C- 2aA - 2bB =0. D. (A): Ax + By - C- 2aA - 2bB =0. Bai 47: Cho AABC cé cac dinh dugc ki hiéu theo huéng an, dung 6 ngoai tam gidc dy hai hinh vung ABDE va BCKF. Goi P Ia trung diém canh AC. H 1a diém doi ximg ciia D qua B, M 1a trung diém doan FH. a. Xéc dinh anh cia hai vecto BA va BP trong phép quay tam B, g6c 90". b.. Ching minh ring DF = 2BP va DF vung géc vdi BP. Bai 48: Cho luc gidc déu ABCDEF tam O. Tim anh ciia tam gidc AOF: a. Qua phép doi xtmg qua dudng thing BE. A. AAOB. B. ACOD. _C. AEOF. D. ABOC. b. Qua phép quay tam O géc quay 120°. A. AAOB. B. ABOC. C. ADOC. —_D, AEOD. Bai 49: Trong mat phing toa do Oxy cho diém A(2; 0) va duémg thing d cé phuong trinh x + y—2=0. a. Tim anh cia A qua phép quay tam O géc 90°. A. A‘(0; 2). B. A‘(0; 3). C. A‘; 0). D. A‘(2; 0). b. Tim anh ciia d qua phép quay tam O géc 90°. AU x+y+2=0. : C. x+y-2=0. B. x-y+2=0. D. x-y-2=0. 50: Trong mat phing Oxy, cho dim A(-1; 2) va dutmg thing d cé phucng tinh 3x+y+1= a. Tim anh cua A qua phép quay tam O géc quay 90°. A. A(Q2;1). Be A'C2;-1). C. AC; -2). De As 2). b. Tim anh ciia d qua phép quay tam O géc quay 90°. A. (d)):x-3y+1=0. C..(d)): x + 3y +1=0. B. (@’):x-3y-1=0. Dz (4): x + 3y- §5. HINH BANG NHAU L KIEN THUC CAN NHO Dinh li: Néu AABC va AA'B'C 1a hai tam gidc bang nhau thi cé phép déi hnh bién AABC thanh AA'B'C. Dinh nghia: Hai hinh goi la bang nhau néu cé phép doi hinh bién hinh ray thanh hinh kia. I. BAI TAP TRAC NGHIEM VA TULUAN Bai Si: Chimg td rang néu AABC va AA'BCC [a hai tam gidc bang nhau thicé phép ddi hinh bién AABC thanh AA'BC. Bai $2: Khing dinh "Hai hinh chit nhat cing kich thuée thi bing nhau’ la diing hay sai? A. Ding. B. Sai. 16 . Khang dinh "Hai tir gidc c6 cdc cap canh tuigng tmg bang nhau va mot cap dudng chéo tuong img bang nhau thi bang nhau" la ding hay s2i ? A. Ding. B. Sai. b. Khang dinh "Hai tt gidc c6 cdc cap canh tuong tng bing nhau va mot cap g6c tuong tng bang nhau thi bang nhau" 1a dung hay sai ? A. Ding. B. Sai. c. Hai tit gidc c6 cdc cap canh tuong tg bang nhau thi cé bang nhau hay khong ? A. C6. B. Khong. Bai $4: Da gidc Idi n canh goi IA n-gidc déu néu tat c& cdc canh cia né bing nheu. Chimg t6 rang hai n-gidc déu bang nhau khi va chi khi ching cé canh bang nhau. Hinh H, gém ba dudng trdn (O, ; r,), (O2 § Fr), (Oy 5 ry) dOi mot tiép xtic ngoai vdi nhau. Hinh H, g6m ba dudng tron (1); 1). (yi ty). (Iai 3) doi mot ti€p xtic ngoai véi nhau. Khang dinh hai hinh H, va H, bang nhau la diing hay sai? A. Ding. B. Sai. 6: Cho hai hinh binh hanh. Hay chi ra mét dubng thang chia mdi hinh binh hanh dé thanh hai hinh bang nhau. A, Dudng thing di qua hai tam ciia hai hinh binh hanh. B, Dudng thang di qua hai dinh cua hai hiah binh hanh. C. Duong thang di tam cita hinh binh hanh thé nat va mot dinh cita hin binh hanh con lai. D. Dudng chéo cia mot trong hai hinh binh hanh dé. Bai 57: Trong mat phing toa do Oxy, cho hai parabol (P) va (P’) lin lugt cé phuong trinh y = ax? va y = ax? + bx +c (a ¥ 0). Khang dinh "Hai parabol 46 bang nhau" 1a dting hay sai ? A. Ding. B. Sai ai $8: Chimg minh rang : Néu mot phép doi |..h bién tam giéc ABC thanh tam gidc A’B’C’ thi nd ciing bién trong tam cua tam gidc ABC tuong tng thanh trong tam cua tam gidc A’B’C’. Bai 59: Cho hinh chit nhat ABCD. Goi E, F, H, K, O, I, J lan luot 1a trung diém cia céc canh AB, BC, CD, DA, KF, HC, KO. Khang dinh "Hai hinh thang AEJK va FOIC bing nhau" 1a dting hay sai ? A. Ding. B. Sai. B3i60:_ Trong mat phing Oxy cho cdc diém A(-3; 2), B+; 5) va C(-1; 3). a. Tim anhciaa A, B, C qua phép quay tam O géc -90°. A. A\(2; 3), B(3; 1) va C(5; 4). C. A'S; 4), BY(2; 3) va C(3; 1). De A'G: 1). BG; 4) va C(2; 3). B. A(2; 3), BGS: 4) va CGD Le / a5g4 | 7 b. Goi AA,B,C, la anh cia AABC qua phép di hinh cé duge bang cach thuc hién hén tiép phép quay tam O géc -90" va phép déi xtmg qua truc Ox. Tim toa d0 cdc dinh cla AA,B,C,. A. A,(2; -3), B,(3; -1) va C,(5; -4). B. A,(2; -3), B,(5; ~4) va C\(3; -1). C. A,(S; -4), B,(2; -3) va C\(3; -D. D. A,(3; -1), B,(5; -4) va C\(2; -3). §6. PHEP VITU 1. KIEN THUC CAN NHO 1. DINH NGHIA Dinh nghia: Cho mot diém O c6 dinh va mét sok khong doi k # 0. Phép bién hinh bién mdi diém M thanh diém M' sao cho OM’ =kOM duge goi la phép vi ne tdm O, ti s6°k.. Ky higu la V§ hac V(O; k). 2. CAC TINH CHAT CUA PHEP VITU Dinh li 1: Néu phép vi ty ti s6 k bign hai diém M va N thanh hai diém M' va N' MN va M'N' = |k|MN. Dinh li 2: Phép vi tu bién ba diém thing hang thanh ba diém thang hang va khong lam thay déi thi ty cua ba diém thang hing dé, Hé qua: Phép vi tu ti sk: = Bién dudng thing thanh dutng thang song song (hoac tring) véi dudng thing d6, bién tia thinh tia. = Bién doan thang thanh doan thing va'd6 dai duoc nhan lén véi |k|. «= Bign tam gidc thanh tam gidc déng dang véi ti s6 déng dang 1a |k |. = Bién géc thanh géc bang n6. 3. ANH CUA DUONG TRON QUA PHEP VITU Dinh li 3: Phép vi tu bién dug tron thanh dutmg tron. Tam vi he ctia hai duéng tron: Cho hai dudmg tron (1); R,) va (Lyi R,) voi R, # F. C6 hai phép vi tr Vg, va V6, bién (1,; R,) thanh (I,; R,). Hai tam vi tu O,, O, va ti s6 k duge xdc dinh nhu sau: * ke2Bt (k > 0 thi goi 1a tam vi tu ngoai, k < 0 thi goi la tam vi tu trong). 2 * 0,0; dtrén dung thang [,1, va si Wh U. BAITAP TRAC NGHIEM VA TULUAN Bai 61: Cac phép sau day 6 phai [a phép vi turkhong ? a. Phép déi xing tam. A. C6. B. Khong. ». Phép doi xting truc. A. C6. B. Khong. c. Phép déng nhat. A. C6. B. Khong d. Phép tinh tién theo vecto khac 0 A. C6. B. Khong. 2. Cac khang dinh sau day cé diing khong ? hep vi ty ludn c6 diém bit dong (tue la diém bin thanh chinh n6), A. Ding. B. Sai. b. Phép vi tu khong c6 thé c6 qua mot diém bat dong A. Ding. B. Sai. c. Néu phép vi ty co hai diém bat dong phan biét thi moi diém déu bat dong. A. Ding. B. Sai. Bai 63: Cho hai dudng thing song song d va d’. Cé bao nhiu phép vi ty véi ti ‘0 k= 100 bién dung thang d thanh dudng thing d’ ? A. Khong c6 phép nao. C. Chi cé hai phép. B. C6 duy nhat mot phép. D. Co rat nhiéu phép. a Cho dudng tron (O ; R). Tim menh dé sai trong cdc ménh dé sau day: A. C6 phép tinh tién bin (O ; R) thanh chinh nd. B. C6 hai phép vi tu bin (O ; R) thanh chinh nd. C. C6 phép doi xg truc bién (O ; R) thanh chinh né. D. Trong ba ménh dé trén cé ft nhat mot ménh dé sai. Bai 65: Trong cac ménh dé sau day, ménh dé nao sai ? A. Tam vi ty ngoai ciia hai duéng tron nim ngoai dudng tron 46. B. Tam vi ty trong cia hai duémg tron khong nim gita hai tam ciia hai dutng won. C. Tam vi tu trong cia hai dudng tron ludn thudc dudng thang néi tam cla hai dudmg tron. D, Tam vj ty cia hai dudng tron c6 thé la diém chung cia cd hai dudng tron, Bai 66: Phép bién hinh nao sau day khong cé tinh chat: "Bién mot dudng thang thanh duéng thang song song hoac tring vi no"? A. Phép tinh tién. C. Phép doi xtmg truc. B. Phép doi xtmg tam. D. Phép vi ty. Bai 67: Trong cdc ménh dé sau day, ménh dé nao sai ? A. . Phép ddi hinh [A mot phép déng dang. B. Phép vj tu 1a mot phép déng dang. C. Phép déng dang la mot phép doi hinh. D. C6 phép vi ty khong phai 1a mot phép doi hinh Bai 68: Trong cdc phép bién hinh sau, phép nao khong phai Ia phép ddi hinh A. Phép chiéu vudng géc4én mot dutng thang. B. Phépdéngnhat. C. Phépvitytis-1. D. Phépddi xing tuc. Bai 69: Trong cdc ménh dé sau, ménh dé nao sai ? A. Phép tinh tién bién dudng thang thanh dudng thing song song hoac trimg v6i nd. B. Phép d6i xing truc bin dudng thing thanh dudng thing song song hoac tring vdi ndé. C. Phép d6i ximg tam bién duémg thing thinh du’mg th§ng song song hoac tring vdi né. D. Phép vi ur bién duéng thing thanh dudng thing song song hodc tring véi nd. Bai 70: Trong céc ménh dé sau day, ménh dé nao SAT? A. C6 mét phép tinh tién bién moi diém thanh chinh n6. - B. C6 mot phép d6i xting truc bién moi diém thanh chinh n6. C. C6 mot phép quay bién moi diém thanh chinh n6. D. C6 mot phép vj ty bién moi diém thanh chinh né. Bai 71: X4c dinh tam vj ty trong va tam vi ty ngoai cia hai dudng tron trong cdc trudng hgp hat duéng-trdn tiép xtic ngoai v6i nhau; hai dudng tron tiép xtic trong v6i nhau; mot dung tron chtta dudng tron kia. Bai 72: Goi F la phép bién hinh c6 tinh chat sau: Véi moi cap diém M, N va anh M’, N' ciia chting, ta luén cé6 M'N' = kMN, trong d6 k Ia mOt s6 khong déi khéc 0. Hay chimg minh rang F la phép tinh tién hoc phép vi ty. Bai Cho hai dudng tron (O) va (O') cat nhau tai A va B. Hay dung qua A mot ig thang d ct (O) 6 M va cat (O')GN saocho M Ia trung diém cla AN. Bai 74: Cho dudng tron (O ; R) va diém I c6 dinh khdc O. Mét diém M thay déi tén |__ dung tron. Tia phan giéc cla g6c MOI cit IM tai N. Tim quy tich diém N. Bai 75: Cho hai dudng tron (O) va (O’) cd ban kinh khdc nhau, tiép xtic ngoai ~~ v6i (O) va (O') lan luot tai B va C. Ching minh rang dutmg thing BC lun di qua mot diém cé dinh. Bai 76: Cho dudng tron (O) cé dutng kinh AB. Goi C Ia diém d6i xtmg véi A qua ~~ qua B va PQ 1a dutng kinh thay d6i cita (O) khéc dung kinh AB. Dudng thang CQ cat PA va PB lan lugt tai M va N. a. Chimng min:: Ang Q 1a trung diém cia CM, N [a trang diém cia CQ. b. Tim quj tich cdc diém M va N khi dung kinh PQ thay déi. Bai 77: Cho dutng tron (O ; R) va diém A c6 dinh. Mot day cung BC thay déi ca P:R) 6 c6 d6 dai khong déi NC = m. Tim qu§ tich céc diém G sao cho GA+GB+GC=0. Cho hai dudng trdn (0) va (0') cé ban kinh khdc nhau, tiép xtic ngoai (©) va (0) lan lugt tai B va C, Chimg minh ring duéng thing BC luon di qua mot diém cé dinh. ai Cho dutng tron (O; R) dudng kinh AB. Mot dudng tron (0') tiép xiic v6i (O) va doan AB lan luot tai C, D, cat duéng thing CD tai (O; R) tai I. Tinh do dai céc doan thang Al va BI. 20 Bai :. Cho hai dudng tron (O) va (O’) cat nhau tai A va B. Hay dimg qua A mot dung thing d cit (0) & M va ct (0) 6N sao cho M la trung diém cia AN. Bai8J: (Tr 29): Cho AABC cé ba ae nhon va H Ja truc tam. Tim-anh cia AABC qua phép vi ty tam H, ti 96 4 = ‘Baig2: “a. Ching minh ring khi thuc hién lién tiép hai phép vi ty tam O ti s6 k, va k, sé duge mot phép vi tu tam O. b. Tim tam ti cu cla phép vi tu trong cau a). A. k,. B. k,. C. kk). k ky D. § 7. PHEP DONG DANG 1 KIEN THUC CAN NHO | 1. DINH NGHIA PHEP DONG DANG Dinh nghia: Phép bién hinh F goi la phép déng dang ti s5'k (k > 0) néu véi hai diém M va N bat ki va dnh M’ va N’ ctia ching ta lun cé M'N' = KMN. Dinh Ii: Moi phép déng dang F ti sO k (k > 0) déu Ia hgp thanh cia mot phép vi tu V ti s6 k va mot phép doi hinh D. Hé qud: Phép déng dang ti s6 k: * Bién ba diém thang hang thanh ba diém thing hang va kh6ng lam thay déi thf ty ciia ba diém thang hang d6. = Bién dutmg thang thang dudng thang, bién tia thanh tia. = Bién doan thang thanh doan thang va d6 dai dugc nhan Ién véi k. = Bién géc thanh géc bing nd. 2. HAIHINH DONG DANG Dinh nghia: Hai hinh goi la déng dang voi nhau néu cd phép dong dang bién hink nay thank hinh kia. U. BAI TAP TRAC NGHIEM Bai 83: Trong céc ménh dé sau day, ménh dé nao sai 2 A. Hai dudng thing bat ki luon déng dang. B. Hai dudng tron bat ki luon déng dang. C. Hai hinh vuong bat ki luon déng dang. D. Hai hinh chit nhat bat ki luon déng dang. Bai 84: Chimg t6 ring néu phép déng dang F bién AABC thanh AA'BC' thi trong tam, truc tam, tam dudng tron ngoai tiép AABC [an lugt bién thanh trong tam, tryc tam, tam dudng tron ngoai tiép AA'BC.. 21 Bai 85: Khang dinh "Cac da gidc déu cé cing sO canh thi déng dang véi nhau” 1a diing hay sai ? A. Ding. B. Sai. Cho hinh chit nhat ABCD, AC va BD cat nhau tai I. Goi H, K, L va J luot la trung diém cla AD, BC, KC va IC. a. Khang dinh “Hai hinh thang JLKI va IHDC déng dang voi nhau" [a diing hay sai? A. Ding. B. Sai. b. Tinh ti s6 déng dang. at, B. t. c. 2. D. 2 4 2 3 Trong mat phing Oxy cho diém I (1; 1) va dutng tron tam I ban kinh } phuong trinh cla duéng tron Ia anh cla dudng tron trén qua phép déng dang cé dugc bang céch thuc hién lién tiép phép quay tam O, géc 45” va phép vi urtam O, ti s6 V2. A. XP +(y- 27% =8. C. x +(y-2)'=4, B. 4 (y-2)=6. Dz x?+(y-2)'=2. Bai 88: Trong mat phang toa d6 Oxy, cho dudng tron tam I(1; ~3), ban kinh 2, Viét phuong trinh anh cla dudng tron (I; 2) qua phép déng dang c6 dugc tir viéc thuc hién lién tiép phép vi ty tam O ti sé 3 va phép d6i xtmg qua truc Ox. A. (x-3% +(y-9% =36.- GC. (x +3)? + (y- 9)? = 36. B. (x -3) +(y-9) = 18. Dz. (x-3P +(y +9) = 18, 22 BAP SO TRAC NGHIEM - LOI GIAI TU LUAN Dap so trac nghiém A. L6i gidi ng ludn: That vay, ta c6: M’=T_,(M) <> MM’ = v @ M'M =-v @M=T _(M). Bai 2: Dép so tréc nghiém D. | > S é > m a & Loi gidi tw ludn: Moi phép tinh tién T theo vecto v = A’ ¢ d' déu bién dudng thing d thanh d’. : Dap 36 tric nghiém C. Léi gidi 1 ludn: Ta nhan thay: * Goi {A} =anbthivia/a nena’ b= (A'}. © Goi (B} =ab' thi via//a' néna’ b= (B)}. Khi dé: « Voi phép tinh tign T, bién a, b theo thé ty thanh a va bi. « V6i phép tinh tién Or bién a, b theo thir ty thanh a’ va b’. Vay, t6n tai hai phép tinh tién bién dudmg thang a va b lan lugt thanh céc dung thang a’ va b'. Dap sé trac nghiém a). C;b). B;c). D. a. d tring d’ khi d song song véi gid ciia vecto u. b. d song song vdi d’ khi d khong song song véi gid cita vecto u. d va d’ khong bao gid cit nhau. Dap 86 tréc nghiém A. Loi gidi ng lugn: Dat a= + Vv ta 06 nhan xét: MM" = MM’ + MM" Vay. phép bign hinh bign M thanh M’ la mot php tinh tién T theo vecto : Dap sé trac nghiém A. Loi gidi ne ludn: Tit gid thiét, taco: MM'= MB- MA = AB. Tic 1a M' Ia anh cia diém M qua phép tinh tién theo A vecto AB. : Vay. quy tich diém M' 1a dudmg trn (0') Ia anh cia dudng tron (O) qua phép tinh tign theo vect AB . Dap sé trac nghiém a). A;b). Ayc). A;d). Aye). A; f). A. a. Tacé: M(x,.coso — y,.sina. + a; x, sina + y,.cosa + b), b. Tacé: N'(x,.cosa ~ y,.Sina + a; x;.Sina + yz.cosa + b), 23 c. Tacé: d= Vix, d. Tacé: (dP = yi w MIN'Y = [(x,.cosar — y,.sina) — (x,.cosa. — y,.sina)}? + + [(X,.sinat + y,.cosa) ~ (x,.sina + y,.cosct)? = [(x, — x,).cosa - (y) — y,)-sina]? + + [(x, — x,).sina + (y, — y,).cosa]? = (XK, — x,)°.cos’a. + (y) — y,)?.sin’a. + +(x, —X,)?.sin’a + (y, — y,)2.cos’a, Xp — X,)?.(cos’as + sin’) + (y2 — y,)2(sin’a: + cos*x) =x +27)? d= (a, -5 +0, @ e. Tit(1) va (2) suy rad =d' (hay MN = M'N)). Vay, phép bién hinh F bao toan khoang cach gitta hai diém-bat ki nén theo dinh nghia no Ia mét phép di hinh. £ Véiee=0,ta thay: [*=*OO- ysis +8 y=xsind+yco0+b ly'=y+b 2 Fla phép tinh tién. a. Phép bién hinh F, bién hai diém M(x,; y,), N(x; y2) thanh hai diém My; -X,), N'(Y3i -%2). Khi d6, ta c6: MN= (J, -y,)? +x, +x.) =¥0e, 8)? #2 -? = Vay, F, JA mot phép di hinh, b. Phép bién hinh F, bién hai diém M(x,; y,), NOx; y2) thanh hai diém M((2x,; y1), N'(2x35 Y2)- Khi dé, ta c6: MN'=x, —2x,)° +(y2 ~y,)° =V40, - x)? nae y)) #MN. Vay, F, khong [& mot phép dai hinh. Bai 9: Dap sé trdc nghiém B. Loi gidi te ludn: Ta c6: TA)=B T{(0)=C; T5(F)=0 Do d6 T= (AAOF) = ABCO. B 0: Thyc hién phép tinh tién T3.:D 4 E. Khi d6 tit giéc BCED I hinh binh hanh, vi P [a trung diém cia CD nénP cling 1a trung diém cita BE. Do dé ta c6: MP = 3 AE < > (AD + DE) = 5 (AD + BC). a Dau bang chi xay ra khi va chi khi: A, D, E thing hang <> AD // BC. Chimg minh tuong tv ta ciing cé: 24 c NO< F(AB +CD). (2) B Dau bang chi xay ra <> AB // CD Cong theo vé (1), (2), ta duge: M 1 MP +NQs 5(AB+BC+CD+DA). G) © oe Vay dé 06 (*) thi dau “=” xy ra & (3) ¢9 dau “=” xay ra tai (1) va (2) {eer CD ABCD [a hinh binh hanh, BC//AD i: Dap sé trac nghiém C. Loi gidi ty ludn: Ta biét rang phép tinh tién theo vecto vas b) bién diém M(x; y) thanh diém M'(x’; y’) vi: x=x+a x'=-3+2= > y=2-1=1 1 = M(-1; 1). =y+b Bail2: Ddp sé trdc nghiém C. Léi gidi tet ludn: Dé phép tinh tign theo vécta Vv bién d thanh chinh né thi vecto V phai c6 gid song song véi dudng thang d. Nhan xét ring dudng thing d c6 vecto chi phutong a (1; 2). Do d6, chting ta chon dap an C. Bail3: Dap sd trac nghiém a). A;b). B. a. Diém A thanh diém A'(1; 3), b. Duong thing d thanh dudng thing d’ c6 phuong trinh duge xéc dinh bang cach: Méi diém M(x, y) € (d) 1a anh cla mot diém My(x, yo) thuge (4) qua phép tinh tién theo vecto v (2; 1), ta c6: Mo(tpyoe(d) [P%0t¥o+1=0 3xy typ +1=0 aoe X= xy =2 © {xy =x-2 MoM y-youl yo=y-l => 3(K~2)+(y- 1) +150 3x +y-6=0. a) Phuong trinh (1) chinh fa phuong trinh ciia (d’ Bail4: Dép so trac nghiém a). A;b). Bic). Cd). D. te ludn: Ching ta biét rang phép tinh tién theo vecto v(-1; 2) bién. diém aMGx y) thanh diém M'(x': y') voi: x=x-1 y=y+2" 1=3-1=2 Ps » . {x =Xq -1=3-1 . Vi A(X y’) thi: A\(2; 7). a. (x's y') thi: (ee na? (2; 7). 25 b. V6i B(x’; y') thi: { vy 4 = C(4: 3). Ya =Yor2 Ye=3 d. Méi diém M(x, y) € (d’) 1a anh ciia mot diém M,(X y,) € (d) qua phép tinh ti€n theo vecto v (—1; 2), ta c6: c. Gia st Cle: vot | ka =tc-l fies Mo(oryore(d) — [R0~2¥043=0 [Xo ~2¥0 +350 — X-Xy=-l 2 {xg =xel =v ° ¥~-Yo=2 Yo=y-2 = (x +1)-2y-2) +3 =x -2y +8=0. “ Phuong trinh (*) chinh f& phuong trinh cia (d’). Bai 15: Dap sé trdc nghiém a). A;b). A. a, Taco ngay: (I, 3): (x - 3)? + (y +2) =9. b. Voi phép tinh tign theo vecto v(—2: 1) thi: T-(, 3) =, 3) v6i,T, = v = (15 -D. Tir d6 suy ra: (1,, 3): (X- L? +(y + 12 =9. Bai 16: Dap so trac nghiém C. Bail7: Dap sé trac nghiém B. Loi gidi i ludn: Tit gia thiét, ta c6; g(x) = f(x +a) +b x) — 3x? + 6x — = [(x +a) + 3(x +a) +1] +b = x'+3ax? + 3(a?+ Ix ta’ 3at¢14+b 2 foo . Vay, ta duge g(x) = f(x ~ 1) +2, tlic 8 c6 vecto ¥(-1; 2). Bai 18: Tir hinh binh hanh MABN, ta cé: Y {NN MN = AB > N= TS (M). ¥) (Y) ‘Tir gia thiét M chay tren (O), suy ra: DX B A BAi 20: Dap so trac nghiém C. Léi gidi ty ludn: Dé chinh 1a hai duémg phan gide ciia géc tao bai d va d’. Loi gidi tu ludn: Vi hinh vudng ABCD sé c6 bén truc d6i ximg, d6 Ia: + Hai dung chéo AC va BD. + Hai dung thing a va b di qua tam va vuong géc véi cdc canh di dién. 26 nghiém B. nghiém C. finh vudng c6 4 truc déi xtmg, bao gém: “Hai duémg chéo. Hai duéng thang theo thi ty di qua trung diém hai canh déi dién. Loi giai ne luan: Méi diém M(x, y) € (d’) la anh cua mot diém M(X, yo) € (d) qua Mo(Xonyo)E(@) —— [3xq -2y9 #1 =0 phép doi xting truc Ox, tacé: 4x =x, 3 4Xq =X Yo Yo=-y => 3x +2y+1=0. (*). Phuong trinh (*) chinh Ja phuong trinh cia (d'), Bai 25: Dap s6 tric inghiém Dz gidi tur ludn: Ta c6 thé lua chon mot trong hai cach sau: Cach 1: (Lay phuong phap tir bai tap 1): Lay hai diém AG (@), Khi dé ta lan Iugt c6: © Dy(A) = A'(O; 2); Do(B) = 5). . a = (d’) di qua hai diém a va'B, tite Ia: eye (a): dee Sm? cucte Ful * qua B'(-1;5) zl 5- Cach 2: Méi diém M(x, y) € (d’) 1a anh cua mot diém M(x), Yo) € (@) qua phép {Mo(xo.¥o € (A) 3xy -Yy t2=0 i xtmg truc Oy, ta 66: { 2) va BCL; 5) thude -Xo 2 4X =-x ly Yo youy = 3x+y-2=0, (*). Phuong trinh nc) chinh 1 phuong trinh cia (d’). Bai 26: Dap s6 wdc nghiém a). A;b). a. Ta c6 thé Iya chon mot trong hai a ‘sau: Cach 1: Méi diém M(x, y)e(C,") Ia anh chia mot diém Mg(Xo. yo) € (C,) qua phép d6: xtmg cé truc Oy, ta co: 2,2 x} +} 4x9 +5yo +1=0 Mo (Xo, ¥o) €(Cy) oe 22h) Oy la tung truc cia MyM. 2 youy =>(xP +y-4-x) + Syt Ls0r+y44xtSy41=0. (*) Phuong trinh (*) chinh la phuong trinh ca (C,’). v37 Céich 2: Dutng tron (C,) e6 tam 1,2: =f) ban kinh R, = “3? Duong trn (C') déi xing v6i dudng tron (C,) qua Oy c6: 3 tam I, '(~: 4 = (C\): (K+ 2) + (y+ oD a bankinhR, e(Cy:xty + ax + 5y41=0 . 27 b. Tacé thé lua chon mot trong hai cdch’sau: Cach 1: Méi diém M(x, y)e(C;’) 1a anh cia mot diém Mg(Xo. yo) € (C,) qua phép di xtmg cé truc Oy, ta cé: 2, Xo + Yo + 10yg -5=0 {Metisse oo R = {Xy =-x Oy la trung truc cha MyM yor, => (xP +y? + l0y-S=0ex? +y’+ 1l0y-5=0. (**) Phuong trinh (**) chinh I phuong trinh ciia (C;’). Céch 2: Dudng tron (C,) 6 tam I,(0; -5) va bén kinh R, = ¥30 . Dudng tron (C;}) d6i xting véi duong tron (C,) qua Oy c6: tam I, '(0;-5) bankinhR,'= (Cy): x + y+ l0y-5=0. Bai 27:_ Dap s6 trdc nghiém a). C;b): C;c). C. Ta lan lugt c6: » Do,(A) = A'(1; 2). * Do,(B) = B(3; -). = Do(AB) = (A‘B)) di hai wu diém A’ va BY, te 1a: (AB): {me (1:2) x-1_ y- Jip 24 SF = (L30F eros (AB): T= Ay (AB) 3x4 2y-7=0. Bai 28: Trudc tien: = Goi A, [A diém d6i xing voi A qua Ox. = Goi A, 1A diém déi xtig voi A qua Oy. = N6iA,A, cat Ox, Oy theo thit ty tai B va C. Ta sé di chtmg minh ABC cé chu vi nhé nhat. ‘That vay: CVaggc = AB + BC + CA = A,B+ BC + CA, =A,A;)2 A,B! + BC + CA, = AB + BC + CA =CV asec Bai 29:_ Gia sit xc dinh dugc hai diém M, N sao cho MN = PQ va AM + BN bé nh&t. Thuc hién phép tinh tién theo vecto PQ, ta duge: A 7 Tig(A)= A's Tyg (M) =N suy ra: AM = A'N va AM + BN = A'N + BN. : Téi day, bai toan dugc chuyén vé viéc tim diém N P MQ |//N thudc d dé A'N + BN bé nhat. Goi A’, 1a diém déi xtmg véi A‘ qua (d), ta oe A'N + BN=A'\N + BN2 A'B. Vay, ta duge: (A') N + BN) in = A, B, dat duge khi A’, N, Bthing hang. Téi day, chung ta c6 cdch dung: hs, 28 + Thuc hién phép tinh tign theo vecto PQ, ta duge Tg (A)=A- + Lay diém A’, 1a diém doi ximg voi A’ qua (d). + N6iA' Beat d tai N. + Thuc hién phép tinh tién theo vecto QP, ta dugc T()=M. Bai 30: Gid sit AH cat dudng tron tai H’. a Ta cé: H'CB = H'AB = HCB => AH'CH can tai C (AN) => BC la dutng trung tc cia H'H > H = D,(H)). AZT lo NE? Va vi H' € (O) nén H € D,,((O)). Bai 31: ) a. Thuc hién phép d6i xing truc Dy, tacd: Dy(O)) = (O)). Khi d6: » Xéc dinh giao diém M cia (O') véi (0). = Xéc dinh diém N 1a diém d6i xting voi M qua d. Bai 32: Dung dudng thing (d) qua A song song cdi BC. Goi B,, C, Hin lugt 1 diém doi xtmg cia B, C B, qua dudng thang (d), ta cd: AC, = AC=b, AB, = AB=c BB, = CC, = 2AH = 2h,, BB' 1 BC. Xét AAB,C, ta cé: AB, + AC>B,C = BC? + BB? b+c2> ya? +4h? eh? s 41 +6)?=2"]=pip-a) eh, pip —a), dem. Nhén xét: Trong 18 giai wren dé chtmg minh tinh chat h,< /p(p—a) chting ta 43 sit dung mot phép d6i xing truc (d), tuy.nhién diéu déng phai minh hoa duge & day 1a tai sao lai chon truc (d) nhu vay, diéu nay c6 thé duge ly gidi so luge nhu sau: -" * Viéc lya chon phép déi xing truc (d) sé nhan duge phén tir trung gian quan trong 1a B,C, phan tr nay duge biéu dién thong qua b va ¢ hodc qua a vah, , tir dé nhan dugc m6i lien hé gitta a, b, ¢ va h,. = Céc em hoe sinh hay ta Idi them cau hdi "C6 t6n tai phép doi img truc khéc (d) khéng va néit c6 thi tinh chat ciia phép d6% xing dé la gi?” Bai 33: a. GoiH,, H,, H; lén lugt la diém d6i xiing cia H qua céc canh BC, CA, AB. * Xét phép d6i xing truc BC, ta duge: Sc: BHC + BH|C=> BH,C = BHC. Vi: BHC + BAC = 180" <> BH|C + BAC = 180° => ttt gige ABH,C noi tiép duge => H,€(O, R). * Ching minh tuong ty, ta dugc: H,¢(O, R), Hy€(O, R)- 29 Tacé: ~*~ “A S.aci((HBC)] = (H,BC) = (O, R) Stcaj((HCA)] = (H,CA) = (O, R) oO S.,s\((HAB)] = (H,AB) = (O, R) = (HBC) = (HCA) = (HAB) = (O,R), dpem B b. Goi M,N, P lain lugt 1a trung diém cae canh BC, CA, AB. Ta c6 ngay: Siac: OF O4; Sica; OM Os: S,,y: OF O, Suy ra: Oj) MN = BA 02053 2PM = AC > 0,0, = AC, AO,0,0,, dpem. Bai 34: Dép sé'tréc nghiém a). B;b). B. Loi gidi 1 ludn: Goi A, la diém doi xing véi A qua Ox suy ra AI; -1). Goi P, = (A,B) Ox, suy ra P,(x; 0), A, B thing hang o> [ARARE a, Paya? 3/2 y=0 y=0 0 Tacé PA + PB=PA, + PB2 A,B. Vay, tathdy: (PA + PB), A,B= YB-2 +341? =2V5, at duge khi: A,, P, B thang hang <> P = P, =; 0). [r= 3. * ly= => Pk +0). 5: Dap sé wae nghiém B. ; Loi gidi.n¢ ludn: Dutmg thang y = x + 2 1a true déi xing cita dé thi ham s6 <> cdc dudng thang vudng géc véi dudng thing y = x + 2 (co dang y = -x +m) néu cat dé thi tai A va B thi trung diém I cia AB phai thudc duémg thing y =x +2. Hoanh do giao diém A, B Ia cdc nghiém cia phuong trinh: Xl Lg tmee-(m-2x-1-m=0 a) x+1 xp =m-2 Gid sit xq, xp 1a cdc nghiém cita (1) thi: {xk e , KaXp=-m-l Goi 1 1a trung diém cia AB, ta c6: I: yy =-X) +m Thay toa d6 cia I vao phuong trinh dudng thang y = x + 2, ta duge: ae = m=2 42.500 (luon ding) © I thudc dutng thing y= x +2 Vay, dudng thing y = x + 218 truc d6i xing cila dé thi ham s6. Dap sé trac nghiém B. ti ludn: Boi mot tam gidc déu thi khong c6 tam doi xing. Dap s6 trée nghiém D. Léi sidi tu ludn: V6i phép quay Q2 (ABCD) = DABC. Bai 38: Dép so tréc nghiem A. Goi O 1a giao diém cua hai truc ddi ximg a va b cla hinh (H). Voi diém M bat ki thuéc (H), ta co: D,(M) = M, = OM=OM, va 0:= G2, D,(M,) =M, = OM, = OM, va Gs = Ou, OM =OM, 6: - TOR, GOG' = 90" va. 0G’ = 0G. 6 trac nghiém &. Co thé thuc hién duge, cu thé: iém A trén d, diing thudc thang dung tia AO, * Ding compa dumg dutng tron (O; OA), dudng tron nay cat dudng thang d tai B va tia AO tai A’. 1g ndi A’ vdi B’ ta dugc duéng thing d’ céin dung. ic nghiém B ne lugn: Mi diém M(x, y) € (d’Ha anh cia mot diém M(%u Yu) € (A) qua phép doi ximg tam O, ta cd: Mo(Xo.¥o) € (4) 3xq ~2yg 150 x => 3(-x) - 2(-y)- 1 =0 yo=ry =>-3x + 2y-1=0. “ Phuong trinh (*) chinh la phuong trinh ctia (d’). Dap sé trac nghiém a). A;b). B. a. V6i phép d6i xing qua truc Oy. = Diém A thanh diém A'(1; 2). Xo 2 4X 31 + Dudng thing d thanh dudng thing d, c6 phuong trinh dugc xéc dinh bang c4ch: Mdi diém M(x, y) € (d,) 1a anh cila mot diém My(Xo Yo) thudc (d) qua phép d6i ximg qua truc Oy, ta c6: Mo(Xo¥o) € (4) 3x9 +o +1=0 Xo =-x © dxy =-x Yo=y yo=y => 3(C-x) +y+1=03x-y-1=0. 2) Phuong trinh (2) chinh la phuong trinh ctia (d,). b. V6i phép d6i xtmg qua géc toa do. = Diém A thanh diém A"(1; 2). = Duong thing d thanh dudng thing d, cé phuong trinh dugc xdc dinh bang cach: Mdi diém M(x, y) € (d,) 1 anh cla mot diém My(xo, yu) thudc (d) qua phép déi ximg qua géc toa dé, ta c6: Mo(Xo,yo€(d) [3X9 +¥o +1=0 Xp =7X © }Xy =x Yo=-y Yor-y => 3(-x) + (-y) + 1=0 3x4+y- Phuong trinh (3) chinh IA phuong trinh cia (d,). Bai 43: Dp so trdc nghiém a). C;b). D. a. V6i diém A(-1; 3) thi Do(A) = A'(1; -3). b. Tacé thé lua chon mét trong hai céch sau: Cach 1: Lay hai diém A(-3; 0) va B(1; 2) thudc (d), khi 46 ta lan lugt c6: = Do(A) = A'(3; 0). * Do(B) = BY(-1; -2). @) qua A'(3;0) + Do(d) = (d’) di qua hai diém A’ va BY tite la: @): {er eo od) 9 (4): 2x — dy -6 = 0 e (4): x= 2y-3=0. Cach 2: Méi diém M(x, y) € (d’) 1a anh cia mot diém M(%. yo) € (A) qua phép Mo(%o,¥o) €() Xy~2yo +3=0 d6i xing tam O, ta cé: {x =-xy © {xy =-« ae youry 2 © (-x) - 2(-y) +3 =0 <> x-2y-3=0. “« Phuong trinh (*) chinh la phuong trinh cia (<’). Bai 44: Dap sé trac nghiém A. Loi gidi tu ludn. V6i phép doi xtmg qua géc O thi: Do((I, 3))=(1,, 3) voi I(—3 2). Tir d6 suy ra: (1, 3): (x +3)? +(y-2)?=9. 32 4 trung diém MM,, ta c6: ABC Ja hinh binh hanh = BC = Ico dinh. Vay F 1a phép doi xing tam 1 b. Vir M,=D,(M) va M € (O) = M, € (O') = B((O)). Bai 46: Deip so tric nghiém D. Lot gid ue ludn: Voi mdi diém M(x: ¥q) € (A) (tite 1 Ax, + By, + C = 0), suy ra ton tai diém M"(xz y) € (A) sao cho: Jx=2a-x x, =2a-x 2S ly =2b-y 2b-y Do do, dudng thing (.) sé c6 phuong trinh: (AN): AQa = x) + Bb = y) + C= 0. (1): Ax + By - C- 2a = 2bB = 0. M Fy K BH (cing bang BD) i |, BH) = 90" yc yo" ae) a c => Q" (A) =H & Q3” (BA) = BH p H Vi O2" (A) =H. 2" (C) = E nen Q" (AC b. ViP La trung diém cla AC nén theo tinh chat cha phép quay ta e6 dinh cita P qua phép quay tren trung ddiém M cia HE. bP = BM BP LBM” saa” Do dé: Qi" (BP) = BM => { Mat khie: BM = 1 DE va BM // DF = BP = 5 DF va DF L BP. 2 A B Bai 48: Dap sd trac nghiém a). Bb). D. a. Tacé: DyfA)=Cy Dy(O) =O; Dy(F) =D Do dé Dy.(SAOF) = ACOD. b. Tacd: QI" (A) =E: QI” (0) =0: Qi?" (Ey =D 20" Do dé Q!?"" (AAOF) = AEOD. Bai 49: Dap so wdc nghiéma). Azb). B. 0 A‘eOy Vi A(2; 0) € Ox nén: Qo yea: | A((0; 2). a. Vi A(2;0) € Ox nén: Qu) (A) joa-oa' > (0; 2). b. Nhan xét ring (d) > Ox = {A}. do dé: Q%” (a) = (a): {we an x6L ng (0) OX = A}. d0.d6: QB = (I Fay" Ta lan lugt thay: = (d) vung géc véi (d) nén (d') c6 phuong trinh: (d'): x -y + C= 0. 33 = (d’) di qua A’ nén:0-2+C=0e>C=2, Tir do, ta nhan duge (d’): xy +2 =0. Bai 50: Dap s6 trac nghiém a). B; b). B. a. Voi phép quay tam O géc quay 90" diém A thanh diém A'(x; y) c6 toa do . _ [OA=O0A, (r+ 2axtey? thoa man: 0? j—— (OA,OA') = 90' OA.OA'=0 (4. f = a=90" ane Seg) -x+2y=0 [x=-2&y=-1 b. Duong thang d thanh dudng thang d’ c6 phuong trinh duge xdc dinh bang cach nhan xét ring A € (d), do dé: : di qua A’ 0 . d) = (d’): oe (i Lid) ‘a lan luot thay: = (d') vuéng géc vi (d) nén (d") 6 phuong tinh: (d’): x - 3y +C =0. 2 (d’)diqua A'nen: -2- 3(-1)+C=0@C=-1. Tir d6, ta nhan duge (d’): x - 3y ~ 1 =0, Bai 51: Goi F 1 mot phép bién hinh bién méi diém M than diém M' sao cho néu pCA +qCB (p,q € R) thi CM'=pC'A'+qCB.. Ta di chimg minh F la mot phép di hinh céin tim. ‘That vay, voi diém N va F bign N thanh N’, tic A néu CN =kCA +1CB (k,Le R) thi CN'=kCA'+iCB Khi dé, ta lin luot c6: MN. = CN -CM =(kCA +ICB)-(pCA +qCB) = (k - p)CA + (1-q)CB MNS UMN SLES BI = pICA? + = qyCBE + 2¢k ~ py = QCA CB. ay i ( pCa’ A'+tqC C'B') . fenton “(I (MN =(MNY=[(k- pC a'+d-geB FP = (k-p)CA” + (1 q)°CB? + 2k - p= Voi gid thigt AABC = AA'BC (nén CA = CA CB = CA’ CB) nén tit (1) va (2) suy ra: MN = MIN’ & F [& mot phép doi hinh. Mat khadc, ta nhan thay phép dai hinh F bién A, B, C lan Iugt thanh A’, B, C, tite fa bién AABC thanh AA'BC pC B CA’ CB’, (2) CB = CB va 34 Bai $2: Dap sé trac nghiém A. Loi gidi ne ludn: Gia sit hai hinh chit nhat ABCD va A'B'CD' cé AB = A'B' va BC=BC. Goi O, O' theo thif ty 1a tam cua hai hinh chit nhat ABCD va A'BCD'. Ta cé: AABC = AA'BC = tén tai mot phép doi hinh f dé AA'BIC' = f(AABC) = A’‘O' = f(AO) = D'= f(D). Vay, ta due: A'BCD' = f(ABCD) => hai hinh chit nhat ABCD va A’B'CD' bang nhau. i $3: Dap sé trde nghiéma). A:b). Asc). B. a. str hai duémg chéo cua hai tt gidc 16i ABCD va A'B'C’D' cat nhau theo thif ty tai O va O'. Gia st AC = A'C, ta c6: AABC = AA'BC' (c.c.c) => t6n tai mot phép doi hinh f dé AA'BC' = f(AABC) = A‘0' = f(AO) => D'= f(D). Vay, ta duge: A'BCD' = f(ABCD) => hai hinh ur gidc 16i ABCD va A'BCD bang nhau. b. Gia sit hai dutng chéo ctia hai tit gide 16i ABCD va A'B'C’D' cat nhau theo thif ty tai O va oO. Gia str a, tacé: AABC = AA'B'C’ (c.g.c) = t6n tai mot phép doi hinh f dé AA'BC = f(AABC) = A'0'= f(AO) = D' = f(D). Vay, ta duge: A'BCD' = f(ABCD) => hai hinh tt gidc 16i ABCD va A'BCD bang nhau. cc. Khong thé, bai hinh thoi ABCD canh a khong thé bang hinh vuong A'B'C’D' canh a, Goi O, O' theo thf ty 1a tam cua hai n-gide déu (H), (H'), va A, A’ 1a hai dinh cita hai n-gide déu d6, Nhan xét ring: QW) TCH) = (H') = (H) = (HI). 007 Dap s6 trdc nghiém A. di ty Ingn: Xét hai tam gidc OO. 0,0, 51, +1 =1h. 0,0, Suy ra: AO,0,0. => t6n tai mot phép doi hinh f dé AO,O,0, = f(AT,L1,) => (H,) = fH) Vay, hai hinh H, va H, bang nhau. Dap sé trée nghiém A. Dap sé trée nghiém A. tic ludn: Viet tai phuong tinh parabol (P’) didi dang: -( 2)- 1,1,,0,0, = 6 +1 = Ish, Loi 35 A By eytoe —|. wt aa (+3) a )dé(P) = TP). Nhu vay, t6n tai phép tinh tién T theo vecto v(— x io a da Vay, hai parabol (P) va (P’) bang nhau Bai 58: Goi M [a tung diém ciia doan BC va G 1a trong tam AABC. Gia st: F(M) = M' va F(G)=G’. “Tittinh chat cia phép di inh. ta suy ra: M ‘la trung diém ciia BC = A'M 1a trung tuyén, (1) AiG a2 ASS 2 AM 3 Cy Tir (1) va (2) suy ra G' 1a trong tam AA'B'C’ Bai 59: Dé chimg t6 AEJK = FOIC ta can di chtmg minh tén tai mot phép doi hinh GF (la hgp cia nhimg phép doi hinh da biét) sao cho GAAEJK) = FOIC. Goi L 1a trung diém OF, ta nhan xét ring: Dy(AEJK) = BELF, D Tz; (BELF) = FOIC. el. Tird6 suy ra: s Te; (Dyg(AEJK)) = FOIC => AEIK = FOIC Bai 60: Dap s6 trdc nghiém a). B:b). B. OM = OM’ a. Tirdinh nghia: Q's (M) = M eK (OM,OM') = & Ta Jan Iuot nhan thay: + ViAe PUD va A’ © PU) nén géc quay a <0. Matkhéc: OA (-3:2) = OA= y(-3 +2? = Vi3, OA’ (2: 3) = OA' = V2? 43? = VIB, (ASA. * ViBe Pa va B' € PCI) nén géc quay eet Vay, ta c6 két Luan Q,.” OB.OB' = (-4)5 +54 =0 = (OB,OB) = 90" Vay, ta cd két luan Q5”" (B) = B 36 + ViCe Pd) va e PU) nen géc quay a <0. Mat khéc: OC (-1: 3) > OC= (-p? +¥# = VIO, vi0. 06 3:1) 0c = V8 +P fu dé, euy ra; OC = OC 0C.0@ =-1).3431=0=(0C Vay, ta c6 két luan Q5”" (C)=C. b. Ta lin lugtc6: - Q5”" (AABC) = AA'BC. - AA,B,C, = Do,(SA'B'C) nén: A, = Do(A) => A\(2; -3), B, = Do,(B) > B,(5; -4), C, = Dy(C) => C3; -1). ic nghiém a). A:b). B; c). A; d). B. ic nghiém a). A;b). By c). A. Loi gidi ue ludn: Nhan xét rang, + Phép chiéu vudng géc lén mot dudng thang khong bio toan khoang cach gitia hai diém nén no khong phai 1a phép doi hinh. + Phép déng nhat va’ phép doi xtmg truc 1 phép di hinh. _* Phép vi ty ti sé ~1 thuc chat Ja phép doi xtmg tam nén né 1a phép doi hinh. Dap s6 trac nghiém B. Loi gidi ng ludn: Phép d6i xting truc bién dung thing thanh dudng thing song song hodc tring v6i né chi trong trung hop truc doi ximg song song hoac tring Khi d6, tam vj ty O thoa man OP =k OF = k chi c6 thé bing -1 = O (tam vi ty trong) [a trung diém ciia IT (chinh 1a tiép diém cua hai dudng tron). Tring hop 2: Néu R # R' thi ta c6 thé xde dinh cde phép vi ty sau: 37 = Lay A'B' 1a mot dutng kinh cla duéng tron (I';R’) va JA 1 mot ban kinh ctia (1; R) sao cho hai vecto IA va I'A' cing hudng. 0. © Duong thing II’ cat AA’ va AB’ lan luot tai O, t O, (tam vi tu ngoai) va O, (fam vi ty trong 4 va O, tring véi tiép diém). B b. Hai dudng tron (i; R) va GB) ep xtic trong véi nhau (R # R’), tac thé xac dinh cdc phép vi ty sau: : = Lay A’B' la mot dudng kinh cia dudng tron (I R’) va IA 1a mot ban kinh ca (1; R) sao cho hai vecto IA va I'A’ cing hung. 0} "Duong thang IT’ cit AA’ va AB’ Lin lugt tai O, (tam vi tu ngoai) va O, (tam vi tw trong). B c. Dudng tron ) nam trong dudng tron (I'; R’), ta xét: Trung hop 1: Néu 11 thi khi d6 tam vi tu O tring véi diém L Vay, ta c6 hai phép vi tu: = Phép vi ty V,(I; k,) vdi k, = = (bién diém M thanh diém M'). © Phép vj ty V,(I; ky) voi ky = -< (bién diém M thanh diém M’). Truémg hop 2: Néu I khong tring véi I’ thi ta 06 thé xéc dinh céc phép vi ty sau: = Lay A’B' la mot dudng kinh cita duéng tron (I'; R') va TA 1a mot ban kinh cita (1; R) sao cho hai vecto IA va T'A’ cing huéng. = Dudng thang II’ cat AA’ va AB' lin luot tai O, (tam vi ty ngoai) va O, (tam vi tu trong). Bai 72: Xét hai tru’ng hop: Truong hop 1: Néu k = 1 thi: M'N' = MN = MNN'M [a hinh binh hanh => MM’ = NN’ ‘Suy ra M,N’ theo thirty 4 anh cua M, N trong phép tinh tign theo vecto. v = MM’. Vay, trong truémg hop nay F 1a mét phép tinh, ‘Truéng hop 2: Néu k # | thi tir ding thc M'N' =k MN suy ra: MN // M'N' va MN # M'N' nén MM’ NN’ = {O}. Tu dé, ta nhan duge: OM'_ ON’ OM ON N => OM' =kOM va ON Suy ra M’, N' theo ther ty 1a anh cla M, N trong phép vi ty tam O ti sk. 38 tick: Gia sit da dumg duge duong thing d cat (0) & M va cat (0') 6N sao cho M 1A trung diém cua AN, goi k = S = 2. Thuc hign phép vi ty V tam A.tis6 k =2 thi N= V(M). b. Cach dung: Ta lan luot thuc hién: Dung (O") = V(O), khi dé (O") > (O*) = {N}. - _N6i AN cat (O) tai M. AN la dudng thang d phai dung c. Ching minh: Ta cé ngay M, N theo thit tu thudc dudng tron (O) va (O'), ngoai raz N=V(M) => AN =2AM => M latrung diém clia AN. d. Bién ludn: Vi (O") cat (O’) tai duy nhat mot diém N nén bai todn chi cé mot nghiém hinh, Bai 74: Huéng dan: Ding phép vi ty tam I bién M thanh N. Bai 75: Gia sit dudmg thing BC cat (O), (O') va OO’ theo thif tu tai A, A’ va 1. Vi C tam ti cy trong ciia (O') va (O") nén: Ge 112 oe Gas BS "SOA 80; cB RY co OA soa'/ops B- OB LR lw OAR => 11a tam ti cu ngoai cua hai dudng tron (O; R) va (O"; R’). Vay, dudng thang BC luén di qua mot diém co dinh 1a tam ti cy ngoai cla hai dudng tron (O; R) va (O'; R’). a. V6i phép vi tu V, ta cé: AB AB AB AB VAM (M) =B: VSM (N)=C; VAM (P)=D; VSM (Q)=E Vi MNPQ 1a hinh vudng nén BCDE ciing la hinh vudng. b. Ta thyc hign theo cdc bude sau: Phan tich: Gia sit da dung duoc hinh vung al ndi tiép trong tam gidc ABC cho trudc. Goi k = fy Thue hién phép vi ty tam A, ti sO k thi hinh on Nl MNPQ sé bien thanh hinh vung BCDE. Suy ra A, P, AI D thang hang va A, Q, E thang hang. Cach dung: Ta lan |uot thuc hién: - Dumg hinh vuong BCDE sao cho BCDE va ABC nam 6 hai bén cla dudng thing BC. - AD cat canh BC tai P; AE cat canh BC tai Q 39 - Dung duéng thang qua P, song song véi BC cat canh AC tai N - Dung dudng thang qua N, song song vdi BC cat canh AB tai M Thi MNPQ [a hinh vudng phai dung Ching minh: Theo cach dumg ta c6 tit gic MNPQ néi tip trong tam gide ABC. Ap dung dinh ly Talet, ta co: 2 AB Do dé: VS: MNPQH BCDE ma BCDE 1A hinh vuong nén MNPQ a hinh vung Bién Iudn: Ta udn chi c6 mot hinh vuong BCDE 6 khac phia cla tam gide ABC d6i v6i dug thang BC nén Iu6n cé chi mot diém P va chi mot diém Q tren canh BC. Vay bai toan cé mot va chi mot nghiém hinh. Bai 77: at, .g, [PBL QB cae a ; a. Tacé: {rn io QB// AP = QB la dudng trung binh cla AACM => Qld tung diém cia CM Taco: (PB+QB _ Bn y AQ AQ 1 QB => BN [a duong trung binh cla AACQ. M => N 1 tung diém cia CQ. b. Ta lan Juot thay: + Voi diém Mthi: CM =2CQ > M= Vé(Q) Va viQ € (O) nénM € (O,) = Ve((O)). 1 CQ N= VEQ + Voi diém N thi: CN = 7 A 1 va vi Q € (0) nen N € (O,) = Vé ((O)). Bai 78: Tir diéu kien: GA +GB+GC = 0 Suy ra G Ja trong tam AABC. B c Goi M 18 tung diém canh BC thi OM 1 BC. ‘Trong AOMC ta c6: OM? = OC? = MC? = R?- d? => OM = VR? = d? Vay tap hop diém M I dudng tron (7) tam O bin kinh VR? Vi AG 2 nen Vil (M)=G 5 : ‘AM Suy ra tap hop cdc diém G 1a dung tron (C) vai (C)= VA"). Vi C tam ti cu trong cua (0') va (O" cA RL © Lowy Bo" CBR" CO" On /oR= 1B. OB LR A ON R => LA tam ti cur ngoai cua hai dung tron (Oz R) va (0%: RB’). Vay, dudng thing BC lu6n di qua mot diém 6 dinh 1a m ti cu ngoai cua hai duOng tron (O; R) va (0%; R’). - 1a tam vj ty duong cua hai duéng tron (O) va (0! + De(O), 1e(O) va ba diém C. Do dé thuc | thang hang. ién phép vi tir tam C, ti 86 = (vi R' 1a ban kinh ciia (0), ta f cé: HE :O60716D>01//0'D=> Ol 1 AB => 11 trung diém ciia cung AB => AI = BI AB _ 2R 5 Khidé: Al=Bl= 22 ==" =Ry2. Wy a 2 1 Bai81: Goi AA'BC = Vj (ABC) khi dé: L = VR (A) => HA = 5 HA do dé A‘ chink la trung diém clia HA. 1 = Vj (B) => HB = HB do d6 B' chinh [A trung diém ctia HB. Nie \ C= Vj(C)= HC = . 4 HE do.d6 C chinh 1a trung diém cita HC. Bai 82: Dap sd inde nghigm C. Lai gidt te ludn: Gia sit hai phép vi ty lan het 6 Mthi: 6 k, Va ky, Khi dé voi mdi diém @ (2) vi (M) =_M, sao cho OM, =k,OM. vie (M,) = My sao cho OM, =k,OM,. Thay (1) vao (2), tadyge: OM k,-k, OM <> M, = Vé*!(M). Vay. khi thuc hién lién tiép hai phép vi tu tam O (lan lugt c6 ti sé k, va k,) sé duoc mot phép vi tu tam O ti sé ky Bai 83: Dap sd tric nghiém D. 4) Bai 84: a. Goi M Ja irung diém cua doan BC va G la trong tam AABC. Gia sit: F(M) = M' va F(G) Tir hé qua, ta suy - _ 1a trung diém cla BIC => A’M' Ia trung tuyén. (1) SS, (2) Tir (1) va (2) suy ra G' 1a trong tam AA'BIC. b. Goi AA,, BB, 18 hai dung cao cia ABC va H la tryc tam AABC. Gia sit: F(B,) = B,', F(A,) = Ay’ va F(H) =H’. = Tirtinh chat cita phép déng dang, ta suy ra: H'= A,A,'OB,B,. (3) = Tirtinh chat bio to’n do Ién géc cua phép déng dang, ta suy ra: A,A\', B,By' [a cc dutng cao cia AA'BC. (4) Tir (3) va (4) suy ra H’ la truc tam AA'BC. c. Ta lan lugt xét: * Goi O 1a tam duéng trdn ngoai tiép AABC. Gia sit: F(O) = Tir tinh chat ti 1é vé do dai gitfa hai doan thang, ta suy ra: O'A'=O0' OC béi OA = OB= OC. Vay, diém O 1a tam dutmg tron ngoai tiép AA'BIC. 85: Dap sé trdc nghiém A. gidi tie ludn: Goi P, va P, [a hai da gidc déu c6 cling s6 canh, ta thuc hién: = Phép quay bién P, thanh P,' cé canh song song véi canh cia P,. * Khi dé, sé t6n tai mot phép vi tu bién P,' thanh P,. ‘Tir d6, theo dinh Ii trén thi tén tai mot phép déng dang bién P, thanh P, nen Jéng dang v6i P,. a Dap sé trac nghiém a). A:b). B. han xét rang: = V6i phép doi xing tam I, ta c6: D\UHDC) = IKBA. D, H \ Vé (IKBA) = JLKL al * Véi phép vi ty tam Cti so > tacé: w ATO \ \ Tir dé, suy ra: JLKL= V2 (IKBA) = V@ (B\THDC)) Do d6, hai hinh thang JLKI va IHDC déng dang vi nhau theo ti s6 i. Bai 87: Dap soirée nghiém A. Léi gidi n¢ ludn: Ta lan luot thuc hién: a. Vi phép quay tam O géc 45" thi: Qf" () = 1,0; V2) > O8" (2) =). 2). 42 b. V6i phép vi tu tam Oi s6 V2 thi: Vo? (A, 2))= (1, 22), Trong d6: OF; = ¥201, = V2 0; V2 )) > 1,0; 2). tam 1,(0;2) Bai 88: Dap so trdc nghiém A. Lot gidi ng ludn: Nhan xét rang: + Voi phép vi tu tam O ti s63, ta duge: V9 (I: 2)) = (1); Ry) trong d6: Ol, = 301 = 1,(3; -9);R, =3.2=6. * V6i phép déi xtmg qua truc Ox ta duoc: DOx((L,; R,)) = Uh; Ry) voi (3; 9) Tirdo suy ra: (1,, 6): (x - 32 +(y-9P =3 6. Tir dé, ta duoc: (1, 2V2 ): 5 on 2V2): x2 + (y - 2% =8. R=22 43 CHUONG II. DUGNG THANG VA MAT PHANG TRONG KHONG GIAN QUAN HE SONG SONG § 1. DAICUONG VE DUONG THANG VA MAT PHANG 1 KIEN THUC CAN NHO 1. MG BAU VE HINH HOC KHONG GIAN Mon hinh hoc khong gian la mon hoc nghién cttu cdc tinh chat cla cdc hinh nam trong khong gian. Hinh hoc khong gian c6 cdc déi trong co bin Ia diém, dudng thang va mat hing. Quan hé thuéc: Trong khong giane a. Voi mot diém A va mot dudng thang d c6 thé xdy ca hai rudng hop: © Diém A shude dudmg thing d, ki higu A € d. = Diém A khong thude dung thang d, ki higu A ¢ d. b. V6i mot diém A va mot mat phang (P) c6 thé xay ra hai trudng hop: = Diém A thuge mat phang (P), ki higu A € (P). © Diém A khéng thuée mat phing c, ki hiéu A ¢ (P). 2. CAC TINH CHAT THUA NHAN CUA HiNH HOC KHONG GIAN- Tinh chat thira nhan 1: C6 mot va chi mot dudng thang di qua hai diém phan biét cho truée. Tinh chat thita nhén 2: C6 mot va chi mot mat phdng di qua ba diém khong thang hang cho truéc Tinh chat thita nhén 3: Tén tai bon diém khong cling ndm trén mét mat phing Tinh chat thita nhén 4: Néu hai mat phing phan bigt c& mot diém chung thi ching c6 mot duimg thang chung duy nhat chita tat ca cae diém chung tia hai + mat phdng do. Tinh chat thita nhén 8: Trong moi mat phing, cde két qua da biét cia hinh hoc phdng déu dig Dinh li: Néu mot duimg thang di qua hai diény phan biét cla mot mat phing thi moi diém cia dutng thang déu thudc mat phang d6. 3. pIEU KIEN XAC DINH MAT PHANG C6 bon edch xac dinh mot mat phing: Cach 1: Mot mat phiing duge xidc dinh néu biét né di qua ba diém A, E, C khong thang hang cla mat phiing. ki hiéu (ABC). Cdch 2: Mot mat phang duge xd¢ dinh néu biét né di qua mot duing thang d va mot diém A khong thudc d.ki higu (A, d). 44 Cach 3: Mot mat phang duge xac dinh néu biét n6 di qua hai dudng thang cat nhau a, b, Ki higu (a, b), Cach 4: Mot mat phang duoc xc dinh néu biét n6 di qua hai dudng thang song song a. b, ki higu (a, b). 4. HINH CHOP VA HiNH TU DIEN Dinh nghia: Cho da gidc A,Ay...A, va cho diém S nim ngoai mat phing chita da gidc d6. N6i S véi cdc dinh A,, Ay, «.. A, ta duoc n mién tam gide SA,A), SAgAg on SAy- Age Hinh gém n tam gide dé va da gidic A\Ay.\, duce goi la hinh chép S.A,AsuAy Trong dé: Ss = Diém S goi la dinh cua hinh chop. * Da gidc A,A,...A, gi Li mat ddy cia hinh chop. jf * Céc doan thing A\Ay Ayé with, Ag i 1a edie canh day cia hinh chép. ‘ * Cae doan thing SA,. SAs, ... SA, gi LA AN : : eae ‘As cdc canh bén cia hinh chép. ay * Cac mién tam gic SA,A, é : Ay SA, iA, 201 la ede mat bén ciia hinh chop. Néu day cia hinh chop 1a mot mién tam giste, tu gide, nd gide, . thi hinh chép tuong tmg goi Ii hinh chép tam gidc, hinh chép tr gidc, hinh chép ngi pide, . Chi ¥: 1. Hinh chp tam gic con goi la hin nidign 2. Hinh titdién co bon mat ki nhimg tam gide déu duoc goi lA hinh nidién déu. 1. BAI TAP TRAC NGHIEM VA TULUAN Bail: Xac dinh tinh ding, sai cua ede menh dé sau: . C6 duy nhat mot mat phing di qua ba diém cho tude. A. Ding B. Sai b. C6 duy nhat mot mat phiing di qua ba diém khong thing hang cho tude A. Ding B. Sai ’ c. Ba diém khong thang hang cing thudc mot mat phang duy nhat. A. Ding B. Sai Cho hai mat phing (P) va (Q) cat nhau theo giao tuyén A. Trén (P) cho dudng thing a va trén (Q) cho dudng thang b. Khang dink "Néu a va b cat nhau thi hai diém phai nim trén A" la ding hay sai ? A. Ding. B. Sai , Bai 3: Cho mat phing (P) vi ba diém khéng thing hang A, B, C cing nam ngoai (P). Khang dinh "Néu ba dudng thing AB, BC, CA déu cat (P) thi cdc giao diém do thing hang" la ding hay sai? A. Ding. B. Sai. 45 XAc dinh tinh dting, sai cua cdc ménh dé sau: a. C6 duy nhat mot mat phang di qua mot diém va mot dudng thang clho trude, A. Ding B. Sai. b. C6 duy nhat mot mat phiing di qua mot diém va mot dudng thang chita diém dé. A. Ding. B. Sai. c. C6 duy nhat mot mat phang di qua mot diém va mot dudng thang khong chtta diém do. A. Ding B. Sai. Bai $: Xéc dinh tinh diing, sai cba céc ménh dé sau: a. C6 mot mat phang duy nhat di qua hai dudng thang cho truéc. A. Ding. B. Sai b. C6 mot mat phang duy nhat di qua hai dudng thang cat nhau cho truéc. A. Ding. B. Sai c. C6 duy nhat mét mat phang di qua hai duémg thang ma hai dudng thang 46 lén Iuot nam trén hai mat phang cat nhau. A. Ding. B. Sai Bai 6: Cho hai dutng thang a va b cat nhau. Mot dudng thang c cat ca a va b. C6 thé két Juan rang ba dung thing a, b, ¢ cling nam trong mét mat phang hay Khong ? A. C6. B. Khong. Bai 7: Cho ba dutmg thing a, b, c khéng cing nam trong mot mat phding sao cho chting déi mot cat nhau. Co thé két ludn rang chting déng quy hay khong ? A. Co. B. Khong. Bai 8: Cho hai dung thing a va b cat nhau tai diém © va dudng thing c cét mp(a, b) & diém I khéc O. Goi M 1a diém di dong tren c khdc I, Khang dinh "Giao tuyén cla cdc mat phing (M, a), (M, b) nam trén mot mat phang c6 dinh" la diing hay sai? A. Dung. B. Sai. Cho hinh binh hanh ABCD nam trong mat phdag (P) va mot diém S nam ngoai mat phang (P). Goi M li diém nam giita S va A: N [a diém nam gitta S va B; giao diém cua hai dudng thang AC va BD 1a O; giao diém cia hai dudng thing CM va SO la I; giao diém ctia hai dug thang NI va SD la J a, Tim giao diém cia mat phing (CMN) véi dudng thing SO. AE Bae Cc. A D. B. b. Xéc dinh giao tuyén cia hai mat phing (SAD) va (CMN). A. ML B. MJ. C. NE D. NI. 46 Thiét dién cua mot hinh uf dién co the 1a: a. Tam gidc hay khong ? AL C6, B, Khong. b. Tit gide hay khong ? A. C6 B, Khong c. Ngii gic hay khong ? AL C6 B. Khong. Baill: Cho diém A khong nam wén mat phang (a) chia ABCD. Lay BE, F la cdc digm kin hot nam trén cae canh AB. AC. a. Khang dinh “Duéng thang EF nam trong mat phaing (ABC)"Ia ding hay sai? A. Bung B. Sai b. Khi EF va BC cat nhau tai 1. Khang dinh “Ila diém chung cua hai mat phing (BCD) va (DEF)" [a dting hay sai? A. Dung. B Sai Bai 12: Cho bon diém A, B.C va D khong dong phang. Goi Gy. G,. Ge. Gy, kin luot A trong tam cua cée tam gide BCD. CDA. ABD. ABC. Khang dinh “AG. BGy, CGc. DG, déng quy” I duing hay sai? A. Ping B. Sai § 2. HAI DUONG THANG SONG SONG 1, KIEN THUG CAN NHO 1. VE FRITUONG BOI CUA HAL DUONG THANG PHAN BIET. Cho 2 dung thang a va b, Can ett vito su déng phang va sé digm chung cua 2 duis au cS bon truding hop saur a thang sony song: cling nim trong MOL mat phang va khong o6 diém chung. tte fir , a (Pivabe (P) alibes b. Hai diténg thing cat nha: chi c6 mot diém chung acitbeea b= {I}. c. Hai duéng thang ting nhat: 66 hai digm chung phan biet amb=([A.B}] eaeb. &. Hai ding théng chéo nhaw: Khong ct 2 thuge mot mat phang. a chéo b (PYHKQ) 2. pItU KIEN Df HAIMAT PHANG SONG SONG Dinh Ii 1: Néu mat phang (P) chifa hai dung thing a, b dt nhau va cing song song véi mat phang (Q) thi (P) song song (Q). adel) Ae Tic la: Ja cit b => (P)//(Q). aMQ) va bIQ a / 3. TINH CHAT Tinh chat I: Qua mot diém nam ngoai mot mat phang, cé mot va chi mot mat phang song song vdi mat phang do. O€(Q) (P)MQ) Céch dung: - Trong (P) dung a, b cat nhau. - Qua O dung a, // a, b// b. - Mat phang (a,, b,) a mat phang qua O va song song vi (P). Hé qud I: Néu dutng thing a song song voi mat phang (Q) thi qua a cé mot va chi m6t mat phang (P) song song vdi (Q). Tite la: O ¢ (P) => 3" Q): { Hé qua 2: Hai mat phing phan biét ciing song song vdi mat phang thi ba thi song song v6i nhau. Tinh chat 2: Néu hai mat phang (P) va (Q) song song thi moi mat phang (R) da cat(P) thi phai cat (Q) va cdc giao tuyén cla ching song song. 52 (P)MQ)d Tuc la: ja=(P)A(R) = a/b. b=(QYA(R) -Dinh li Ta — lét trong khong gian: Ba mat phing doi mot song song chin trén hai cat tuyén bat ki cic doan thang tuong ting ty Ié. a, b {(P) MQTIKR) Tic las Ja 0(P)= A, va a(Q)=B, va aA(R)=C, [bO(P)= A, WW bA(Q)=B, va DA(R)=C, nal 1B A,B Aas 4, HINH LANG TRU VA HINH HOP. ry Dinh nghia hinh lang tru: Hinh lang wu ki mor hinh da din c6 hai mat nam trong hai mat pling song song goi la hai day va tat cei cde canh khong thuoe hai day déu song song voi nhau. Trong d6: Cac mit khdc véi hai day goi 1a céic mat ben cia binh lang tr. = hai mat bén goi lA canh cua hinh lang tru, = Tuy theo da gidte di tam gisic, lang tru Uf giac,... Tir dinh nghia cia hinh lang try, ta kin lugt suy ra cic tinh chi a. Cac canh ben song song va bing nhau b. Cac mit bén va cd chéo la nig hinh binh hanh. cc. Hai day la hai da gid ic canh tuong (mg song song va . Dinh nghia hinh hép: Hinh lang tru c6 day la hink binh kant goi la hinh hép. a. Hinh hop c6 tat ca cdic mat bén va cdc mat day déu 1a hinh chit nhat goi Ja hinh hop chét nhat. b. Hinh hop co tat c hinh lap phuong. a ¢6 hinh King tru es eee) CZ BA B, 7 Al A B Chit ¥: Cac duing chéo cia hinh hop ca Al t nhau tai trung diém mdi dung. 33 §. HINH CHOP CUT Dinh nghia: Cho hinh chép SA,A)...A,. Mot mat phang (P) song song véi mat phing chita da gidc day cat cdc canh SA,, SA;, .... SA, theo thif tu tai A’, A’ ws. A’, Hinh tao béi thiét dién A’,A’, wA’, va day A,A>...A, ciia hinh chép cing véi cc mat ben A,A,A%A’), AAA A, os A,A,A’,A’, goi la mot hinh chép cut. Trong dé: AL A = Day ciia hinh chép goi Ia déy /én cita hinh chép cut, con thiét dien goi Ia day nho cia hinh chép cut. = Cc mat con lai goi 1a cde mat bén cla hinh chdp cut. * Canh chung ciia hai mat bén ké nhau nh A,A’), A,A’>, ..A,A’, goi la canh bén cua hinh chép cut. Tuy theo day 1a tam gidc, tt gidc, ng gidc, .. ta cé hinh chép cut tam gidc, hinh chép cut tt gidc, hinh chép cut ngii gidc Tinh chat: Vi hinh chép cut, ta c6 cdc tinh chat sau: 1. Hai day cia hinh chép cut 1a hai da gidc déng dang. 2. Cac mat bén cua hinh chép cut la cdc hinh thang. 3. Céch canh ben ciia hinh chép cut déng quy tai mot di’m. I. BAI TAP TRAC NGHIEM VA TULUAN Bai 26: Xéc dinh tinh diing, sai cla cdc ménh dé sau: Hai mat phang phan biét ciing song song véi mot dung thang thi song song v6i nhau. A. Ding. B. Sai. b. Hai mat phang phan biét cing song song vdi mot mat phang thi song song véi nhau. A. Ding. B. Sai. c. Néu hai mat phing song song thi moi dudng thang nam trén mot mat phang déu song song voi mat phing con lai. A. Ding. B. Sai. d. Néu hai mat phing song song thi méi dudng thing nam trén mat phing nay déu song song véi moi duéng thang nam trén mat phang kia. A. Ding. B. Sai. e. Néu hai mat phing phan biet Jan luot di qua hai dudng thing song song thi song song véi nhau. A. Ding. B. Sai. f. Néu mot dudng thang cat mot trong hai mat phang song song thi cat mat phang con lai. A. Ding. B. Sai. 54 fc dinh tinh ting, sai cla cic ménh dé sau: a. Hinh hp 1a mot hinh lang tru. A. Diing. B. Sai. b. Hinh lang tru c6 tat ca cdc canh song song. A. Ding. B. Sai. c. Hinh lang tru cé tat ca cdc mat ben bing nhau A. Ding. B. Sai d. Hinh lang tru c6 tat c& céc mat bén fa hinh binh hanh. A. Diing. B. Sai. e. Hinh hép c6 cdc mat doi dign bing nhau. A. Diing. B. Sai, Tim ménh dé SAI trong cc ménh dé sau day: A. Néu hai mat phing c6 mot diém chung thi chting con c6 v6 sé diém chung khac nifa. B. Néu hai mat phang phan biét cuing song song véi mot mat phang tht ba thi chting song song véi nhau. C. Néu hai duémg thang phan biét cling song song véi mot mat phang thi song song v6i nhau. D. Néu mot dudng thang cat mot trong hai mat phang song song vdi nhau thi sé cat mat phang con lai. Bai 29: Tim ménh dé diing trong cdc ménh dé sau : A. Néu hai mat phang (a) va (B) song song véi nhau thi moi dudng thang nam trong (a) déu song song véi (8). B. Néu hai mat phing («) va (B) song song véi nhau thi moi duéng thing nim trong («) déu song song vdi moi dutmg thang nam wong (B). C. Néu hai dung thang song song vdi nhau Jan luot nim trong hai mat phang phan biét (a) va (B) thi (a) va (B) song song véi nhau. D. Qua mot diém nam ngoai mat phang cho trudc ta vé dugc mét va chi mot dudng thang song song véi mat phang cho trudc dé. Cho hai dudng thang chéo nhau a va b lan lugt nam trén hai mat phang song song (P) va (Q). Héi néu diém M khong nam trén (P) va khong nam tren (Q) thi c6 bao nhiéu duéng thang di qua M cat ca a va b. Aol. B. 2. Cc. 4. D. V6 sé. Bai 31: Trong mat phang (P) cho hinh binh hanh ABCD. Qua A, B, C, D lan luot vé bon dutng thing a, b,c, d doi mot song song véi nhau va khong nim trén (P). Mot mat phang ct a, b, c, d lan luot tai 4 diém A’, BY, C’, D'. Tir gidc A’BCD 1a hinh gi? 55 A. Hinh thang. C. Hinh chit nhat. B. Hinh binh hanh D. Hinh vuong Bai 32: Cho hinh binh hanh ABCD. Goi Bx, Cy, Dz lin Iugt 1 cde dung thang song song véi nhau di qua B,C, D va nam vé mot phia cita mat phang (ABCD), déng thoi khong nam trong mat phing (ABCD). Mot mat phang di qua A va cat Bx, Cy, Dz lan luot tai BY, C’ . D’ véi BB’ = 2, DD" = 4. Khi dé CC” bang: A}. B. 4. Cc. 5. D. 6. Cho hinh ling try tam gide ABC.A’B'C’. Goi I. J Lan lugt 18 trong tam ic tam gide ABC va A‘B'C’ (hinh tg 79). Thiet dign tao boi mat phang cua (AU) véi hinh kang tru da cho la: A. Tam gide can. C. ffinh,thang. B, Tam gidc vuong. D. Hinh binh han. Cho hinh binh hanh ABCD. Qua A, B.C, D kin lugt vé bon nia duémg, ig Ax. By, Cz, Dt 6 cing phia doi v6i mat phang (ABCD). song song voi nhau va khong nam trong mat phang (ABCD). Mot mat phing (B) Kin luot cat Ax, By, Cz ya Dial ABC va DY ‘ (mg minh (Ax, By) // (Cz. DD. CO BD, J = A'C’ 9 B'D’. Chimg minh U // AA’. c. Cho AA’ = x, BB’ = y. CC’ = z. Hay tinh DD". Aoxty+z Beoxty-z Gox-ytz Dex-y-z Cho tit dien ABCD. Goi M fi trung dig ciia AB. Hoi mat phang (P) qua M, song song véi ca AD va BC 6 di qua trung digm N ciia CD khong ? A. C6. B. Khong. Cho tit dign déu’ SABC canh bang a. Goi I fa trung diém AB, M fa mot diém di dong trén doan Al. Qua M vé mat phang (c) song song voi (SIC). a. Thiét dién tao boi (a) va te dién SABC.Ia: . A. Tam gidc can tai M. C. Hinh binh han. B. Tam gisic déu D. Hinh thoi b. Chu vi cua thidét dién theo AM = x [a A. x + V3). ©. 3x1 + V3. B. 2x1 + V3) D. Khong tinh duge. ' Bai 37: Cho hinh vuong ABCD va tam gidc déu SAB nain trong hai mat phang khac nhau. Goi M Ih diém di dong wén doan AB. Qua M vé mat phiing (a) sorg song véi (SBC). Thiét diéri tao boi (a) va hinh chép S.ABCD a hinh gi? A. Tam gide. —B. Hinh binh hanh, C. Hinh thang. D. Hinh vudng. Goi N. P. Q kin luot li giao cla mat phang (a) v6i céc dudmg thang CD, S, SA. Tap hop cdc giao diém I cua hai duéng thang MQ va NP 1 : A. Duong thing. C. Doan thing song song véi AB. B. Nita dudng thing. D. Tap hop rong. 56 Bai 2 Cho hinh lang tru tam gic ABC.A'BC. Goi H 1a trung diém cla canh AB a. Chimg minh rang dutmg thang B© song song voi mat phiing (AHC). b. Tim giao tuyén d ctia hai mat phing (ABC) va (A’BC). Chimg minh rang d song song véi mat phang (BB'C’C). c. Thiét dién cua hinh lang tru ABC.A'BC khi cat boi mat phang (H, d) 1a hinh gi? \. Tam gidc. — B. Hinh binhhanh. C. Hinh thang. —_D. Hinh vuong. Bai 40: Chimg minh rang t6ng binh phuong tat ca céc dung chéo ctia mot hinh hop bang téng binh phuong tat ca céic canh ca hinh hop d6. §5. PHEP CHIEU SONG SONG I. KIEN THUC CAN NHO 1, PHEP CHIEU SONG SONG Cho mat phang & va mot dudng thing / khong song song véi &. Véi mdi diém M trong khong gian, dudng thing qua ! M song song véi / sé cit a tai diém M’. Diém M* duoc goi li hink chien song song cua diém M tren mat phang a theo phuong /. Mat phang « goi la mat phaing chien Phép dat tuong img méi diém M trong khong gian voi hinh chiéu M’ cta n6 trén « duge gi kat phép chi¢u song song lén mat phdng actheo phucng I. Chi y: Néu a // thi binh chiéu ciia a én & la mot diém tren a (chinh fa giao diém cia a v6i a), do vay cdc tinh chat trong phin sau chi xét nhimg doan thing hoc dung thing khong song song v6i /. 2. CAC TINH CHAT CUA PHEP CHIEU SONG SONG Dinh li 1: Phep chiéu song song bién ba diém thang hang thanh ba diém thing hang va khong lam thay doi Mur ugctia ba diém do. Hé qud: Hinh chiéiu song song ctia duéng thang la dung thang, ctia tia la tia, eta doan thang la doan thang. Dinh li 2: Hinh chiéu song song ctia hai dudng thdng song song la hai dung thang song song hode tring nhau. a M Hé qua: Hinh chiéw song song ciia mot hink binh hanh khong ném trong mat phiing song song voi phucng chié la mot hink’binh hanh 57 Dinh li 3: Phép chiéiu song song khong lam thay doi ti s6' dé dai ctia hai doan thing hode song song hodc cing nam trén mot duéng thang. Tem: AB A'B BC BC 3. HINH BIEU DIEN CUA MOT HINH KHONG GIAN TREN MAT PHANG Ta thutmg vé cdc hinh khong gian nhu hinh chép. hinh lang tru, ... tren bang hay trén-trang gidy, cdc hinh vé d6 goi la hinh biéu dién ctia mot hinh khéng gian trén mat phdng. Dinh nghia: Hinh biéu dién ctia mot hinh H trong khéng gian la hinh chiéu song song ciia H lén mot mat phdng nao dé theo mét phuong chiéu nao d6. Céc yéu cau d6i voi mot hinh biéu dién: 1. Hinh biéu dién phdi ding: Dé vé ding ching ta cén quan tam téi céc yeu 16 duoc bao toan sau: a. Su thang hang va thet tu cia cdc diém trén mot dudng than; b. Sursong song ca cdc dutmg thang, cc tia hotic cdc doan thang c. Ti s6d6 dai cua cdc doan thing cing phuong. Nhu vay, ede tinh chat ciia hinh khong thay déi qua phép chiéu song song déu duce gitt nguyén trén hinh biéu dién 2. Hinh biéu dién phai noi: Gitp ching ta dé tudng tung. Chting ta c6: = Tam gide: Mot AABC cé thé xem 1a hinh biéu dién cia mot tam gidc bat ki (déu, can, vudng). «~ Hinh binh hanh: Mot binh binh hanh ABCD cé thé xem 1a hinh biéu dién cua cdc loai hinh binh hanh nhu hinh vu6ng, hinh chit nhat, hinh thoi va hinh binh hanh bat ki. * Pudng tron: Dé biéu dién duong trdn ching ta sit dung mot hinh Elip. BAI TAP TRAC NGHIEM VA TULUAN 41: X4c dinh tinh diing, sai clita céc ménh dé sau: a. Hinh chiéu song song ciia hai duémg thing chéo nhau cé thé tring nhau. A. Diing. B. Sai. b. Hinh chigu song song ciia hai duéng thing chéo nhau thi cat nhau. A. Diing. B. Sai. Bai 42: Xac dinh tinh ding, sai cua cdc ménh dé sau: a. Hinh chiéu song song cia hai duong thang chéo nhau cé thé song song v6i nhau. A. Ding. B.S b. Hinh chiéu song song cia hai dudng t tring nhau, song song véi nhau. A. Diing. B, Sai. g chéo nhau cé thé cat nhau, 58 Xéc dinh tinh ding, sai ctia cdc ménh dé sau: inh chiéu song song cla hai dudng thang cat nhau cé thé song song véi nhau. A. Ding. B. Sai. b. Hinh chiéu song song ctia hai dudng thang cat nhau thi cat niiau. A. Ding B. Sai. Bai 44: Xéc dinh tinh ding, sai cua cic menh dé sau: a. Hinh chiéu song song ciia hai dudng thang cat nhau c6 thé tring vdi nhau. A. Ding B. Sai. b. Mot dudng thang cé thé song song véi hinh chiéu song song ciia nd. A. Ding. B. Sai. Bai 45: Xdc dinh tinh ding, sai cua céc ménh dé sau: a. Mot dudng thang ludn cat hinh chiéu song song ciia n6. A. Ding. B. Sai. b. Mot dutmg thang cé thé triing véi hinh chiéu song song ciia 16. A. Ding. B. Sai. Bai 46: Cho hinh hop ABCD.A,B,C,D,. Tim diém I trén dutmg chéo B,D va diém J trén dung chéo AC sao cho IJ // BC,. Tinh ti so z \ AL 2. B. I. c. 1 2° 3° BAI TAP LAM THEM . Xdc dinh tinh diing, sai cla céc ménh dé sau: Hai dudng thang chéo nhau thi khong cé diém chung. A. Ding. B, Sai. b. Hai dudng thang khong cé diém chung thi chéo nhau. A. Ding. B. Sai. c. Hai dung thang chéo nhau thi khong cing thudc mot mat pang. A. Ding, B. Sai. d. Hai dudng thang song song thi chéo nhau A. Dung. B. Sai. * Bai 4s: a8: ae dinh tinh ding, sai clia cic ménh dé sau: ~~ a. Hai mat thang phan biét ciing song song véi mot mat phang thif ba thi song song véi nhau. A. Ding. B. Sai. b. Hai mat phang phan biét cling song song v6i mot dudng thang thi song song vGi nhau A. Ding. B. Sai. 59 c. Hai matphing phan biét khong song song thi cat nhau. A. Dung. B. Sai. d. Hai mat phing phan biét cing song song véi mot mat phiing thi ba thi song song véi nhau. A. Ding. B. e. Mot duong thang cat mot trong hal duomg thang song song thi cat dudng thang con lai. A. Ding. B. Sai. f. M6t mat phang cat mot trong hai duéng thang song song thi cat duéng thing con lai. A. Ding. B. g. Mot deme thang cat mot trong hai 1 at phang song song thi cat mat phang con ‘lai. A. Ding. B, Sai. 49: Cho tt dién ABCD. Goi M, N lan lugt la trung diém cua cic canh AD va BC; G IQ trong tam ABCD. Khi d6, giao diém cia duéng thang MG va mp(ABC) A. bDiém C. B. Diém N. C. Giao diém cua dudng thing MG va dudng thing AN. D. Giao diém cia duéng thang MG va dudng thang BC. 50: Cho tr dién ABCD va ba diém E, F, G lin lugt nam trén ba canh AB, BC, CD ma khong triing voi cdc dinh. Thiét di¢n cua hinh th dién ABCD Khi cat boi mp(EFG) la: A. Mot doan thang. C. Mot hinh thang. B. Mot tam gic. Bai 51: Cho ti di¢n ABC! BC, CD ma khong tring vii bai mp(JK) la: A, Mot doan thang. C. Mot hinh thang. B. Mot tam giac. D. Mot ngii gidc. Bai $2: Cho hinh chép S.ABCD. Goi AC 4 CD = J, AD 4 BC = K. Dang thite nao sai trong cac dang thitc sau? A. (SAC) 7(SBD) = SI. C. (SAD) (SBC) = SK. B. (SAB) 4 (SCD) = SJ. D. (SAC) 9 (SAD) = AB. Cho ttt dién ABCD. Goi M. K lin lugt Ia trung diém cua LC va AC,N la diém trén canh BD sao cho BN = 2ND. Goi F 1a giao diém cia AD va mat phing (MNK). Trong cdc ménh dé sau, ménh dé nao dung? A. AF=FD. B, AF=2PD. C. AP=3FD. D. FD=22F. ic dinh. Thiét dign cua hinh tr dign ABCD khi cat 60 Cho hinh chdp S.ABCD. Mot mat phang khong di qua dinh nao cla hinh chép cat céc canh SA, SB, SC < SD lan luct tai A’, B,C, D'. Goi O a giao diém cla AC va BD. Tim ménh dé diing trong cdc ménh dé sau: A. Cac dutng thing A’C, B'D', SO doi mot chéo nhau dc duémg thing A’C, B'D', SO déng phing. C. Cac dung thing A’C: BD’, SO déng quy. D. Hai dudng thing AC’ va BID’ cit mhau con hai dudng thing A'C va SO chéo nhau. 55: Cho tr dign ABCD. Goi G va E lan lot 1a trong tam cba AABD va . Ménh dé nao dudi day ding ? A. Duéng thang GE song song voi duémg thing CD. B. Dutng thing GE cat dudng thang CD. C. Hai dudng thang GE va CD chéo nhau. D. Duong thing GE cat dudng thing AD. Cho ti dign déu-ABCD c6 can bing a. Goi G 1a trong tan. AABC. Cat Bai 56: ur dién boi mat phing (GCD) a dién tich cia thiét dién la: a Bai 57: Cho ia chép S.ABCD cé day 1a hinh binh hanh. Goi I, J lan luot 1a trung diém cia AB va CB. Khi d6, giao tuyén ciia hai mat phing (SAB) va (SCD) la dung thang song song voi: A. Dutng thing AD. C. Dudng thing BI. B. Dutng thang BJ. D. Dudng thang I. 58: Cho hinh chép S.ABCD co diy 1a mot hinh binh-hinh. Goi. A’, B', Cy D’ lan luot 1 trung diém cia cdc canh SA, SB, SC, SD. Tim ménh dé ding trong cdc ménh dé sau: A. A'B'// mp(SAD). C. mp(A'CD’) // mp(ABC). B. A'C // mp(SBD). D.. A'C'// BD. 59: Cho tr dién déu ABCD cé canh bang a, diém M trén canh AB sao cho AM = m (0 < m 0 cho truce). a. Chting minh rang MN song song véi mot mat phang c6 dinh. b. Tim tap hop cdc diém I thudc doan MN sao cho IM = KIN. Bai 66: Cho hai dudng thang chéo nhau a, b. Trén a lay hai diém phan biét A, B; tren b lay hai diém phan biét C, D. a, Ching minh ring AC va BD chéo nhau. b. MIA mot diém tén canh AC, N 1a mot diém trén canh BD. MN c6 thé song song véi AB hoac CD duge khong ? c. Oa diém wen doan MN. Ching minh ring AO cét CN, va BO cét DM. Bai 67: Trong mat phing a, cho tf giéc ABCD, S la mot diém khong thudc a. Goi I, J 1 hai diém cé dinh wén SA va SC véi SI > IA va SJ < JC. Mot mat phing B quay quanh IJ cat SB tai M, SD tai N. a. Chimg minh ring IJ, MN va SO déng quy (véi O'la giao diém cia AC va BD). Suy ra cdch dung diém N khi biét diém M. 62 b. ADcat BC tai E, IN cat MJ tai F. Chg minh S, E, F thang hang. c. IN cat AD tai P, MJ ct BC tai Q. Chtmg minh rang PQ luén di qua mot diém c6 dinh khi di dong. Cho hinh chép SABCD day 1a hinh binh hanh tam O. Goi M,N, P kin lugt [a trung diém SB, SD va OC. a. Tim giao tuy€n cla (MNP) véi (SAC) va tim giao diém cita SA véi (MNP). b. Tim thiét dién ciia hinh chop voi (MNP) c. Tinh ty sé mat phing (MNP) chia céc canh SA, BC va CD. :, Cho hinh chép S.ABCD, day ABCD 1a hinh thang, cdc canh day AD = a, BC = b. Goi I, J lan lugt la trong tam cdc ASAD, ASBC. Tim giao tuyén cila (SAD) véi (SBC). Tim giao tuyén ciia (BCI) vai (SAD). Tim giao tuyén cia (ADJ) véi (SBC) Tim d6 dai doan giao tuyén cua hai mat phang (ADJ) va (BCI) gidi han bdi hai mat phang (SAB) va (SCD) Bai 70: Cho hinh chop S.ABCD, déy ABCD 1a hinh vu6ng canh bang a, mat ben SAB Ia tam gidc déu. Cho SC = SD = aV/3. Goi H, K lan luot 12 tung diém cita SA, SB. M la mot diém trén canh AD. Mat phang (HKM) cat BC tai N. a. Chimg minh HKMN 8 hinh thang can. b. Dat AM = x (0 < x < a), tinh dién tich cla tr giéc HKMN theo a, x. Tinh x dé dién tich nay nhé nhat. c. Tim tap hop giao diém ciia HM va KN; cla HN va KM. Bai 71: Cho chop S.ABCD cé day ABCD 1a hinh binh hanh. Goi M, N lan uot la trung diém cdc canh AB, CD. Goi P a trung diém cila SA. a. Chtmg minh MN song song vdi cdc mat phng (SBC) va (SAD). b. Chiig minh rang SB song song voi (MNP). c. Ching minh rang SC song song voi (MNP). d. Goi G, va G, theo thit ty 1a trong tam AABC va ASBC. Chimg minh G,G, song song véi (SAD). Baj 72: Cho hinh chdp S.ABCD déy ABCD 1a hinh thang cé day lén BC = 2a, AD =a, AB = b. Mat bén SAD IA tam gic déu. a 1a mat phang qua diém M wén canh AB va song song véi SA va BC, a cat CD, SC, SB lan luot tai N, P,Q. a. Chimg minh MNPQ 1a hinh thang can. b. Tinh dién tich thiét dién theo a, b va x = AM, (0 < x MeEA. 0 m 7 = (P)=(Q). 65 Bai 6:, Dap s6 trac nghiém B. Loi gidi tu lun: Khong thé ket luan ring ba duéng thang a, b,c cirg nim trong mot mat phang, béi néu a, b, c cling déng quy tai A thi chting c6 thé khong déng phing (thi dy ba duémg thing AB, AD, AA,, cua hinh lap phuong ABCD.A,B,C,D,) ai 7: Dap s6 trac nghiém A. L6i gidi tg ludn: Vi ba dutng thang phan biét a, b, c. Gia sit: anb={Al,bac=(B}.coa=(C}. Xét hai trudng hgp: Truéng hop 1: Ba diém A, B, C la ba diém phan biét. Do a, b, c phan biét nén A, B, C 1a ba diém khong thang hang. Vay chting xdic dinh mot mat phang (ABC). Ta c6: * Duong thing a cé hai diém A, C thudc (ABC), néna € (ABC). * Tuong tu b € (ABC) vac € (ABC). Vay, ba dung thang a, b,c cling thudc mot mat phang (ABC) — Mau thuan. Truéng hop 2: Hai trong ba diém A, B, C tring nhau, gid sir A = B. Néu A #C thia=c, mau thudn. Do dé, ta phai c6 A= C > A=B=C o> a,b,c déng quy. ‘ay, ba dudng thang a, b, c déng quy. Dap so tréc nghiém A. Loi gidi te ludn: Nhan xét rang: (M, a) 0 (M, b) = MO < (O, C) = cé dinh. Bai9: Dap so trac nghiém a). A; b). B. a. Trong mat phang (SAC), ta co: CM 1 SO = {I} = {1} =(CMN) SO. b. Trong mat phang (SBD), ta c6: NIM SD = (J} = {J} =(CMN) 4 SD => (SAD) 7 (CMN) = MJ Bai 10: Dap sé rac Thiet dién cia mot hinh te din co thé |: vé dudi day): A yy 1 i M A i i aos. of D 0 P a B ‘Thiet dign ciia mot hinh tit dign khong thé 1a ngii gidc boi tit dién chi c6 bn mat. am gidc, tir gide (nh trong hinh 66 Li: Dap so trac nghiém a). Azb). A. A Ta c6 nhan xét: E © ABC (ABC) > E € (ABC), E F e ACC (ABC) = F € (ABC), B D Tir do, suy ra EF c (ABC) b. Ta cé nhan xét: I ¢ EF c (DEF) = I € (DEF), c Ie BCc(BCD) > 1 € (BCD), (x) I Tir do, suy ra 1 € (BCD) > (DEF). A Bai 12: Dép 6 tric nghiom A. Loi gidi te hun: Ta c6 thé trinh bay theo hai céch sau: C4ch J: Goi M 1a trung diém CD, ta c6é nhan xét: B D IG MGs _MGe = 1 6,6, //BA. M MB MA 3 GxG _ GG 646g a1 AG BG) AB 3° Chimg minh tong ty ta cing nhan duoc khang dinh AG, va CG. cing cat nhau tai G. Vay, ba doan AG,. BGy, CG,, DG, déng quy tai G. Céch 2: Goi G 1a trong tam cia te dién ABCD (trung diém doan MN). NOi AG efit BN tai A’ ta cin chimg minh A la trong tam ABCD. Ké NN’ song song véi AA'(N’ € BM), khi d6: Goi G Ia giao digm cla AG, va BGg, ta cé: NN’ [a dudng trung binh cla AABA' N c = NB=N'A\. () : GA‘ li dudng trung binh cua AN'MN = N'A'= MA‘. (2) P M Tir (1) va (2) suy ra BA’ = 2MA‘. C Va vi A’ thudc trung tuyén BM ciia ABCD nén A’ [a trong tam ABCD, tiic 1a A’ = Gy hay néi cach khaic AG, di qua G. Ching minh tuong ty, ta c6 BG,, CGe di qua G. Vay, ba doan AG,, BGy, CGe déng quy tai G. Bai 13: Pép so trdc nghiém a). A;b). B:c). Byd). A a. Ménh dé nay 1a dting, bai néu trai lai gid sit chting c6 diém chung thi suy ra chting déng phang. b. Ménh dé nay Ia chung. c. Ménh dé na d. Ménh dé nay [a ding ~ Ban doc tir gidi thich. i, bGi hai dudng thang song song cing khong co diém 1a sai, boi hai duéng thing cé thé cat nhau. rdc nghiém a). D; b). D. A a. Hai dudng thing MQ, NP chéo nhau, bai néu trai lai tiie 1a: MQ va NP déng phing . = bén diém M,N, P, Q déng phing N => MN va PQ déng phing B => AB va CD déng phang . diéu dé 1 mau thudn. b. Hai dudng thing MP. NQ chéo nhau, boi néu tri lai tic 1a: MP va NQ déng phing => bon diém M,N, P,Q déng phiing C => MN va PQ déng phing => AB va CD déng phing. diéu ee 1a mau thudn. Bails: a. Nhan xét ring ba mat phang phan biét (PQRS), (ABC), (ACD) cat nhau theo ba giao tuyén PQ, SR, AC, do dé ba dudng thing PQ, SR, AC hoac doi mot song song hoac déng quy. b. Nhan xét rang ba mat phang phan biét (PQRS), (ABD), (BCD) cat nhau theo ba giao tuyén PS, QR, BD, do d6 ba dudng thing PS, QR, BD hodc doi mot song song hoac déng quy. BAi 16: Dap s6 trac nghiém a). B;b). D. a. Tacd: (ABC) 4 (ACD) = c =P I (ABC) 4 (PQR) = PR = Qx//PR/JAC. (ACD) 9 (PQR) = Qx PR/J AC b. Ta cé: (ABC) (ACD) = AC (ABC) > (PQR) = PR (ACD) 9 (PQR) = Qx. PRAAC= {I} => Qy, PR va AC déng quy tai I. Gia sit Qx cit AD tai Sthi S chinh Ia giao diém cita nghiém A. u lun: Xét ABC voi 1, R, P thing hang, theo dinh Ii Melélaus, ta PARB IC _, _, IC _1 B PB RCIA IA 2° Xét AACD voi 1, Q, S thing hang, theo dinh If Mélélaus, ta dug SA QD IC 1, SA 4 554 = 28D, dpc SD QC ‘IA sD 68 a. N6i AG cat BN tai A’, ta can chimg minh A’ [a trong tam ABCD. Ké MM song song vdi AA’ (M' € BN), khi d6: MM" :a dudng trung binh cia AABA’ = M'B=MA’. (dl) GA‘ la dudng trung binh cla AM'MN => M'A'=NA‘. (2) ‘Tir (1) va (2) suy ra BA’ = 2NA’. b. Dap sé trac nghiém B. Loi gidi ne lugn: Xét AABA' voi M, G, ‘ thang hang, theo dinh li Mélélaus, ta GAL! Ga =3Ga', dpem. GA 3 Bai 19: Dép s6'trdc nghiém a). B; b). B;c). A;d). Bye). A. Ménh dé nay sai, boi a va b c6 thé ct nhau. Ménh dé aay sai, béi a va b c6 thé cat nhau. Ménh dé nay ding ~ Ban doc tie gidi thich. é nay sai, bdi a va b c6 thé cat nhau. 6 tric nghiém a). B;b). A;c). By d). Ave). By). A. sai, boi (P) c6 thé chia b. Ménh dé nay dting - Ban doc tu gidi thich. Ménh dé nay sai, béi (P) c6 thé song song véi b. Ménh dé nay ding, boi gid sit: a O.(P) = [M]} = (a, b) 0 (P) = Mx. ‘Trong mat phang (a, b) vi a song song véi b va a cat Mx tai M nén b ciing sé BOTP BO Aalo Te A M N ‘dc nnghiém a). Ab). A. a. Nhan xét ring: MN // BC c (BCD) = MN // (BCD). B c b. Tacé: (ABC) 4 (BCD) = BC x 7 (ABC) (DMN) = MN - (BCD) 4 (DMN) =d MN // BC Bai 22: Dap sd tric nghiém a). Arb). Ayo). A. a. Thiét dién co thé 1a hinh thang, cu thé néu mat phing chtta MN (véi_ _M © ABvaN € AC) va song song vi AD. => d//MN// BC c (ABC) = d // (ABC). 69 Khi d6, thiét dign duge xéc dinh nhu sau: x + Trong (ABD) ké Mx song song véi AD va cit BD tai F. N = Trong (ACD) ké Ny song song véi AD va cat BD tai E. B c Tir d6, suy raz F E NE // MF => MNEF a hinh thang. D b. Thiét dién c6 thé la hinh binh hanh, cu thé néu mat phang di M (vdi M € AB) song song véi AD va BC. Khi do, thiét dién duoc xdc dinh nhu sau: = Trong (ABC) ké Mt song song véi BC va cat AC tai N. * Trong (ABD) ké Mx song song v4i ADB va ct BD tai F. * Trong (ACD) ké Ny song song véi AD va cat CD tai E. A "ASE \/ ° D Khi d6, tir cach dung ta suy ra MF // NE. qd) Mat khdc, ba mat phang (MNEF), (ABC) va (BCD) cat nhau theo ba giao tuyén MN, BC, EF va MN // BC nén MN // EF. (2) Tir (1) va (2) suy ra thiét dién MNEF 1a hinh binh hanh. c. Thiét dién cé thé 1a hinh thoi, cu thé véi thiét dién dugc dumg nau trong cau b). Khi d6, dé MNEF 1 hinh thoi diéu kign la: MN = MF. “ Mn= AMBC | 3) AB Mrs 2D BMG (a) AB . aw x, AM.BC _ AD.BM AM AD 6. é *) tro tha i = = —_—=—.. Khi d6, digu kign (*) ted thant: ANE pe a -& : . AM _ AD i qua dié v6 B sao cho AM _ AD Vay, mat phang (P) di qua diém M (vdi M € AB sao cho EM" BC ) song song véi AD va BC sé cit ttt dien theo mot thiet dién la hinh thoi. Bai 23: Dap s6'trdc nghiém D. A Lai gidi tu ludn: Ta C6: ni (ABC) 9 (BCD) = BC MNE)= B c as MN Bx // MN/IBC. (BCD) 9 (MNE) = Ex x k MN // BC D Gia sir Ex cit BD wi F, Vay, thiét dién 18 hinh thang MNEF. 70 dign duge xc dinh bang cach: + Trong mat phing (ABCD) ké Ox song song véi AB, Ox cat AD va BC theo thi ty tai M va N. + Trong mat phang (SBC) ké Ny song song véi SC, Ny cat SB tai P. + Trong mat phing (SAB) ké Pz song song v6i AB, Pz cat SA tai Q. Khi dé, tt giéc MNPQ [a thiét dién can dung. b. Dap sé tréc nghiém A. Loi gidi ur ludn: Boi MN va PQ cling song song véi AB nén: MN // PQ => MNPQ 1a hinh thang. Bai 25: Dap s0 trdc nghiém C. Léi gidi ne ludn: Thiét dién duge xéc dinh bang cach: * Trong mat phang (ABCD) ké Mx song song voi BD, Mx cat AC va AD theo thtf tu tai I va N. = Trong mat phing (SAB) ké My song song vi SA, My cit SB tai R. * Trong mat phing (SAC) ké Iz song song véi SA, Iz cat SC tai Q. * Trong mat phing (SAD) ké Nt song song véi SA, Nt ct SD tai P. Khi d6, ngii gidéc MNPQR 1A thiét dién cin dung. Bai 26: Dap sé tréc nghiém a). B;b). A;c). A;d). Bye). By f). A. a, Ménh dé nay sai, bdi theo hé qua 2 ciia bai hoc 3 thi hai mat phang d6 cé thé cat nhau. b. Ménh dé nay ding theo hé qua 2 c. Ménh dé nay ding - Ban doc tur gidi thich d. Ménh dé nay sai, béi hai duémg thang nhu vay cé thé chéo nhau e. Ménh dé nay sai, béi hai mat phang d6 cé thé cat nhau. f. Ménh dé nay dung -— Ban doc nu gidi thich. Bai 27: Dap sé trac nghiéma). A:b). B;c). Bid). Aze). A. a, Ménh dé nay diing - Ban doc tw gidi thich. b. Ménh dé nay sai, béi canh ben khong thé song song v6i canh day. c. Ménh dé nay sai, béi khi hai canh day khong bang nhau thi hai mat ben tuong tmg cling khong bang nhau. ad. Ménh dé nay ding - Ban doc tit gidi thich. e. Ménh dé nay ding — Ban doc tit gidi thich. Bai 28: Ddp sé trdc nghiém C. Léi giai tu luén: Néu a va b cing song song véi mat phang (P) thi chiing cé thé cat nhau hodc chéo nhau. 7 Bai 29: Dap so irc nghiém A. Loi gidi ne ludn: Lay dutng thang a bat ki thudc (a). Gia sit trdi lai a khong song song vdi (B), suy ra: a (B) = {M] => M 1a diém chung cita (ct) va (B) — Mau thudn. Vay, néu hai mat phang (a) va (B) song song véi nhau thi moi duéng thing nam trong (a1) déu song song véi (B). Bai 30: Dap s6 trac nghiém A. Loi gidi ne ludn: Nhan xét rang: - Mat phang (M, a) la duy nhat. - Mat phang (M, b) 1a duy nhat. - Vi(M, a) #(M, b) béi a va b chéo nhau nén (M, a) > (M, Lb) = Mx. Khi do: = Mx Kh6ng thé song song véi a ( mau thudn (Mx, b)) do dé Mx néu trai lai thi Mx va b sé chéo nhau — * Mx khong thé song song véi b (vi néu trdi Jai thi Mx va a sé chéo nhau — mau thudn (Mx, a)) do dé Mx cat b. 6 duy nhat n ot dudng thang di qua M cat cd a va b. : Dap sé trac nghiém B. Loi gidi ue ludn: Nhan xét rang: {(a,by/e.d) (A'B'C'D') A (a.b) = A'B' = A'B // CD. | A'BIC'D')A(¢,d)=C'D' (a,d) (b,c) (ABCD) A(a.d)=A'D' = A'D'// BC. (ABCD) A (b,c) = BC Tudo, suy ra A’B'CD' la hinh binh hanh. Dap so tréc nghiém D. Dap so tréc nghiém D. [Ax/I Ce a, Nhan xét rang: | AB//CD = (Ax, By) // (Cz, Db. a b. Nhan xét rang: (a,b) /Mc.d) (A'B'C’'D') A (a,b) =A'B! = A'B // CD. (ABCD) a(c,d)=C'D' (a,d) /[(b,c) (A'B'C'D') A(a,d)=A'D' = A'D'// BC. (A'B'C'D') 4 (b,c) = BIC’ Tir dé, suy ra A°B'C’D' 1a hinh binh hanh. Suy ra IJ la dung trung binh cia hinh thang AA'CC, do dé II // AA. 2 c. Dap so tréc nghiém B. Loi gidi ng luan: Tic két qua cau b), ta C6: w= Scan’ cc) = 2 (BB + DD) => DD'= AA'+ CC - BB =x +y~z. =. Dap sd wdc nghiém A. ii tu ludn: Mat phang (P) cé di qua trung diém N cia CD, bdi: = Mat phang (Q) chtta AD va song song vdi BC. A * Mat phang (R) chtta BC va song song véi AD. Khi d6, ba mat phing (P), (Q), (R) song song véi nhau sé chan trén hai cat tuyén AB va CD cdc doan IS thang tuong tg ti lé, cu thé: Def B aM BM AB DN AM {\ EN EN ofc CN BM 7 N => N la trung diém CD. c a. Gid si: AC’ A'C = (N} = N 1a trung diém AC’ va A'C => BC// NH ~ tinh chat dutmg trung binh => BC // (AHC), dpem. b. Gia sit; AB’ A'B= [M} = (A'BIC) 0 (A'BO) = MN. Tir tinh chat dudng trung binh, suy ra: MN // BC c (BB'C'C) = MN // (BB'C'C), dpem. c. Dap sé tréc nghiém C. Léi gidi ne ludn: N6i MH cat AB wi P (P a wung diém AB), khi d6: (H, d) > (ABC) = Px // MN // BC, => Px cat AC tai Q (Q la trung diém AC). (H, d) 0 (A'B' Hy // MN // BC // B' => Hy cit A'C tai R (R 1a ung diém A'C)). Khi d6, ta duge thiét dién 1a hinh binh hanh HPQR. Bai40: Trudc tien ta di chimg minh ménh dé “Téng bink phuong cdc dudng chéo ciia mot hink binh hanh bang téng binh phuong cde canh". ‘That vay, vdi hinh binh hanh ABCD, theo dinh li ham sé6 césin ta cé: AC ~2AB.BC.cos ABC. (1) D c BD? = CD” + CB - 2CD.CB.cos BCD. (2) = CD’ + DA?-2AB.BC.cosBCD. (2) B Cong theo vé (1) va (2), ta duge: AC + BD? = AB’ + BC + CD? + DA? 2AB.BC(cos ABC. + cos BCD) = AB? + BC? +CD? + DAY, dpem. 73 St dung ménh dé trén, ta thay: D Cc DB'+BD° . =A'A7+ AC + CC + CA + : +BD +DD +DB’+BB" c (AC? + BD*) + (A'C? + BD”) A B =(A'A’ + BB’ + CC’ + D'D*) + (AB’ + BC’ + CD’ + AD?) + + (A'B? + BC? + CD? + A’D®), dpem. ai41: Dap so trdc nghiém a). Bib). A. 42: Dap so tac nghiem a). Bib). A. ig, Khi ta chiéu chung theo phuong chiéu la m6t dudng thang song song Vi mat phang (P), biét (P) song song v6i hai duéng thang chéo nhau do, i — Ban doc tif gidi thich. A phép chiéu song song bao toan tinh song song cuia céc dutmg thing. . béi chting cé thé tring nhau. Bai 44: Dap so irde nghiema). Azb). A a. Dang, boi khi ta chiéu theo phuong chi mat phang chtta hai dudng thang cat nhau do. b. Diing, khi mat phang chi¢u song song v6i dutme thang do. Bai 45: Dap so trae nghiem a). Bib). A. a. Sai - Ban doc tit gidi thich. b. Dung, khi mat phang chiéu chita duémg thang d6. Bai 46: Dap so trac nghiém C. L6i gidi ne ludn: Thue hién phép chiéu song song: * Mat phang chiéu (ABCD) va phuong chiéu BC,. * Tanhan duge anh cia B, 1a diém N. * Noi ND cat AB va AC theo thi ty tai M va J. * Qua J ké dudng thang song song véi BN cat B,D tail. Suy ra U/// BC, y 1a mot dudng thang song song Voi Tit cdch dung. ta c6 A, BNB,C, la hinh binh hanh => BN = B,C,. Tir do: _ JN CN 2 N—- ID _ wet Trong ANDB,, ta cé: — = =~. rong a 1B, JN 2 a Bai 47: Dap s6 tae nghiém a). Avb). Bic). Az d). B. a. Ding. : b. Sai, boi hai dug thing song song cing khong cé diém chung. c. Ding. d. Sai, béi hai duémg thing song song thi déng phing con hai duimg thang chéo nhau thi khong déng phang. 4 Dap 36 trac nghiém a). B;b). Byc). Ard). Ave). Bf). Arg). A. a. Sai, boi khi dé chting c6 thé cat nhau hodc chéo nhau b. Sai, béi néu hai mat phang cat nhau theo giao tuyén a va a //t thi hai mat phang d6 déu song song voi b. c. Ding. \ tréc nghiém C. in luot thuc hién: . Mo. . * Trong (AA'D'D) ké NP // AI voi 1 1a trung diém DD’ (ta cé Al // MO). - * Trong (CC'D'D) néi PO cat CC tai Q. * Trong (CC’B'B) ké QR // BJ voi J 1a trung diém CC (ta c6é BJ // MO). » Noi MR. Khi d6, thict dién la ngii gidc MNPQR. 61: Dap sé trac nghiém A. 62: a. Tirgid thiét, ta suy ra: AM = 2MO = M 1a trong tam AABD. BN = 2NO = M Ia trong tam AABE. Suy ra DM 7 EN = {I} [a trung diém ca AB vatacé: x =1N 2 Ls MN//DE, dpom. b. Tacénhan xét; AM. AM _ BN _ AN: Mn, DF a AD “AC BF AF => MN, // (DEF), dpem. c. Mat khac, ta cé: MM’ // CD. (2) Tir (1) va (2) suy ra: (MNN’M’') // (DEF). Bai 63: a. Goi M va M' theo thi tu 1a trung diém cua BC vi va B'C, suy ra: 2 pp cc 2 AA, MMO B,C, = {M,}, v6i M, la trung diém B,C. , 4 Trong (AA'M'M) ta co: AG. AS" 2 = aq = am, AM AM 3 tir d6, suy ra GG' song song va bang canh bén cia hinh lang tru. 75 s. AiG, AG 2 B M b. Tacd; S101 AG 2 (a) is AM, AM 3 \I 7 a B = G, 1a trong tam cia tam gidc A,B,C. 1 c. Tirkét qua (*), ta cé: B pol ee GiG= SAA + SMM'= SAA + .5( BB +CC) = FAA +BB+ CC). Chimg minh tuong tu, ta cing c6 G,G = aa +B,B+C,©. Bai 64: D a. Tirgid thier; AM _D'N Sh AB D'D A suy ra MN, AD’, BD thudc ba mat phang doi mot song song véi nhau. Vi béi BD // B'D' nén: MN //(ABD). a) 4 ~ cing. AM _ BP Tir gid thiét:. —— =—— Dee tet A M B suy ra MP, AB’, BC thugc ba mat phang doi mot song song v6i nhau. Vi béi BC // AD' nén: MP // (ABD). 2) Tir (1) va (2), suy ra (MNP) // (AB! b. Dé c6 dug thiét dién, ta thuc hien: + Ké Mx // BD va cat AD tai S. = NOiSN. ; + Ké Py // BD’ va cit CD' tai R. + KéPz//BC va cat BB tai Q. Khi dé, luc giéc MSNRPQ 18 thiét dién can dung. Bai 65: a. Goi My, Ny lin luct Ja hai diém e6 dinh thuge céc tai Ax, By sao cho: AMo BNy Tird6, suy ra; AM _ AM BNy BN = MN, MoNo, AB theo thif ty thudc ba mat phang doi mot song song véinhau. Vay, ta co két luan rang MN song song voi mat phang c6 dinh (P) chita M,N, va song song voi AB. : tir dé ké Ox’ va Oy’ theo thit ty song song vdi Ax va By. Tir d6, theo tinh chat t dudng phan gidc ta cé két Tuan tap hop cdc diém J 1a tia phan gidc Oz ciia géc x’Oy’ . 76 Bai a. Gia sir AC va BD khong chéo nhau, suy ra: AC va BD déng phang => AB va CD déng phing diéu nay mau thuan véi gid thiét. Vay, ta c6 AC va BD chéo nhau. b. MN khong thé song song véi AB hoac CD boi: = Néu MN // AB thi: MN va AB déng phing => AM va BN déng phang © AC va BD déng phing diéu dé mau thudn, = Néu MN //CD thi: MN va CD dong phang => CM va DN dong phang = AC va BD dong phang. mau thuan. c. Ta lan tuot: = Ching minh AO cat CN: Ta cé:, O € MN = O &€ (CMN) va A € CM => A € (CMN) = AO vACN déng phing. CN khong thé song song véi nhau boi néu: AO // CN n ngoai doan MN, mau thuan véi gia thiét. Vay. ta cd ket luan AO cat CN. ‘Tuong ty ta chimg minh duge BO cit DM. s ir. SO =0, : O,EUCPB>O, ef, ©, € SOc (SBD) = 0, € (SBD). T Suy ra: O, € 8 9 (SBD) = MN y. ba dudmg thang J. MN. SO déng quy tai ¢ Nhu vay, khi biét diém M ta chi can noi MO, cat SD tai N. b. Nhan xét rang: (SAD) > (SBC) = (S. F thing hang. c. Trude tién, vi J khong song song voi AC nén: VOac 1a mot digm cd dinh. Nhan xét rang: B > (ABCD) = (1. P.Q) => T. PQ thang hang > PQ ludn di qua diém c6 dinh T. : i 68: Ta lain luot thuc hién + Noi MN cat SO tai O, * N6iO,P cat SA tai § Vay, ta duge: (MNP) 9 (SAC) = PS,. (MNP) 9 SA = S, b. Ta lan luot thuc hién: + N6i S\N kéo dai cét ADD, DD « N6iS\M kéo dai cat AB tai B, + N6i B,D, cat CD. CB theo thir urtai Ds. By 7 Khi d6, ta duge 5 doan giao tuyén la S\M, MB,, B,D,, D,N va NS,. Do dé thiét dign can tim la da gidc S;MB,D,N. c. Ta lin luot cé: * MN la dudng trung binh cia ASBD nén O, [a trung diém SO, suy ra: Po,//sC => WS = PC 21, SVA BPA a3 * -Xét ASAD vi S,,.N, D, thang hang, theo dinh li Mélelaus, ta duce: § iS GH) GA | oe eee 7 1 S; ND'D,A’S,S DA 3 eo NY" + Xét ASAB v6i S,, M, B, thing hang, theo dinh Ii Mélélaus, ta duoc: D, D A =U Q) 5 MB BA’ S,S BA gS Tit (1), (2), suy ra: Ss, BD // B,D, => B,D, la dudng trung binh cla ACBD => nén B,, D, theo thi ty [a trung diém BC, CD do dé: oa =lva BB 1s D,C B,C Chai ¥: Dinh If Melélaus 6 noi dung nhu sau: " Trén cdc canh AB, BC, CA ciia AABC (hod trén phan kéo dai ciia ching) ldy cae diém C,, A,, B, thi C,, A, By B BOA thing hang khi va chi bhi: 8B BE GA yw, A,\CB,A CB S$ x AD (SAD) va BC « (SBC) M N AD // BC = Sx //ADIIBC. fA RK (SAD) (SBC) = Sx * VV a AD € (SAD) va BC (BCI) B a b. Tacd: | AD//BC = ly AD// BC (SAD) (BCI) = ly va ly cat SA, SD theo thi ty tai M, N. AD (ADJ) va BC € (SBC) c. Tacé: }AD// BC = Jz// AD// BC (ADJ) (SBC) = Jz va Jz cat SB, SC theo thi ty tai E, F. d. Gia sit AE ct BM tai H va CN cit DF tai K, va ta can di tinh d dai HK. Ban doc te lam dua trén dinh liTa ~lét, dip 86 HK = z (a+b). 78 KH € (MNKH) va AB (ABCD) S KH//AB 7 D (MNKH)4(ABCD) = MN LX B NC => MN // AB// KH. a Ma: khic, ta lai cé: ASAD = ASBC (c.c«.) => SAD = SBC => AHAM = AKBN (c.g.c) => MH = SK (2) Tir 1) va (2) suy ra MNKH 1a hinh thang «an H b. Ta 26 ngay: MN =aya KH => AB= 4 \ Troig ASAD, ta c6: SA? +AD? -SD* 2SA.AD_ = mir -(MN=HK) =! Jia” ax +3. a) 1 Te eee geet Suinkn = 3 (MN + KHDHP = Bas 16x? - 8ax + 3a* = 3 Vi6x* = ax + 3a? : 3 Ta 6 bién déi: Syygi = ie Vay, ta duge (Syinkitain = ae dat duge khi x = : c. Huteng dan: Qui tich la doan SE, trén St // AD. 719 Bai 71: a. Trong hinh binh hanh ABCD, ta cé MN Ia dudng trung binh, do 46: MN // BC c (SBC) => MN // (SBC). 7 MN // AD c (SAD) => MN // (SAD). b. Trong ASAB, ta cé MP [a dudng trung binh, do dé: Q SB // MP c (MNP) => SB // (MNP). f NS c. Tacé thé Iya chon mét trong hai cach sau: x c Cach I: Ta 06: A M B AD < (SAD) va MN € (MNP) AD // MN = Kx // AD // MN. (SAD) 0(MNP) = Px Gia sit Px cit SD tai Q, suy ra Q 18 trung diém SD. Trong ASCD, ta c6 NQ [a dudng trung binh, do dé: SC /] NQ < (MNP) => SC // (MNP). Céch 2: Goi O 12 trung diém MN, suy ra O [a trung diém AC. Trong ASAC, ta cé OP 1a dudng trung binh, do dé: SC // OP < (MNP) => SC // (MNP). 4d. Goi Kf trung diém SB. Ta c6: Cot CK a = GG, // MK. (1) Mat khc, trong ASAB, ta co MK 1a duéng trung binh, do dé: MK // SA. (2) Tit (1) va (2) suy ra: G,G, // SA c (SAD) = G,G, // (SAD). Bai 7: a. Ta lan lugt cd: SA // a SA c (SAB) => MQ//SA. MQ €(SAB)na BC//a MN € (ABCD) Va = MN // PQ// BC. PQ (SBC) Na Nhan xét ring: MQ - BM _ BQ _ CN _CP _ NPS? vig einip, SA BA BS CD CS. SD Vay, thiét dién MNPQ la hinh thang can. 80 b. Gia sti AB cat CD tai I, ta 6: a AD = ; BC => AD [a dung trung binh ciia AIBC MN _ IM _IA+AM _ b+x a(b +x) do dé IA = AB=b va: = => MN= Be) 1h) IA+ AB nC weg: PQ_SQ_AM_X Trong ASBC, tac; PQ . . a © Bc" sp AB be . MQ_BM_b-x Trong ASAB, tacé:. MQ. BM _b=* _ iy — SA AB Ob — Xét hith thang can MNPQ, ha duéng cao QH, ta c6: a=: JMG? one Q’ (MN-PQY _ Quis (MQ - ME = yma a oercaees|t Gases ; 1 (b+x) 2ax] V3a(b-x) Sieg = (MN + PQ).QH = ab ax) Baoan) a, DungJIH// AB. H © AC. Nhansét ring: YA 2 BLA yep. HC Ic” 1D Gi ola mat phing chity AB va song song v6i CD, suy ra at 1k mat phing c6 (HI) ff air (HJ) eat BD tai K, dé thay HIKJ 1a hinh binh hanh. Qua M ké PQ sony Voi AB (P ¢ HI-vaQ € JK). Tacé: APA BQ = E va EM AB=F. Nhguvét ring: PD. PLM! 24 Big diém chia CD theo i sé k. - “EC PH MJ 1A MPMI 1B MQ MJ =k => F la diém chia AB theo ti sok. 81 Vay, tap hgp diém M chia doan IJ theo ti s6 k 1a doan EF véi E, F lan lugt | 4iém chia CD va AB theo ti s6 k. Bai 7. | a /(SAB) a. Ta lan luot cé: { MN = a (ABCD) = MN // AB. [AB = (SAB) Lap luan tuong tu ta ciing cé: NP // BS. PQ // CD, QM // SA. Nhan xét rang: MN // PQ bdi AB // CD. Fl IP s=sa MQ ., DQ cP _ NP SSS” MQ=nNP. S DS SB Vay, thiét dign MNPQ la hinh thang can, ¥ b. Dé MNPQ ngoai tiép duge mot dugng tron diéu kign 1a: MN + PQ=MQ+NP = MN+PQ ay iC N Trong ASAD, tacé; M@ = PM_2=*§ uQgera-». 2) SA DA a Trong ASCD, 1a c6; PQ = 5Q_AM _* = pgex, @) cD Sb AD a Gia sit AB cit CD tai O va OD = y, ta cd: P 0 ae =4 s3ysatyoy= OA moo N a aye N HM MN _ OM _ OD+DM _ 37"7* s Bai 7: a. “= = = 4—— => MN=3a-2x. (4) AB OA OD+DA a Vay, véi x = . thi MNPQ ngoai tigp duge mot dudng tron. Khi dé, xét hinh thang cin MNPQ, ha dung cao QH, ta cé: QH = (MQ? -MH? = (v0? (58) eros 4 uy ra ban kinh dudg tron noi tiép MNPQ [a r = 30H = Cau c) va d) hoc sinh ti lam. Tir gia thiét, ta duge: 7d MM, “ BB, = AA, <= AMM,A, la hinh binh hinh => AM//A,M,. b. Chon mat phang phy (AMM,A,) chtta A,M. Nhan xét rang: (AMM,A,) > (AB,C,) = AM, AM, 0 A\M= I. suy ra AM (AB,C,) = L c. Goi O = AB, 4 A,B, khi dé ta nhan duge: B M, (AB,C,) 9 (BA,C,) = OC,, | chinh 1a duong thing d can tim. Nir d. Chon mat phang phy (AB,C,) chtia d (chifa OC,). Nhan xét rang: (AMA,) 0 (AB,C,) = AM, a c AM, 9 OC, =G. suy rad (AMA) = Dé thay G 1a trong tam AAB,C,, bdi trong AAB,C, thi G 1a giao diém cua hai dudmg trung tuyén, Bai 76: a. Nhan xét ring: 24: = = WJ // BC c (ABC) = I // (ABC). b. Ta lin luot c6: We (UO) va BC e (ABC) LBC => Ox // U // BC. (JO) (ABC) = Ox va Ox cat AB va AC theo thif ty tai Eva F. Noi El cit A,B, tai H va noi FI cat A,C, tai G. Nhu vay, thiét dién Ia ti giée EFGH. (ABC) A,B,C) Nhan xét rang: 1tsorecases =EF => EF //GH = EFGH 1a hinh thang. (JO) A(A,B,C,) = GH Vi AABC nén AA,B,B = AA,\C,C, do dé EH = FG. Vay. thiet dién EFGH [a hinh thang can. B * Trong ABC, ta c6: EE our= 3, + Trong AA,B,C,, ta cd: HG - AH | BE! igs’, BC, AB, BA 3 3 + Trong AIBE, ta c6: IE? = BI? + BE? ~ 2BI.BE.cos iBE 83 Khi d6, xét hinh thang can EFGH, ha dung cao HM, ta cé: HM = JeH?—Me? = eH? -{EE—HG) l LN Son = SEF +HG).HM -1 (448) av39 _ a? V39_ C= i Cha ¥: Trong loi a. Gcau a), ching ta cé thé str dung nhan xét: ” VJ 1a duéng trung binh cla AA,BC <> IJ = 3Bc (ta c6 I = b. Khi d6, trong cau b), ching ta c6 thé tinh do dai HG dua uén tinh chat ua dong, tung binh ciia hinh thang EFGH nhu sau: Ue IEF + HG) = HG = 2) - EF = 25 Bee Bai 77: True tien ta di chimg minh ménh dé “ring binh phucong cde dang chéo ctia mét hinh binh hanh bang ting bink pluong cdc canh". ‘That vay, véi hinh binh hanh ABCD, theo dinh If ham s6 cosin ta e6 AC? = AB? + BC - 2AB.BC.cos ABC. (1) D c BD? = CD’ + CB? - 2CD.CB.cos BCD. (2) LJ = CD? + DA? - 2AB.BC.cos BCD. (2) Cong theo vé (1) va (2), ta dude A B AC + BD’ = AB’ + BC’ + CD’ + DA? - 2AB.BC(cos ABC. + coz BCD ) = AB’ + BC’ + CD? + DA*, dpem. D Cc Sir dung ménh dé trén, ta th / : A'C + CA? + DB + BD? = M B =AA+AC+CC4+CAT+ + BD? + DD? + D'B? + BB’ D ic = VA? + BB’ +CC'+ DID?) + (AC? + BD’) + (A'C’ + BD") A NY Seber Gee DD +BC'+CD' + AD‘) + +(A'B? + BC? + CD? + : Bai 78: a. Goi O. O, theo thi ty 1a tam cua cdc hinh binh hanh ABCD va .\,B,C,D,, ta co: A,O//CO, O11, BDA) I(B,D,O. BD//B,D, b. Vi AC, AO, CO, cing nim trong mat phing (ACC,A,) nén goi: G=AC, 0 A,0 vaG, = AC, ACO). 84 = Trong AA, BD, diém G thudc trung tuyén A,O va vi AO // AC, nén: GO ao __! GA, AC do d6. G la trong tam AA,BD. = Ching minh tuong ty G, 14 trong tam ACB,D,. Nhan xét rang OG, O,G, theo thet tr la dutng trung binh cia AACG, va AA,C,G nén: AG = GG, = G,C, tuc 1a G, G, chia doan AC, lam 3 phan bang nhau. c. Kéo dai B,G, cat CD, tai P. ta c6 P 1a trung diém cua CD, vi G, la trong tam ACB,D, [a B, €(A,B,G,) va C,D, €(CDD,C,) Tacd: {A,B CD, => Px // A,B, // C,D, ((A,B,G,) 0(CDD,C,) = Px gid str Px theo thi ty cat CC, va DD, tai M va N. Khi d6, ta nhan duge thiét dién A,B,MN 1a hinh binh hanh. d. Ta lan lugt c6: = Trong (A,B,CD) gia sti: A,KODC=1. A * N6i IO cat BC va AD theo thit ty tai E va F «Noi KE cat B,C, tai H. N6i A,F va A,H nhan duoc thiet dién A,FEH [a hinh binh hanh. Bai 79: * a. Goi O, O,. I theo thi ty 1a tam ciia céc hinh vuong ABCD. A,B,C,D, va BCC,B,. . wy a4 a Nhan xét ring: MO = AD = > A,D, = <2 MOGN [a hinh binh hanh = MN//OC, ad) ee ae eee NI = —BB, = +AA, = ~DD, = PD 2 2 a ¢ NIDP [a hinh binh hanh = PN// DI. @ “Tir (1) va (2) suy ra (MNP) // (BDC)). AO, /C,O mar khsc: [A/G _, cag Dy BDC) B,D, // BD Vay. (MNP) song song vGi cac mat phang (.AB,D,) va (BDC, ). b. Tirkét qua cau a), ta nhan xét {(AB,D, )//(MNP) J(AB,D,) 0(A,B,C,D,) = B,D, [OMNP) A(A,B,C,D,) = Nx suy ra Nx song song véi B,D, va cat C,D, ti FR trung diém ctia C,D,. (C,BD)/( MNP) (C,BD) 4( ABCD) = BD (MNP) ABCD) = My suy ra My song song véi BD va cit AD tai Q la trung diém cua AD. Kéo dai FN cat A,B, tai G. néi GM cat BB, tai E \igy. thiet dign ctia hinh lap phuong voi mat phang (MNP) Ia luc giéc MENFPQ. Dua theo tinh chat dudng trung binh ta thay nay MENFPQ [a luc gic déu 6 d6 dai canh bang =. Khi do: Sys Bai 8 a. Tirgia thidt, ta duoc: GK // AD, AG 0 DK = E la trung diém BC, suy ra: oS me =} = K la trong tam ABCD. 86 CHUONG III. VECTO TRONG KHONG GIAN QUAN HE VUONG GOC § 1. VECTO TRONG KHONG GIAN x 2 > : SU DONG PHANG CUA CAC VECTO 1, KIEN THUC CAN NHO 1. VECTOTRONG KHONG GIAN Vecto, «i phép todn vecto trong khong gian duge dinh nghia hoan toan giéng nhu ircg mat phang, chting cé céc tinh chat da biét. Quy tde hinh hop: Cho hinh hop ABCD.A,B,C,D,, ta luon c6: = AB+AD + AA, Trong tam cia ae : Diém G li trong tam cia tt dién ABCD khi va chi khi: GA + GB + GC + GD =0. ; 2. SUDONG PHANG CUA CAC VECTO. DIGU KIEN BE BA VECTO DONG PHANG Dinh nghia: Ba vecto duge goi la dong phdng néi gid ctia ching song song véi mot mat phang. Dinh li I (Diéu kién dé ba vecto dong phdng)y: Cho ba vecto khong cing phuong vecto a va b. Khi do ba vecto a, b, ¢ déng phang khi va chi khi c6 cdc s6 m,n saocho ¢ =ma +nb. Hon nia, céc sm, nla duy nhac. Dinh li 2: Néu ba vecto a, b va ¢ khong déng phing thi véi vecto d bat ki, ta déu tim duge cdc s6 m,n, p sao cho d = ma +nb + pe. Hon nila, cdc s6 m,n, pladuy nhat. i. BAI TAP TRAC NGHIEM VA TULUAN Bail: Ba vecto a, b, ¢ c6 déng phang khong n€u mot trong hai diéu sau day xay ra? a. C6 mot vecto trong ba vecto dé bing 0. A. C6. B. Khong. b. C6 hai vecto trong ba vecto dé cing phuong. A. C6. B. Khong. : Cho hinh hop ABCD.A’B'C’D". Tim gid ti cua k thich hop dién vao dang thifc vecto: a. AB+BC'+DD'=kAC. A. k=0. B. k=1 C. k=2. D. k=4. 87 c. AC+BA'+k(DB+C Dy =o. A. k=0. B. k= =2. D. k=4. Bai 3: Cho hinh tt dién ABCD. Goi M va N lan luot 1a tung diém cia AB va CD. Tim n gid tri cia k k thich hop dién vao dang thie vecto: a. MN= k(AD + BC). Ae go bee c. D. k=. 2 3 b. MN =k(AC+ BD). A k= >. Bok=2 cr p.k=!. Bai 4: Cho hinh tt dign ABCD. Goi G Ia trong tam cia tam gic AE ABC. Tim gi trj cua k thich hop dién vao dang thtte vecto: Das +DB+DC=kDG. A.k=! ae ce D. k= 4 Goi M va N lén luot 12 trung diém ciia cde canh AC va BD cla tt dién ‘ABCD. Goi I fa trung diém cua doan MN va P fa mot diém bat ki ong khong gian, Tim gia tri cla k thich hop dign vao ding thiic veeto: a. TA+(2k-1DIB+kIC+ID =0 A. k=0. B. k C. k=2 D. k=4 b. Pl=Kk(PA + PB + PC + PD). Ake, B. Cc. k=4, 2 Bai 6: Cho hinh chép S.ABCD. Chtmg minh ring néu ABCD 18 hinh binh hanh thi SB + SD = SA + SC. Diéu nguge la dung khong ? b. Goi O a giao diém cua AC va BD. Chimg t6 rang ABCD 1a hinh binh hanh khi va chi khi SA + SB+SC + SD = 480 , Trong khong gian cho AABC, a. Chtmg minh rang néu diém M thudc mat phang (ABC) thi cé ba s6 x, y,z ma x+y+z= 1 saocho OM = xOA + yOB +z0C véi moi diém Q. b. Neuge Jai, néu co mot diém O trong khong gian sao cho OM = xOA + yOB + 20€, trong dé x + y +2 = | thi diém M thuge mat phiing (ABC). Bai 8: Cho hinh chop S.ABC. Lay ede diém A’, B', C’ lan lugt thudc cac tia SA, SB, SC sao cho SA = aSA’, SB = BSB’, SC = cSC’, trong dé a. b, ¢ 1a cac 86 thay 88 dei, Tim moi lién he gid a. be ede mar phang (ABC) di qua trong tam cia AARC. Av atb+c=l Couthtes3. B. atb+c=2 Do atb+esd. Cho hinh lang tru tam gic ABCC’ 66 AN © a, ABS a. Hay phan tich (hay bic thi) vector BC qua cae vecto a, A. BC a+ hee c. BC B. BC =a+b-< Dz b. Hay phan tich (hay bicu thi) seeto B A. BC =at+b—c C B. BC =a-b+e D. Bai 10: Trong cac ménh de sau day. menh dé nao la ding ? A. Tit AB=3AC tasuy ra BY B. Tit AB=~3AC tasuy ra CBAC C. Vi AB = -2AC + SAD nen bon diém A, B.C. D ciing thude mot mat phiing. D. Néu AB=—! 5 HC thi H ta rung digm eta doan AC. -atbee U1: Tim ménh dé sai ailirong cde ménh dé sau day A. Vi NM+NP=0 nénN la trung diém cua doan MP. B. Vi 1a trung diém cia doan AB nén tir mot diém O bat ki ta c6 : 1 lox + 0B) oi C. Tit he thie AB = 2AC-8AD ta suy ra ba véc to AB. AC, AD déng phang D. Vi AB BC. CD + DA 0 nén bon diém A, B,C. D cing thugc mot mat phang Cho AABC. Lay diém S nam ngoai mat phang (ABC). Trén doan SA diém M sao cho MS 2MA_ va wen doan BC lay diém N_ sao cho NB = -2 NC. Chiing minh rang ba vécto AB, MN. SC déng phing. 89 § 2. HAI DUONG THANG VUONG GOC 1. KIEN THUC CAN NHO | 1, GOC GIA HAI DUONG THANG BAT Ki TRONG KHONG GIAN Dink nghia 1: Gée gitta hai dudng thang a, b Id gée gitta hai ding thang a’, b’ cing di qua mgt diém va Ian luot song song voi ava b. . Chi: y: Dé xéc dinh (a, b) ta c6 thé lay diém O nim ngay trén mot trong hai duéng thang do. 2. HAI DUONG THANG VUONG GOC _— b Dinh nghia 2: Hai duong thing goi la vudng gée voi nhau néiu gc Dia ching bang 90". Nhan xét: Cho hai duéng thing song song. Dudng thing nao vuéng géc voi dutmg thang thir nhat thi vung géc voi duémg thang thet hai. Tite 1a: { He clb. cla 1. BAI TAP TRAC NGHIEM VA TULUAN Bai 13: Moi khang dinh sau Ia ding hay sai ? ; ai duémg thing ciing vung géc véi dudng thing thif ba thi song song véi nhau. A. Dang. B. Sai, b. Hai duémg thang cing vuong géc v6i duimg thang thi ba thi vuong g6c v6i nhau. A. Ding. B. Sai. Trong céc ménh dé sau day, ménh dé nao la DUNG ? A. Néu dudng thing a vuong géc véi dudng thing b va duéng thing b vuong géc v6i dudmg thang c thi a vudng géc vdi c. B. Néu dutmg thing a vuong géc véi dung thing b va dutng thing b song song v6i dudng thang c thi a vu6ng géc vdi c. C. Cho ba during thang a. b, c vuong géc v6i nhau timg doi mot. Neu cé mot dutmg thang d vudng goc véi a thi d song song véi b hoac c. D. Cho hai dudng thang a va b song song voi nhau. Mot dudng thang c vudng goc vGi a thi c vudng géc véi moi dudng, thang nam trong mat phang (a, b). ; Menh dé nao sau day la DUNG ? ‘A. Hai dudng thing cing vu6ng géc voi mot dudmg thang thi song song véi nhau. B. Hai dutmg thang cing vudng géc véi mot dutmg thang thi vudng g6c v6i nhau. C. Mot dudng thang vuong géc vdi mdt trong hai dudng thang song song thi vudng goc véi dudng thang kia. D. Mot dudng thang vudng géc véi mot trong hai dudng thing vuong g6c VGi nhau thi song song voi dutng thang con lai. 90 Bai l6: a. Cho vecto n khéc 6 va hai vec a. khong cing phuong. Ching minh rang néu vector n vudng gde vai ca hai vector a va B thi ba vécts na va b khong dong phang b. Chimg minh rang ba vecto cing vung géc véi vecta nhac O thi déng phang, Tir dé suy ra. che dung thing cling vuong g6c véi mot dudng thang thi cng song song véi mot mat phing. Bai 17: Cho hinh lap phuomg ABCD.EFGH e6 canh bang a. Tinh AB.EG 22 Aw Ba’ y2 C. a V3 D. 8: Cho hinh ur dign ABCD c6 AB = AC = AD ¥i 60’, BAD = 60". Goi I va J Jan lot la rung diem cla AB va CD. Hay xc dink g6e C cap vector sau day a. AB va CD AL 45", B. 60 C. 90" D. 120” b ABYa D, AL 45" B. 60" C. 90", D. 120". c CD au. A. 45" B. 60) Cc. 90! D. igF. Bai 19: Cho hinh chép S.ABC co Si\ = SB = SC va ASB = BSC = CSA. Hay xdc dinh gée gidta cic cap vecto sau diy a. SA va BC A. 45", B. 60° Cc. 90" D. 120" b. SB va AC A. 45". B. 60) Cc. 90! D. 12 © va AB A. 45°, B. 60 Cc. 90". D. 120° Cho hinh lap phuong ABCD.EFGH. Hay xac dinh gc giifa céc cap vecto sau u day: a, AB va EG A. 45" B. 60". Cc. 90". D. 120". A. 45". B. 60°. C. 90" D. 120". A, 45°. B. 60) Cc. 90". D. 120". oO 21: Trong khong gian cho hai tam gide déu ABC va ABC’ 6 chung canh AB va nam trong hai mat phang khdc nhau. Goi M,N. P. Q Kin luot 1a trung diém ctia cée canh AC. CB. BCC. a. Hay xde dinh géc gia AB va CC. A, 45". B. 60". C. 90". D. 120". b. Ti gidc MNPQ Ia hinh gi? A. Hinh thang. C. Hinh chit nhat. B. Hinh binh hanh. D. Hinh vuong Bai 22: Cho tir dien ABCD. a. Tim gid tri ca k thich hop dién vao dking thiic vecto: ABCD + AC.DB + AD.BC =k A. k=0. B. k=. Ck D. k=4. b. Tir ding thie tren hay suy ra ring néu wt dien ABCD c6 AB LCD va ACL DB thi AD 4 BC Bai 23: Trong khong gian cho hai hinh vuong ABCD va ABCD’ cé chung canh AB va nam trong hai mat phang khdc nhau, lan lugt c6 tam O va O'. a. Hay xdcdinh goc gidta AB va OO’. A. 45". B. 60° Cc. 90". D. 120°. b. Ti gidc CDD'C’ Ia hinh gi? A. Hinh thang. C. Hinh chit nhat. B. Hinh binh hanh. D. Hinh vuong. Bai 24: Cho S$ Ia dién tich \ABC. Tim gid tri cla k thich hop dién vao dang yAB AC LAB. §3. DUONG THANG VUONG GOC VOI MAT PHANG 1. KIEN THUC CAN NHO ; 1, DINH NGHIA DUONG THANG VUONG GOC VOI MAT PHANG Dinh li mé ddu: Néu duomg thing d vung goc véi hai dutmg thang cat nhau a va b nam trong mat phang (P) thi nd vudng géc voi moi duéng thang nam trong (P). facatb d |dtavadib Nhu vay: =Vecaluoncddle. 92 Dinh nghia 1: Mot ducng sing gor ta vueng gic voi mot mat phang hi nd rudng goe vai moi duing thang chu tong mat phang do. Dinh li 1: Néu duong thang d vuong géc véi hai dudng thing cat nhau a va b nam trong mat phang (P) thi dudng thang d vudng géc véi mat phang (P). 2. CAC TINH CHAT Tinh chat I: Qua mot diém O cho trude cé duy nhat mot mat phang (P) vuong g6c Vi mot dudng thang d cho trudc. Céch ding: le = Qua O dug dudmg thang dl" // d. . y hai mat phang phan bier (Q) va (Ry cing di qua d°. Trong (Q) dung duong thang a qua 1 O va vudng géc void’. Hong (R) dung duong. thang b qua O va vudng gov void? Khi d6, mat phang (a, b) chinh la mat phang can dung Tinh chat 2: Qua mot diém O cho trude c6 duy nhat mot duong thang d vung goc vGi mot mat phang (P) cho trude. Céch dung: = Lay dudng thing a nim trong (P). Ps) = Dung mat phing (Q) qua O vuong gée véi a cat (P) theo giao tuyen b. fa = Trong (Q) dung dudng thang d qua O va vudng g6c vdi b Khi do, dudmg thang d chinh la duémg thing can dung. 3. LIEN Hf GIU'A QUAN HE SONG SONG VA QUAN Hf: VUONG GOc CUA DUONG THANG VA MAT PHANG Tinh chat 3: a. Cho hai diang thang sone song. Mat phat thang nay thi cing vuong oe voi duwny thany b. Hai dtémg thang phan bict citi voi nhau. Tinh chat 4 a. Cho hai mat phany song song. Pudng thang mio vung go vei mat phing nay thi cing \uone ge vai mat phang his b. Hai mat phang phan bict cung vuong gde Vii mot dudng thang thi song song véi nhau Tinh chat 5: a. Cho dutmg thing a va mat phang (P) song song vii nhau. Duong thing na: wudng ade vet (1 thi cin Porerrc nt b. Neu mot duémg thang va mot mat phans (khong chia dame than: cling vuong géc vii mot dysmg thang thi song Song v1 nhaw nao vuong poe vii done hia goo mot mat phing thi song song uo 93 4. DINH Li BA DUONG VUONG GOC Dinh nghia 2 (Phép chigu vudng géc): Phép chiéi song song tron. dé phutcmg chiéiu vudng goc vai mat chiéu goi la phép chiéu vudn 6c ow Chit y; Phép chiéu vuong géc c6 day di cdc tinh chat cia phép chiéu song song. ( we, Dinh li 2 (Dinh li ba dudng vudng géc): Cho duong thing a cé hinh chiéu twén mat phang (P) la duéng Tt? thang a’. Khi dy, mot duémg thang b nam trong mp (P) al yuong géc véi a khi va chi khi n6 vudng géc véi a’. Tic sal bc(P)ega Lb. za a 5. GOC GIUA DUONG THANG VA MAT PHANG Dinh nghia 3: Goc gitta duémg thang a va mat phdng (P) la g6¢ gitta duing thdng a va hin chiéu a’ cia né trén (P), ki higu 1a (a, (P)) hay (P), a). ge i Dac bier: @ Khia thudc (P) hoae a song song véi (P) thi (a, (P)) = 0! * Khia vuong géc véi (P) thi (a, (P)) = 90" Nhur vay, ta ludn 06 0 < (a, (P)) $ 90 1. BAI TAP TRAC NGHIEM VA TULUAN Bai 25: Khdng dinh "Mot cutmg thing vuong géc véi hai dudng théng phan biet trong mit phing (P) thi né vudng géc voi mp(P)" c6 ding khong ? A. Diing B. Sai. Cho hai duéng thang a, b vi mat phiing (P). Cac ménh dé sau ding hay a. Néva//(P) vab L(P)thib ia. A. Dung. B. Sui. b. Néua//(P)vab Lathib L(P). A. Ding. B. Sai. c. Néua//(P)vaal/binibi//(P). —* A. Ding. B. Sai. d. Neva L (a) vab L athib/(@). A. Ding. B. Sai. Bai 27: Cho diém Sc6 hinh chiéu trén mat phang (P) la H. Voi diém M bat ki trén (P) (M khong tring H), ta goi doan thang SM [a duéng xién, doan hang HM 1a hinh chiéu cia duéng xién d6. Chimg minh ring: a. Hai dudng xién bang nhau khi va chi khi hai hinh chiéu cia chung bing nhau. b. V6i hai dudng xien cho trudc, duéng xién nao dai hon thi c3 hinh chiéu dai hon va ngugc lai, dung xién nao cé hinh chiéu dai hon thi dai hon. Bai 28: Cho tir din ABCD. Tim diém O céch déu bon dinh cua ti dien. 94 Bai 29: Cho hinh ti dign ABCD co AB, BC, CD doi mot vudng géc va AB = a, BC =b,CD=c. a. Tinh do dai AD. A. Va eb +e C. Vat bec? B. Va? tbc. D. V-a+ b. Chi ra diém cach déu A. B.C, D. A. Trung diém cita AB. C. Trung diém cia AD. B. Trung diém tia AC. D. Trung diém cua BC. 30: Cho hinh ti dién OABC cé ba canh OA, OB, OC doi mot vudng géc. a. Chtmg minh tam giée ABC cé ba géc nhon b. Chimg minh ring hinh chiéu H cia diém O trén mat phang (ABC) tring vei tryc tam AABC. c. Chimg minh ring = sto ~ “OH OA? OB? OC? + Cho hinh chép SABC cé SA (ABC) va AABC khong vuong. Goi H vil K Kin Iuot fi trye tm cia cic AABC va SBC, a. Ba dudng thang AH, SK. BC thoa man: A. Doi mot song song. C. Déng quy B. Doi mot chéo nhau. D. Dap dn khac. b. Tinh s6 do clia gée (SC, (BHK)). A. 45" B. 601" C. 90". D. 120’. ¢. Tinh s6 do cita géc (HK, (SBC)). A. 45" B. 60", iC. 908 D. 120". Cho hinh chop S.ABC = b. Goi G 12 trong tam ABC. a. Ching minh rang SG 1 (ABC). b. Tinh SG. 6 ABC la tam gic déu canh a va SA = SB = SC eae 3 3 c. Xét mat phang (P) di qua A va vudng géc vdi dudng thang SC. Tim hé thie lien hé giita a va b dé (P) cat SC tai diém C, nam gitta S va C. A. b>av2. Boa>bv2. C.a 3a € (P): aL (Q). Hé qua 1: a. Néu hai mat phang (P) va (Q) vudng géc vi nhau va A 1a mot diém nam trén (P) thi dudng thang a di qua A va vu6ng géc véi (Q) sé nim trong (P). b. Néu hai mat phing (P) va (Q) vuéng géc vdi nhau thi bat cit duémg thang a nao thudc mat phang (P), vudng géc vdi giao tuyén cla (P) va (Q) sé vudng géc véi mat phang (Q). Hé qud 2: Hai mat phang cat nhau cling vudng géc vdi mat phang thif ba thi giao tuyén cla chting vudng géc véi mat phang thit ba. Hé qud 3: Qua duing thing a khong vudng géc vdi mat phang (P) cé duy nhat mot mat phang (Q) vudng géc voi mat phang (P). 3. HINH LANG TRU DUNG. HINH HOP CHU NHAT. HINH LAP PHUONG Dinh nghia 3: Mot hinh ldng tru duge goi la hinh lang try ding néu cdc canh bén cia nd vudng géc vai cdc mat day. Nhan xét ring cdc mat bén ctia hinh ldng tru ding la nhiing lunh chit nhat vd déu vudng géc voi day. D, c B Cc Di Ic B {eo A B A & B Lang try Lang tru ding Lang tri déu Ta c6 cdc trudng hgp: 1. Mot hinh lang tru ding c6 day la mot mién da gidc déu dugc goi la lang tru déu. Nhu vay, lang try déu c6 cdc mat bén 1a nhimg hinh cho nhat bang nhau. 2. M6t hinh ling tu ding c6 day Ia hinh binh hanh dugc goi la hinh hép dimg. Nhu vay, hinh hop ding c6 bén mat bén 1a nhimg hinh chit nhat va hai day 1a hinh binh hanh. 3. Mot hinh lang try ding cé day lat hinh chit nhat duoc goi la 1.inh hop chit nhdt. Nhu vay, hinh hop chi nhat c6 s4u mat déu [A nhimg hinh chi nhat. 4. Hinh hop cé tat cd cdc mat déu 1a hinh vuéng goi la hinh lap phuong. 97 4. HINH CHOP DEU VA HINH CHOP CUT pEU Dinh nghia 4: Mot hinh chép duge goi la hinh chép déu néiu day ctia n6 la mién da gide déu va chan duéng cao ctia hinh chép tring véi tam ctia da gide déu dé Nhan xét rang cdc canh bén ctia hinh chép déu thi ; bang nhau va cdc mat bén ciia né la nhiing tam gide can bang nhau. Doan thang néi dinh cla hinh chép véi trung diém cia mot canh déy bat ki goi 1a tung doan cua hinh of: AL chép déu. a eas Dinh nghia 5: Mot hinh chop cut duce cat ra tit mot hinh chép déu duce goi la hinh chop cut déu Khi do: a b "Hai day 1a hai da gidic déu va dong dang. Ur = Duong ndi am OO, cua hai day goi 1a duémg cao cla hinh chép cut déu * © Cac mat ben cua hinh chép cut déu la nhing BOC hinh thang can va bang nhau. * Doan thang n6i trung diém cua hai canh day thudc mot mat ben goi La trung doan cia binh chép cut déu. . BAI TAP TRAC NGHIEM VA TULUAN 36: Cac ménh dé sau ding hay sai? Hai mat phang cing vudng géc véi mat phang tht ba thi song song véi nhau : oy A. Ding. B. Sai. b. Hai mat phang cing vudng géc véi mat phiing thit ba thi vung géc v6i nhau A. Ding. B. Sai. Cae ménh dé sau diing hay sai a. Qua mot dudmg thang cho trude c6 duy nhat mot mat phing vuong g6e vi mot mat phing cho trude. A. Ding. B. duy nhat mot mat phang di qua n hai mat phang cat nhau cho truéc A. Ding. B. Sai. c. Cae mat phang cing di qua mot diém cho trude va vudng gde véi mot mat phang cho truéc thi ludn di qua mot dutmg thang cé dinh. A. Ding. B. Sai. ai 38: Céc ménh dé sau dting hay sai ? Hinh lang tru cé hai mat bén 18 hinh chi nhat 1a hinh lang tru ding. A. Diing B. Sai. ai diém cho trude va vudng g6e Voi 98 b. Hinh chép c6 day a da gidc déu va ba canh ben bing nhau 1a hinh chép déu. A. Ding. B. Sai Cho hinh hop ABCD.A'BICD' cé AB = a, BC = b, CC’ = ¢, Néu AC = 'D = Va’ +b? +c? thi hinh hop dé cé6 phai [8 hinh hop chi nhat khong A. C6. B. Khong. Bai 40: Trong cdc ménh dé sau day. hay tim ménh dé diing ? A. Hai mat phang phan biét cling vudng géc voi mot mat phang thir ba thi song song véi nhau. B, Néu hai mat phing vudng géc véi nhau thi moi duéng thing thudc mat phang nay sé vudng géc véi mat phang kia. C. Hai mat phang (a) va (B) vudng géc véi nhau va cat nhau theo giao tuyén d. Vi mdi diém A thudc («) va méi diém B thudc (B) thi ta c6 dutng thing AB vuong géc vdi d. D. Néu hai mat phang (a) va (B) déu vudng géc voi mat phang (y ) thi giao tuyén d cia (a) va (B ) néu c6 sé vudng géc véi (y). Bai 41: Trong cc ménh dé sau day ménh dé nao 1a dting ? A. Hai dudng thang phan biét citing vung géc véi mot mat phang thi song song. B. Hai mat phang phan biét cing vudng géc voi mot mut phang thi song song. C. Hai dudng thang phan biét ciing vudng géc voi mot dudng thing thi song song. D. Hai dutmg thing khong cit nhau va khong song song thi chéo nhau. 2: Trong céc ménh dé sau, ménh dé nao [a ding ? A. Hai dudng thing phan biét ciing song song véi mot mit phang thi song song véi nhau. B. Hai mat phing phan biét cung vudng géc véi mét mat phng thi cat nbau. C. Hai dudng thang phan biét ciing vudng géc vi mot dudng thang thi vudng géc véi nhau. D. Mot mat phang (a) va mot duéng thing a khong thudc (a) cing vudng géc voi dudng thang b thi (a) song song véi a. Bai 43: Cho hinh hop chit nhat ABCD.A’B'C’D’ cé AB =a, BC = b, CC’ ‘Chimg minh ring mat phing (ADC’B’) vudng géc voi mat phing (ABB’A’). b. Tinh do dai duémg chéo AC theo a, b,c. A. AC = Va? +b? +c? C. AC = va? +b? -c?. B. AC = Va? -b? +c? . D. AC = y-a? +b? +07. Bai 44: Tinh do dai dudng chéo cua mot hinh lap phuong canh a. A. av2. B. ay3 C. 2a, D. av5. Bai 45: Cho hinh lap phuong ABCD.A'B'CD'cé canh bang a. a. Chimg minh ring AC vudng géc véi hai mat phing (A'BD) va (BCD b. Cat hinh lap phuong béi mat phang trung truc cla AC’. Thiét dién 1a hinh gi? A. Tam gidc déu. C. Nagi gidc déu. B. Hinh vuong. D. Luc gidc déu. c. Tinh dién tich thiét dién dé. a 28 i B. a’, D. a Bai 46: Cho hinh chép S.ABCD cé day ABCD [a hinh vudng canh a va SAL (ABCD), SA = x. Xéc dinh x dé hai mat phing (SBC) va (SCD) tao véi nhau géc 60". a 3a A. x==. B. x=a. Cc. x D. x= 2a. Bai 47: Cho hai mat phing vung géc (P) va (Q) cé giao tuyén A. Lay A, B cling thuge A va lay C © (P), D € (Q) sao cho AC L AB, BD 1 AB va AB=AC=BD. a. Thiét dign cia ti dién ABCD khi cit boi mat phing (cx) di qua diém A va vung géc véi CD [2 hinh gi? ‘A. Tam gide vuong. C. Tam gidc déu. B. Tam gidc can. D. Hinh vuong b. Tinh dién tich thiét dién khi AC = AB = BD =a. a2 2 SC ae Oe 12 8 8 12 Bai 48: Hinh hop ABCD.A‘BCD' [a hinh hop gi néu thod man mot trong cde diéu kién sau: a. Titdién ABCD cé cc canh déi bing nhau. A. Hinh hop thoi. C. Hinh lap phuong. B. Hinh hop chit nhat. D. Dap s6 khéc. b. Titdién ABCD’ c6-cdc canh doi vuong géc. A. Hinh hop thoi. C. Hinh lap phuong. B. Hinh hop chit nhat. D. Dap s6 khac c. Tidién ABCD Ia ti dien déu. A. Hinh hop thoi. C. Hinh lap phuong. B. Hinh hop chit nhat. D. Dap s6 khic. Bai 49: Cho hai mat phang («) va (B) vung géc véi nhau. Ngudi ta lay trén giao tuyén A cia hai mat phang dé hai diém A va B sao cho AB = 8cm. Goi C 1a mot diém trén (a) va D 1a mot diém trén (B) sao cho AC va BD cling vudng g6c Véi giao tuyén A va AC = 6cm, BD = 24cm. Tinh do dai doan CD. A. 30cm. B. 26cm. C. 22cm. D. 20cm. Bai 50: Cho hai mat phang (a), (B) cat nhau va mot diém M khong thudc (a) va khong thudc (B). a. Qua diém M cé bao nhiéu mat phang (P) vudng géc voi («) va (B). A. i. B. 2. Cc. 3. D. Vo 36. b. Néu (a) song song véi () thi két qua trong a) sé thay déi nhu thé nao ? A. 1. B. 2. Cc. 3. D. V6s6. Bai 51: Cho hai AACD va ABCD nam trén hai mat phang vudng géc voi nhau Bi CD = 2x. Goi I, J lan luot 1a trung diém cic AB va CD. C. AB= Va? +x? =vav . B. AB= 2(a?- x’). D. AB b. Tinh IJ theo a va x. A. U= Iya? B. y= Ves), D. U= c. Véi gid tri nao ciia x thi hai mat phng (ABC) va (ABD) vudng géc ? a4, B. 2, c 82 op, a3. 3 2 7 Cho hinh chép S.ABCD c6 day ABCD 1a mot hinh thoi c6 eanh a va c6 = SC =a. Chimg minh ring: a. Tinh géc gidta hai mat phang (ABCD) va (SBD). A. 30", B. 45°. C. 60". b.Xéc dinh dang cia ASBD A. Tam gidc vudng. C. Tam gi B. Tam gidc can. D. Tam gidc déu. Cho hinh chép tit gidc déu S.ABCD c6 cdc canh bén va cdc canh déy déu bang a. Goi O 1a tam cia hinh vuéng ABCD. a. Tinh d6 dai doan thang SO. af, ay2 c. 3 p. 3 ; sey Pak 3, b. Goi M [a trung diém cia doan thing SC. Tinh géc gitta hai mat phing (MBD) va (SAC). A. 30°. B. 45”. Cc. 60". D. 90°. 101 c. Tinh 49 dai doan OM. A. 2. av3 2 B. . a . D. d. Tinh géc giita hai mat phing (MBD) va (ABCD). A. 30°. B. 45". C. 60". D. 90". 54: Cho hinh chép S. ABCD cé day ABCD A mot hinh thoi tam I canha va c6 ave goc A =60",canh SC= SS* va SC wong g6e v6i mat phiing (ABCD). a. «Tinh géc giita hai mat phing (SBD) va (SAC). A. 30". B. 45°. iC. 60;. D. 90". b. Trong tam gidc SCA ké IK vudng géc voi SA tai K. Hay tinh do dai IK. A. 2. av2 av3, p. 2. 5 B. c. 5. A c. Tinh s6 do cla gc BKD. A. 30". B. 45°. C. 60°. D. 90". Bai 55: Cho AABC va mat phing (P). Biét géc gitta mp(P) va mp(ABC) 1a ¢, hinh chiéu ca AABC trén mat phang (P) la AA'BC.. Tim hé thtéc lién-hé gitta dién tich AABC va dién tich AA'BIC. As Syge = Sage-Sing. C. Syac = Saac-tang. Be Syuc = Sage-Coso. D. Sype = Sage-COl. 2 « § 5. KHOANG CACH I. KIEN THUC CAN NHO 1. KHOANGCAGHTUMOT DIEM DEN MOT MAT PHANG, BEN MOT DUONC THANG Dinh nghia 1: Khodng cach tit diém M dén mat phdng (P) (dén dung thang a) Ia khodng cach giita hai diém M va H, trong dé H la hinh chiéu vuong sc cia diém M trén mat phdng (P) (trén duong thang d). 2. KHOANG CACH GIU'A DUONG THANG VA MAT PHANG SONG SONG , GIA. HAI MAT PHANG SONG SONG Dinh nghia 2: Khodng cach gitta duéng thang a va mat phdng (P) song song voi ala khodng cach tit mot diém nao dé ctia a den mat phang (P). Dinh nghia 3: Khodng cach gitta hai mat phang song song 1a khesing cach tie mot diém bat ki ctia mat phdng nay dén mat phang kia. 3. KHOANG CACH GIU/A HAI DUONG THANG CHEO NHAU Dinh Ii: Cho hai dutng thang chéo nhau a va b, luén cé duy nhat mot duémg thang d cat cd a va b, va vudng géc voi méi dudng thing ay. Dusmg hang d duge goi la duéng vudng géc chung clia a va b. Dinh nghia 4: Khodng cach gitta hai duéng thang chéo nhau la dé déi doan vudng géc chung cia hai dung thdng do. 102 I. BAI TAP TRAC NGHIEM VA TU LUAN Bai $6: Cac ménh dé sau ding hay sai ? a. Dudng thing A [a dudng vudng géc chung ctia hai dubng thang a va b néu 4 vudng géc vdi a va A vudng géc véi b. A. Ding. B. Sai. b. Goi (P) la mat phang song song véi ca hai dudng thang a, b chéo nhau. Khi d6, dudng vudng géc chung A cla a va b Judn ludn vesng géc voi (P). A. Ding. B. Sai. c. Goi A 1a duéng vudng géc chung cia hai duéng thang chéo nhau a va b thi A 1a giao tuyén cila hai mat phang (a, A) va (b, A). A. Ding. B. Sai. d. Cho hai dudng thing chéc nhau a va b. Dudng thing nao di qua mot diém M trén a déng thdi cat b tai N va vudng géc voi b thi dé 1a dudng vudng géc chung cla a va b. A. Ding. B. Sai. e. Dudng vudng géc chung A ciia hai dudng thing chéo nhau a va b nim trong mat phing chifa duéng nay va vudng géc véi dutng kia. A. Ding. B. Sai, Bai $7: Cho hinh lap phuong ABCD.A"B’C’D’ canh a. a, Ching minh ring cdc khoing cach tir cdc diém B,C, D, A’, B’, D’ dén dudng chéo AC’ déu bing nhau. b. Tinh khoang céch tir C dén AC. av6 ay 3° an. 3 : Cho hinh chép tam gide déu S.ABC cé canh day bang 9a, canh ben bang 2a. Tinh khoang cach tir S t6i mat day (ABC). A. a. B. 2a. C. 3a. D. 4. Cho wf dign déu ABCD canh a. Tinh khoang céch gitta hai canh d6i A. B. vp. 2, a 103 Bai 61: Cho hinh hop chit nhat ABCD.A’B'C’D' c6 AB=a, BC = b, CC’ =. a. Tinh ‘shoang céch tir B dén mat phing (ACC’A’). no eo 2ab __ Bab dab 4 . Be. . Va? +b? va? +b? Var +b? va? +b? b. Tinh khoang cach gitta hai duong thing BB’ va AC’. i ab 2ab c 3ab __4ab__ Vase Vata Var? | Vabab? Bai 62: Che hinh lang tru ABC.A’BCC cé tat cd cdc canh déu bang a. Géc tao béi canh bén va mat phang day bang 30”. Hinh chiéu H cia diém A tén mat phang (A'B'C) thudc dudng thang BC. a. Tinh khoang céch gitta hai mat phing day. a. 2. B. 2. c 82. 3 2 2 b. Tinh khodng céich gitta hai duémg thing AA’ va BC. a a a2. B. ¢, a3 | 5 5 cS D. Bai 63: Cho hinh lap phuong ABCD.A'BCD’ cé canh bang a. ‘Tinh khoang cach gitfa hai dung thing BC’ va CD’. x = b — c — D. 3, Bai 64: cho hinh lap phuong “ABCD. A’B'C’ D c6 canh bing a. Tinh khoang cdch ctia hai dudng thang BD’ va B’C. Bai 6S: Cho hinh hop chit nhat ABCD.A'BCD'cé AB= AA' =a, AC = 2a. a. Tinh khoang cach tir diém D dén mat phing (ACD). eel 5 5 3 3 b. Tinh khoang céch gitta hai dudng thing AC’ va CD’. A. 4. B. =. Cc. i Bai 66: Cho hinh hop thoi ABCD.A'BCD' cé cdc canh déu bing a va BAD = BAA’ = DAA’ = 60°. Tinh khoang cach gitta hai mat phing day (ABCD) va (ABCD). 3 3 Bai 67: Cho hinh chép S.ABCD day ABCD Ia hinh chit nhat va AB = 2a, BC = a. Céc canh bén ciia hinh chép bang nhau va bing a-V/2 . 104 a. ‘Tinh khong céch tir S dén mat pnang day (ABCD). av2 ay2 4 2 F [dn Iugt 1a trung diém ciia cde canh AB va CD; K [a diém bat ki thude dugng thing AD. Hay tinh khosing céch gitta hai duéng thing EF va SK theo a. a p — = 2 5 a a Bo cok p, 241 3 3 5 7 Bai 68: Hinh chop S.ABCD co day [a hinh thoi tam O canh a va c6 géc aay : 34 BAD = 60", Dudmg thang SO vuong géc véi mat phang (ABCD) va_ SO = a ' Goi E la trung diém cia doan BC, F la trung diém ciia doan BE a. Tinh géc gita hai mat phang (SOF) va (SBC). A. 30". B. 45" C. 60". D. 90". b. Tinh khoing cach tir O dén mat phing (SBC). : a are A=. B. = Cc. aa p. 2 8 4 4 a c. Tinh khoiing cach tir A dén mat phing (SBC). 42 : “i : A 22. n=. c. 3 co. = 2 3 3 BAI TAP LAM THEM Bai 69: Cho hinh ti dién ABCD cé trong tam G. Ménh dé nao sau day 1a sai ? OA +OB+0C + OD) A. OG ( — 4. se ~ ” B. GA+GB+GC+GL 0. c. AG = 2(AB+AC+AD) D. AG FAB +AC+ AD). Bai 70: Cho hai duémg thing phan biét a, b va mat phiing (P), trong dé a 1 (P). énh dé nao sau day 1a sai ? A. Néub//(P) thi b La. B. Néub 4 (P) thi b//a. C. Néu b//a thi b 1 (P). D. Néub La thi b //(P). Bai 71: Tim ménh dé dung trong cdc ménh dé sau: A. Hai dudng thang cing vung g6c véi mot mat phang thi song song. B. Hai dudng thang phan biét cing vudng géc vi mot dutmg thang thi song song. C. Hai mat phang phan biét ciing vudng g6c voi mot dutmg thang thi song song. D. Hai mat phang phan biét ciing vuéng géc voi mot mat phang thi song song. 105 2: Ménh dé nao sau day 1a ding ? . Hai mat phang vudng géc vdi nhau thi moi dudng thany, nam trong mat phang nay sé vudng géc vdi mat phang kia. B. Hai mat phang phan biét cing vudng géc véi mot mat phang thi vudng géc vi nhau. C. Hai mat phang phan biét cing vudng géc véi mot mat phang thi song song véi nhau. Ba ménh dé trén déu sai. Trong cdc ménh dé sau, ménh dé nao ding? A. Cé duy nhat mot dudng thang di qua mot diém cho truée va vudng goc voi mot dudng thang cho trudc. B. Co duy nhat mot mat phang di qua mot dung thang cho trudc va vung géc v6i mot mat phang cho trudc. C. C6 duy nhat mot mat phing di qua mot diém cho truéc vi vudng géc vei mét mat phang cho truéc. D. C6 duy nhat mot mat phang di qua mot diém cho trudc va vudng géc v6i mot dudng thang cho trudc. Bai 74: Tim ménh dé diing trong cdc ménh dé sau: A. Néuhinh hop ¢6 hai mat la hinh chit nhat thi no 18 hinh hop chit nhat. B. Néuhinh hop cé ba mat Ja hinh chit nhat thi né [a hinh hop chit nhat. C. Néu hinh hop co bon mat [a hinh chit nhat thi n6 [a hinh hop c hi nhat. D. Néu hinh hép cé nam mat Ja binh chit nhat thi né Ja hinh hop chit nhat. Bai 75: Trong cdc ménh dé sau, ménh dé nao ding? A. Néu hinh hop cé fai mat la hinh vung thi no 1a hinh lap phvong. B. Néu hinh hop c6 ba mat chung mot dinh [a hinh vuéng thi no [a hinh Jap phuong. C. Néu hinh hop co sdu mat bang nhau thi n6 [4 hinh lap phyong. D. Néu hinh hop c6 bén dutmg chéo bang nhau thi né [a hinh lap phuong. Bai 76: Cho hinh chép S.ABC co day ABC Ia tam gidc déu. Tim ménh dé ding trong cdc ménh dé sau: A. S.ABC [a hinh chép déu néu cdc mat bén cla n6 [a tam gidc can. B. S.ABC 1a hinh chop déu néu cdc mat bén cita né 1a tam gidc can véi dinh S. C. S.ABC [a hinh chép déu néu géc gitta cdc mat phang crita cdc mat bén va mat phang chifa day bang nhau. D. SABC la hinh chép déu néu cc mat bén cé dién tich bing nhau. Bai 77: Tim ménh dé ding trong cdc ménh dé sau: A. Dung vudny géc chung cia hai dudng thang chéo n'iau thi nam trong mat phing chia duong thang nay va vuong géc véi duéng thing kia B, Duimg vudng géc chung ciia hai dudmg thing chéo nhau thi vung g¢> vGi mat phang chita duéng thing nay va song sony voi dudng thang kia. C. Dudng vudng géc chung cia hai dudng thing chéo nhau néu né vuong géc vdi cd hai dudng thing 46. D. ménh dé trén déu sai. Bai78: Hinh ti dién ABCD cé AB, AC, AD doi mét vudng géc va AB= AC = AD = 3. Dién tich ABCD bang: 9 9 27 . au A a C. 27. vp. 2 Bai 79: Hinh hop chit nhat ABCD.A'BCD' c6 AB = AA’ = AD = a va AAB = A'AD = BAD = 60°. Khi d6, khoaing cach gitta céc duémy thing chia céc canh d6i dien cua ti dién A’ABD bang: C. av2. p. 3 A. ay2 . B. 2. . 2 2 2 80: Titdi¢n OABCc6 OA = OB= OC=ava AOD = AOC = 60, BOC =90'. a. Chitg td ring AABC [a tam gidc vudng va OA 1 BC. b. Tim cutmg vudng géc chung IJ cila OA va BC, tinh khoang cach gitta hai dudng thang OA va BC. c. Chtg minh ring hai mat phing (ABC) va (OBC) vudng géc véi nhau. 81: Cho hinh chop S.ABC c6 SA = SB = SC = a, ASB= 120°, BSC = 60°, CSA = 90". a. Ching to rang AABC 1a tam gidc vuong. b. Tinh khoang céch tir S dén mat phang (ABC). Bai 82: Cho hinh chép S.ABCD cé day Al 1a hinh vudng canh a, SAL (ABCD). Hai diém M va N lan lugt thay déi tren hai canh CB va CD, dat CM =x, CN=y. Tim hé thtic lign hé gidta x va y dé: a. Hai mat phang (SAM) va (SAN) tao véi nhau géc 45". b. Hai mat phang (SAM) va (SAN) vudng géc véi nhau. Bai 83: Tam gidc ABC vudng cé canh huyén BC nam trong ma phing (P), canh AB vi AC lan lugt tgo v6i mat phing (P) céc géc B va y. Goi a la géc tao boi mat phang (P) va mat phing (ABC). Ching minh rang: sin’o. = sin’B + sin’y. Bai 84: Cho ut dién OABC cé OA, OB. OC doi mot vudng géc véi nhau va OB = b, OC = c. Goi H [a hinh chiéu cua O trén mat phang (ABC). Tinh dién tich tam gidéc HAB, HBC, HCA. 107 Bai 85:_Cho hinh lang tru dimg ABC.A'BC cé day ABC IA tam gidc vuong tai dinh C, CA = a, CB = b; mat ben ABB'A’ IA hinh vuong. Goi (P) la raat phang di qua C va vudng géc véi AB’. a. Xéc dinh thiét dign cia hinh lang tru da cho khi cat béi (P), Thiét dign 1a b. Tinh dién tich thiét dién noi trén. MOt tit dién duoc goi la gén déu néu cdc canh déi bang nhau timg doi mot. Voi ti dién ABCD, ching to cdc tinh chat sau 1a tuong duong: a. Tidién ABCD 1a gan déu. b. Céc doan thang néi trung diém cap canh déi dién doi mot vudng géc voi nhau. c. Céc trong tuyén (doan néi dinh véi trong tam cla mat déi dién) bang nhau. d. Téng cdc géc tai méi dinh bing 180”. Bai Cho tr dién ABCD. Goi M, N lan lugt Ja trung diém cua BC va BD; P 1a mot diém thay doi trén doan thang AD. a. Xéc dinh giao diém Q ctia mat phing (MNP) va canh AC. Tit gide MNPQ 1a hinh gi? b. Tim qui tich giao diém I cla QM va PN. c. Tim quy tich giao diém J cla QN va PM. Bai 88: Cho hinh hop ABCD.A'B'C’D’. Diém M nam giita A va D. diém N nam gia C va’ sao cho AM CN MD NC a. Chimg minh rang dung thing MN song song voi mp(ACB’). b. Xac dinh thiét dien cUa hinh hop Khi c&t boi mat phang di qua MN va song song véi mp(ACB’). Bai Cho hinh chép S.ABC. Goi K va N lan luot 1a trung diém cia SA va BC, M [a diém nam gidta S va C. a. Chimg minh rang mat phing di qua K, song song véi AB va SC thi di qua diém N. b. Xac dinh thiét dien cia hinh chdp S.ABC khi cat béi mat phing (KMN). Chimg t6 ring KN chia thiét dién thanh hai phan cé dién tich bang nhau. Bai 90: (Tr 126): Hinh chép tr gidc déu S.ABCD cé canh day ba 1g a va canh bén bing av2. a. Tinh khodng cach tir S dén mp(ABCD). b. Tinh khodng cach giita duéng thing AB va mp(SCD). c. Tinh khoang cach gitta hai duéng thing AB va SC. d. Goi (P) la mat phing di qua A va vuong géc voi SC. Hay xdc dinh thiét dién cia hinh chép Khi cat béi (P). Tinh dién tich thiét dién. e. Tinh géc gitta dudng thing AB va mat phang (P). 108 DAP SO TRAC NGHIEM - LOI GIA TULUAN Dap sé trdc nghiém a). Azb). A. Al B Dap s6 tréc nghiém a). B; b). B;c). B. - D, © a. Sit dung quy tic ba diém, ta c6: AC = AB+BC = AB+BC+CC = AB+BC+DD' : Vay, véi k= | thod man hé thie. ! : b. Tic6: 2 c VT = BD -(B'D'+ D'D) = BD-B'D = BD+ DB = BB’. Vay, vi k = 1 thoi man he thie, yy thé bai céc vecto bing nhau, ta duoc: AC+CD'+DB4+BA = AD'+D'A = AA k = | thoa man he thi Dap sé ac nghiém a). A:b). A. iM a. Te kin lugt ¢6 hai cach biéu dién: MN =MA + AD+ DN. ay D B MN = MB+BC+CN. (2 Cong theo vé (1) va (2), ta duoc 2MN = es eee c <> MN LaD+ BC). Vay, voi k = ; thoa man hé thtéc. b. Te kin lugt ¢6 hai cach bigu dién: MN =MA + AC+CN. 3) MN=MB+BD+DN. (4) Ceéng theo vé (3) va (4), ta duge: 2.MN = (MA + MB) +(AC + BD) + (CN + DN) = AC + BD pac + BD). Vay. véi k= — thos man he thie. Y tric nghiém C. D ng lugn: Ta Kin luot 66: a) 2 —— A DC=DG+GC. @) c Céng th theo ve (1) (2) va (3), ta duge: DA + DB+DC 3DG +(GA+GB+GC) = 3DG. Viy, véi k = 3 thoa man hé thitc. 109 Dap s6'mreic nghiém a). B:b). D. a. Ta lan lugt c6: IA+1C= 21M. (1) Ic + ID= 2IM (2) Cong theo vé (1) va (2), ta duge: 1A + 1B +IC+ID= 20M + IN) =0. Vay, véi k = I thoa man hé thic. A b. Ta lan lugt cé: QB) M (4) PC=PI+IC (5) D Cc PD=PI+ID. (6) Cong theo vé (3), (4). (5) va (6), ta duge: N PA +PR+PC+PD = Seite ICD) B © PI iA +PB+PC+ PD). Vay, v6i k = — 7;™ man hé thitc. Bai 6: a. Néu ABCD 1a hinh binh hanh voi tam O thi SB+SI =2SO, SA +SC=2S0, tird6, suy ra: SB+SD = SA+SC. Diéu ngugc lai ciing dung, th: SB+3D = SA+SC < SB-S, © ABCD 18 hinh binh han, b. Trude tién, vi O 12 giao diém cua AC va BD nen: = 0,A,C thing hing, suy ra 3m: 02 moc. * 0,B,D thing hing, suy ra 3n: OB = nOD. Khi d6; 480 = SA + SB+SC + SD = (SO + 0A) + (SO + OB) + (SO +. 0C) + (SO + OD) = 480 + (OA + 0C) + (OB + OD) SC-SD < AB=DC n+1=0 © O [a trung diém cia AC va BD <> ABCD [a hinh binh hanh. = (m+1)OC +(n+1)OD = 0 of ={ a. Véi diém M thudc phing, suy ra 3 y, z sao cho: AM = | =yAB +2AC ¢ OM - OA = y(OB - OA)+20C- OA) > OM=(l-y-2)0A-+ yOB + 20C Dat x = Ty = 2 (ie 1a x+y +2= 1) thita duge: OM =xOA + yOB+z0C. phing (ABC) thi ba vecto AB, AC, AM déng 110 b. Naugc lai, ta bién déi: OM =xOA + yOB + 20 = (1—y -z)OA + yOB + zOC = OM - OA | COB ~ OA)+210C- OA ) = AM =yAB +zAC => ba vecto AB, AC, AM déng phing = M € (ABC). Bai 8: Dép so trac nghiém C. Loi giai ae Goi Gla trong tam AABC, ta sOX,y,Zmax+y+ sao cho: xSA + ySB + 2SC = SG A’ B 6 ide nghiém a). D:b). B Nee hinh binh hanh, ta cé: BC = BC+BB = BC-AA = AC- AB- AA =c-b-a b. Theo quy tac hinh binh hanh, ta c6 BC = BB'+B'C = AA'+BC = AA'+AC-AB =a+c-b. Bail: Dap sé trac nghiém C. Loi gidi tue ludn: Ta lan luot c6: a. V6i (A) thi tir, AB = 3AC < - BA =~3CA , do d6 (A) Ia sai. b. VGr(B) thi tir’ AB=-3AC <> AC-AB=2AC @ BC=2AC Do ds (B) 1a sai. c. Véi (C) thi tir, AB = -2AC+5AD © ba vecto AB, AC, AD déng phing <> bon diém A, B, C, D cling thudc mot mat phing . Do d6 (C) 1 ding. 4. Voi (D) thi ti: AB =-38C <> 2BA = BC = A nim gitta BC => Bkhong thé 1a trung diém cia doan AC. Do dé (D) 1a sai. i Dap sé trac nghiém D. ue ludn: Ta lan luot cé: a. Voi (A) thi N diing [a trung diém cia doan MP, do do (A) ding. b. V6i (B) ding vi dé chinh 1a quy tac trung diém. WL c. Voi (C) ding vi thoa man dinh Ii vé su déng phang cia ba vecto trong khong gian. 4. V6i (D) thi bing quy te ba diém ta nhan thay ding thie: AB + BC + co + DA dting v6i moi diém A, B, C, D nén cing diing véi ttrdién ABCD. Do dé (D) la sai. Bai 12: Ta c6 thé trinh bay theo hai céich sau: s Céch 1: Ta lea lugt c6: MN = MA + AB + BN Mi <2 2MN =2MA + 2AB +2BN a) A MN =MS+SC+CN. Q) “xX ‘Cong theo vé (1) va (2), ta duge: N 3 MN =(2MA + MS) + 2AB + SC + (2BN + CN) 0 0 © MN = 2AB+ 556 <> AB, MN, SC déng phing. Céch 2: Lay diém E trén SB sao cho SE = 2EB, ta c6: SM _ SE y= AB// EM c (MNE), MA EB /n|=0 digu d6 mau thudn voi gid thiet. Vay, ba vécto n, a va b khong déng phiing. 112 b. Véiba vecto a, b, c ¢ cing vuong géc vGi vecto a. Khi d6, néu a va b khong cing phuong thi n, a va b khong déng phang, tic 14 ton tai ba s6 x, y, z sao cho: e=xn +ya+zb => ¢.n =xn.n +ya. n+zb.n =0= xn? >x=0. Tic 1a, ta c6 biéu dié a +2b => ba vecto a, b, ¢ déng phing. Tir d6 suy ra, céc dutmg thing cling vudng géc voi mot dutmg thi cling song song véi mot mat phang. Bai 17: Dap 86 trdc nghiém A. LP) Loi gidi ne ludn: Ta 6: (AB, EG) =(AB, AC) = BAC=45", AB.EG = | AB|.| BG | .cos( AB, EG) a.ay2 .cos45" = a’, do.dé (A) la sig, Bail8: Dap 96 trdc nghiém a). C;b). C;0). C. a. Tacé: ABCD = AB(AD- AC) = ABAD- AB.AC B*.cos60" — AB.cos60" = 0 = (AB, CD) = 90". b. Tirgid thiet, suy ra AABC, AABD déu. Ta cé: AACD = ABCD (c.c.c) => JA = fone eee JL AB. => (AB, l)= 90". c. Tacé: AABD = AABC (c.c.c) => ID = IC = AICD can tai I > I 1 CD $s => (Li, CD) = 90". Dap s6 tréc nghiém a). C:b). C;c). C. a. Tacé: SABC = SASC- BB) = AS-AB K = SA2.cos CSA - SA?.cos ASB =0 . c (SA, BC) =90". ; Ching minh tuong ty, ta cing nhan duge: (SB, AC) = 90", (SC, AB) = 90°. Bai20: Dap sé trdc nghiém a). A; b). Byc). C. G a. Tac: (AB, EG)=(AB, AC)= BAD =45", bdi ABC vudng can tai B. b. Tacé: (AF, EG) =(AF, AC)= bdi AAFC [a tam gidc déu. c. Tacé:(AB va DH)=(AF, AE) = BAE= 90", boi AFE Ia hina vudng. FAC =60", ES) 113 Bai 21: Dap sé trae nghiém a). C:b1. a. Goia li canh cia tam gide déu. ta AB.CC = AB(AC- AC) = ABAC-ABAC | (AB, CC ) = 90". b. Nhan xét ring: u“ i MN = 1B va PQ QMN = (MN, MQ) = (AB, (2) 1) va (2) suy ra MNPQ [a hinh chit nhat. ; dic nghiém A : Ta Kin luot c6: = AB(AD~- AC) a) = AC(AD ~ AB) (2) AD.BC = AD(AB - AC) @) Cong theo vé (1), (2) va AB.CD + AC.DB + AD.BC =0. dpem: b. AB LCD va AC 1 DB thi 3), ta duge: — —— —— Cc AB.CD =0 va AC.DB =0= AD.BC = AD 1 BC. Bai 23: Dap sé trac nghiém a). Cb). C. = AB LOO. dpem. b. Nan xét rang: " w w CD = ABvaCD' = AB=CD=CD. (1) DCC = (DC. CC’) = (AB, 00') = 90", (2) Tir (1) va (2) suy ra CDD'C’ Ia hinh chi nhat. 114 !ABI.IACI thoa man hé thitc. Léi gidi ne ludn: Khang dinh tren ia sai vi cn thém diéu kien hai dudng thing 46 pha ct nhau dc nghiém a). A:b). B;c). B;d). B. a. Di ne boi: a // (P) => Ja’ c (P) sao choa//a',b 1 (P)=>b La’. Khi dé: g(a, b) = g(a’, b) = 90" >a L b. b. Sai, boi khi dé b c6 thé nam trong (P). c. Sai, béi khi dé b c6 thé vudng géc véi a. d. Sai, béi khi dé b cé thé nim trong (a). Bai Huong dan: Str dung tinh chat bing nhau cia tam gidc vudng. Goi d 1a duéng thang di qua tam I dudng tron ngoai tiép AABC va yuéng goc voi (ABC). Mat phang trung truc cua doan AD cat d tai O thi O cach déu bon dinh cia ti dién, that vay trudc tién ta c6 ngay: OA =OD. qd) Mat khéc: AOIA = AOIB = AOIC (c.g.c) > OA = OB = OC. (2) Tir (1) va (2), suy ra: OA = OB = OC = OD tue 1a, diém O cach déu bon dinh cua tit diés Dap sé trdc nghiéma). A;b). C. CD LAB . Tacs: CD L(ABC)=> CD .L AC=3 ACD vudng tai C a {epiee : = => AD? = AC’ + CD? = AB’ + BC’ + CD? =a +b? +c? . = AD= Va? +b +c2. b. Goi O [a trung diém ciia AD. = ViAACD vudng tai Cnén: OA=OC=OD. (1) 11S CD1LAB Ta cé: ABL ac {rc tan (BCD) A => ABLBD= AABD vudng tai B Oo = OA = OB=OD. (2) Vay, diém O cach déu A, B, C, D. B D Bai 30: a. Gid sit OA =a, OB =b, OC =c. C Xét AABC wudng tai O, ta cd: AB? = OA? + OB? =a?+b?, BC’ =OB?+OC?=b* +c’, AC’ = OA? + OC =a? +2, “ aap ABT+AC2-BC? a? +b? +a? +07? -(b? +07) 0s BAC = ———_"—__—__ = — — a ee 5 : 2ABAC ead = g6c BAC nhon. Chimg minh tuong ty, ta duge céc géc ABC, ACB déu nhon. Vay, cdc géc ciia tam gide ABC déu nhon. b. Tirgid thiét: OH (ABC) = OH 1 BC. @ (OA LOB - Ta cb: OALO OA LBC. 2 7 {on toc? ae @ Ti (1) va (2) suy ra BC L (OAH). Chimg minh tuong ty ta nhan duge CA 1 (OBH). Tacé: BC L (OAH) =>BC L AH. 3) AC L (OBH) > AC 1 BH. @) Tir (3) va (4) suy ra H [a truc tam cia ABC. A c. Gia sir AH ct BC tai K, suy ra OK BC. = Trong AOBC vudng tai O, ta cé: 1 1 OK? OB? OC? . " Trong AOAK vudng tai O, ta c6: a K 1 1 1 1 1 — = — —} , dpem. Oa? * OK? ~ OA? OB Oc “P° Bai 31: Dap s6 trac nghiém a). C;b). C;c). C. ; BC LAE a. Goi {E} = AH BC, ta c6: = BC L (SAE) > BC J. SE BC LSA => SE la dutng cao cla A SBC => K € SE. Vay, ba duong thing AH, SK, BC déng quy tai E. BH LAC . 6: L b. Taco: {prten => BH | (SAC) = BH 1 SC. qd Mat khic, ta cé: BK 1 SC. (2) Tir (1) va (2), suy ra: SC 1 (BHK). “) 116 c. Tir(*), suy ra SC 1 HK. (3) Mat khac, tira), suy ra BC 1 HK. (4) ‘Tir (3) va (4), suy ra HK 1 (SBC). Bai 32: oi M Ia trung diém BC, ta c6: s BC L CLAM _, BC 1 (SAM) = BC LSG. BC LSM Chimg minh tuong tu, ta cling nhan duge: AB L SG = SG 1 (ABC). B A b. Dap so rae nghiém B. M Léi gidi ne ludn: Trong AGSA vudng tai G, ta c6: c fala ¥ sc? = sa?—Ga? =p? | #49) =pr- \ cc. Dap s6 trae nghiém B. : Dé diém C, nim gitta S va C diéu kign 1a ASAC (can tai S) 24 gc? ac? <> KSC <90" > cosASC > 04 SA_+SC = AC’, g 2SA.SC = 2b? ~ a> 0. a7 > 2b’ a> bv2 d. Dap sé trac nghiém B. Léi gidi ut ludn: Ta c6: SSAC = ASBC (¢.c.c) > SC L B,C = SC 1 (ABC,) => A ABC, can tai C, chinh 1a thiét dien, Pon? Tacé: AC, = SA.sin ASC = SA V1 -cos? ASC =b (5 } 2 = Saase, = Ta hin lugt c6 ABCD = AB(AD-AC) = ABAD-ABAC, (ay AC.DB = AC(AD ~ AB) ‘ ACAD ~ AC.AB, 2 AD.BC = AD(AB- AC) c! D = AD.AB-AD.AC. (3) Cong theo vé (1), (2) va (3), ta duc 0 = ABCD + AGDB+ AD.BC = AD.BC @ AD 1 BC. 117

You might also like